Wikipedia:Teahouse: Difference between revisions
→How to be an effective editor?: new section |
Adding {{pp-sock}} |
||
Line 1: | Line 1: | ||
{{Short description|Community Q&A hub for new editors}}{{pp-sock|small=yes}} |
|||
{{skip to top and bottom}} |
|||
{{User:MiszaBot/config |
{{User:MiszaBot/config |
||
|archiveheader = {{Automatic archive navigator}} |
|archiveheader = {{Automatic archive navigator}} |
||
|maxarchivesize = |
|maxarchivesize = 400K |
||
|counter = |
|counter = 1245 |
||
|minthreadsleft = |
|minthreadsleft = 15 |
||
|minthreadstoarchive = |
|minthreadstoarchive = 25 |
||
|algo = old( |
|algo = old(48h) |
||
|archive = Wikipedia:Teahouse/Questions/Archive %(counter)d |
|archive = Wikipedia:Teahouse/Questions/Archive %(counter)d |
||
}} |
}} |
||
{{clear}} |
|||
{{User:HBC Archive Indexerbot/OptIn |
|||
{{Wikipedia:Teahouse/Header}} |
|||
|mask=Wikipedia:Teahouse/Questions/Archive <#> |
|||
|leading_zeros=0 |
|||
|indexhere=no |
|||
}} |
|||
{{TH question page}} |
|||
<!-- Questions go here. Please post new questions at the BOTTOM of the page. --> |
|||
==How to be an effective editor?== |
|||
How to be an effective editor?16:23, 29 January 2017 (UTC) |
|||
==edit== |
|||
How to edit page and create new pages ? Also , how to change the name of a main article?16:22, 29 January 2017 (UTC) |
|||
==About Speedy deletion nomination== |
|||
Hello, |
|||
== Technical question about the long hyphen == |
|||
Please tell me, is creating an informative page of any shop or firm on wikipedia, according to the terms and policies of wikipedia[[User:Abhishek.moonat|Abhishek.moonat]] ([[User talk:Abhishek.moonat|talk]]) 15:31, 29 January 2017 (UTC) |
|||
:Hi [[User:Abhishek.moonat|Abhishek.moonat]]. It is not. An [[encyclopedia]] is a compendium of articles on topics of knowledge, already substantively written about by people out in the world and is never the place to first discuss topics, that the world has not already recognized. The vast majority of small businesses in the world are not [[WP:N|notable]], as we use that word to define the standard I summarized of previous recognition in writing. Generally, this means that unless [[WP:IRS|reliable]], [[WP:SECONDARY|secondary]] and [[WP:INDEPENDENT|independent]] sources have written about the shop or firm in substantive detail, a stand-alone article is not warranted. Please also be aware of our [[WP:COI|conflict of interest]] guideline, and that people owning or involved in a business, wanting to write about that business, [[template:uw-paid1|must provide disclosure of paid editing]]. Best regards--[[User:Fuhghettaboutit|Fuhghettaboutit]] ([[User talk:Fuhghettaboutit|talk]]) 15:44, 29 January 2017 (UTC) |
|||
Hi! |
|||
==Judgement of Articles== |
|||
How are articles are judged? Wat qualities should an article possess so that it is eligible for posting it worldwide? I have checked out all the article-related Wikipedia pages but i still cannot understand. Please help. |
|||
I've been editing the timeline of Polermo where the long hyphen dominates, but I can't seem to generate one.Typing a regular hyphen, gives me just that - a regular hyphen, typing two hyphens gives me two hyphens (--) and trying to make one through the keboard shortcut which I found on internet forums (Alt+0151), just gives me one that's too long (—). So far I've been copying and pasting existing long hyphens which is kind of annoying, does anyone have any better solutions? |
|||
[[User:Faceless Wikipedian|Faceless Wikipedian]] ([[User talk:Faceless Wikipedian|talk]]) 14:53, 29 January 2017 (UTC) |
|||
:Hi [[User:Faceless Wikipedian|Faceless Wikipedian]]. As threshold matters, an article should: |
|||
# be on a [[WP:N|notable]] topic (as demonstrated by [[WP:REFB|citation]] to [[WP:IRS|reliable]], [[WP:SECONDARY|secondary]] and [[WP:INDEPENDENT|independent]] sources); |
|||
# have only [[WP:V|verifiable]] content; |
|||
# engage in [[WP:OR|no original research]]; |
|||
# be written from a [[WP:N|neutral point of view]]; and |
|||
# not violate any part of [[Wikipedia:What Wikipedia is not]]. |
|||
:Though I think this goes a bit beyond the scope of your direct question, after these threshold matters are met, the path to a great article would be to aspire to meet the [[Wikipedia:Featured article criteria|featured article criteria]]. Best regards--[[User:Fuhghettaboutit|Fuhghettaboutit]] ([[User talk:Fuhghettaboutit|talk]]) 15:30, 29 January 2017 (UTC) |
|||
Thanks! [[User:Moonshane1933|Moonshane1933]] ([[User talk:Moonshane1933|talk]]) 14:38, 22 December 2024 (UTC) |
|||
==My article is not on any Google search== |
|||
I have written an article entitled [[Bill Elliott (musician)|Bill Elliott]] (musician), but the article appears to be transparent on any Google search, even using the exact words. After I was finished with it in my sandbox, I blanked the namespace article by mistake, (instead of my sandbox version) but undid it. The article shows up OK within Wiki, but not outside Wiki. The same is true for Google searching for a keyword within the article. Can you help? Thanks--[[User:Eagledj|Eagledj]] ([[User talk:Eagledj|talk]]) 14:21, 29 January 2017 (UTC) |
|||
:Hello, @[[User:Moonshane1933|Moonshane1933]]. I think you're talking about an em-dash. See [[MOS:EMDASH]] [[User:ColinFine|ColinFine]] ([[User talk:ColinFine|talk]]) 14:52, 22 December 2024 (UTC) |
|||
: Welcome to the Teahouse. A decision was taken some months ago that new pages would be NOINDEXed until they have been accepted by the [[WP:New pages patrol|New pages patrol]] process. This came as a surprise to many editors. There is an increasing queue (currently 15618 pages) awaiting patrol, and a backlog of more than 3 months. --[[User:David Biddulph|David Biddulph]] ([[User talk:David Biddulph|talk]]) 14:33, 29 January 2017 (UTC) |
|||
::Yes! That's what I meant! Thank you! [[User:Moonshane1933|Moonshane1933]] ([[User talk:Moonshane1933|talk]]) 15:15, 22 December 2024 (UTC) |
|||
:I don't think you could find a better character in '''"unicode table"'''. |
|||
:This "[[List of Unicode characters|article]]" is listing the most common characters. <br /> <br /> |
|||
:There are also the "[[Unicode block]]" entry on Wikipedia that can be maybe helpful. [[User:Anatole-berthe|Anatole-berthe]] ([[User talk:Anatole-berthe|talk]]) 14:54, 22 December 2024 (UTC) |
|||
::Excellent. Thank you too! [[User:Moonshane1933|Moonshane1933]] ([[User talk:Moonshane1933|talk]]) 15:16, 22 December 2024 (UTC) |
|||
:::I don't think ressources I shared with you will help you but I hope it will. [[User:Anatole-berthe|Anatole-berthe]] ([[User talk:Anatole-berthe|talk]]) 15:45, 22 December 2024 (UTC) |
|||
:Ignoring the [[Minus sign]], there are three 'horizontal line' characters most commonly used in text, the hyphen, the [[N-dash]] and the [[M-dash]]. There are various ways to insert the latter two; usually I do so with [alt]+0150 and [alt]+0151. Despite being a former professional book editor, I have not previously encountered a "long hyphen" (a term not found anywhere in Wikipedia). Note that the lengths of all these characters may look different in different typefaces: I suspect your "long hyphen" is an N-dash. [Apologies for semi-overlap with answers above.] {The poster formerly known as 897.81.230.195} [[Special:Contributions/94.1.223.204|94.1.223.204]] ([[User talk:94.1.223.204|talk]]) 17:00, 22 December 2024 (UTC) |
|||
==How to replace an existing citation with a more accurate link== |
|||
::@[[User:Moonshane1933|Moonshane1933]] If you use the source editor, which you can do even if you mainly edit with the visual editor, you'll find that the N-dash and M-dash appear at the foot of the editing window, where you can click on them to insert them into text. Other useful tags like <nowiki><ref></ref></nowiki> are also available with a single click. [[User:Michael D. Turnbull|Mike Turnbull]] ([[User talk:Michael D. Turnbull|talk]]) 14:11, 24 December 2024 (UTC) |
|||
How to replace an existing citation with a more accurate link |
|||
:::OOOOOOOHHHH... THANK YOU! That makes life easier! I hadn't even thought of looking at the source editor, because it always looks headache inducing to me. I'll give it a try. Thank you so much. [[User:Moonshane1933|Moonshane1933]] ([[User talk:Moonshane1933|talk]]) 13:07, 25 December 2024 (UTC) |
|||
[[User:Niccasey|Niccasey]] ([[User talk:Niccasey|talk]]) 13:23, 29 January 2017 (UTC) |
|||
::Yes, well, the "long hyphen" is a term that I coined, simply because I lacked the knowledge of its correct name, So I would have been very surprised if it had appeared in Wikipedia. Anyway, thank you, oh mysterious IP poster, I hope our paths cross again! [[User:Moonshane1933|Moonshane1933]] ([[User talk:Moonshane1933|talk]]) 13:03, 25 December 2024 (UTC) |
|||
:::@[[User:Moonshane1933|Moonshane1933]], some Christmas goodies for you: |
|||
:::— [https://www.merriam-webster.com/grammar/em-dash-en-dash-how-to-use Merriam-Webster Dictionary] has a nice clear explanation about the both kinds of dashes and the hyphen, with good examples. |
|||
:::— The way the two kindts of dashes is written is '''em-dash''' (for '''—)''' and '''en-dash''' ( for '''– )''', even though we pronounce the terms "''M dash''" and "''N dash''." |
|||
:::— Why these terns? Because the em-dash is exactly the width of capital ''M'' and the en-dash is exactly the width of capital ''N.'' |
|||
:::— If you have a Macintosh, there's a real simple way to make the dashes: the '''em-dash''' by pressing Control Option Hyphen at the same time, and the '''en-dash''' by pressing Option Hyphen at the same time. |
|||
:::—Did you notice how [[User:Nick Moyes|Nick Moyes]] creatively renamed Dasher, one of Santa Claus's eight reindeer, in his "Seasonal Greetings from all at the Teahouse" post to fellow editors below? |
|||
:::—You may be pleased to know that I found [https://www.google.com/search?client=safari&rls=en&q=long+hyphen&ie=UTF-8&oe=UTF-8 an online reference to a "long hyphen."] So, then, you weren't completely alone in doing that. But as [[Special:Contributions/94.1.223.204|94.1.223.204]] commented above, in professional editing we just don't use it. Like [[User:ColinFine|ColinFine]], )I think anyone who ''did'' say "long hyphen''"'' would probably be thinking of the em-dash; though I also think what [[Special:Contributions/94.1.223.204|94.1.223.204]] said above is also technically correct, that the term would have to refer to the en-dash (that's the next size up for a hyphen, after all). [[User:Augnablik|Augnablik]] ([[User talk:Augnablik|talk]]) 06:08, 27 December 2024 (UTC) |
|||
::::@[[User:Augnablik|Augnablik]], What a great reply! I thoroughly enjoyed every bit of it! And I learned a lot (not to detract from the other contributors, each of whom taught me something new - thanks, everybody) - a special thank you for the meanings of the em-dash and en-dash (I love that type of thing), and for drawing my attention to Nick Moyes' "Seasonal Greetings", and of course for finding me an ally in calling the en-dash a "Long hyphen" (don't worry, now that I know the correct terminology I will use it and hopefully amaze my friends...). Thank you again and Merry Christmas! [[User:Moonshane1933|Moonshane1933]] ([[User talk:Moonshane1933|talk]]) 12:58, 29 December 2024 (UTC) |
|||
::::@[[User:Augnablik|Augnablik]] Not to be a naysayer, but I think the bit about em dashes being named for being 'M' width is a false etymology. I too would have loved if it were true, but I think it's actually based on the [[em (typography)|em]] unit as described in [[Dash#Em dash]].  — [[User:Kilvin the Futz-y Enterovirus|Kilvin the Futz-y Enterovirus]] ([[User talk:Kilvin the Futz-y Enterovirus|talk]]) 10:23, 30 December 2024 (UTC) |
|||
:::::Well, @Kilvin the Futz-y Enterovirus, not to be a counter-naysayer (!) but there are many online sites with support for the width of the em-dash equal to M and of the en-dash equal to N. [https://www.grammarly.com/blog/punctuation-capitalization/dash/ Here’s just one, offered by Grammarly]. (Scroll down to ''What Is an En-Dash?'' and ''What Is an Em-Dash?'') [[User:Augnablik|Augnablik]] ([[User talk:Augnablik|talk]]) 12:06, 30 December 2024 (UTC) |
|||
::::::Hm... not to be a counter-{{hairspace}}''counter''-{{hairspace}}naysayer I mean I don't really think {{'}}''many online sites''{{'}} and Grammarly (unsourced, could just be mirroring untrustworthy sources) are [[WP:RS]]. But you're right; my searches show similar results and don't really yield great answers to this matter (ideally there'd be a page like "Many people say that an em dash is named for X, but actually it is Y, and this misconception came from Z" linking to many reliable sources). I guess my personal bias is towards the people going "false etymology!" that acknowledge both X and Y rather than one group who simply assert the "fun" explanation with seemingly no awareness of the other explanation. I mean, as linked previously, the [[em (typography)|em]] and [[en (typography)|en]] are typographical units whose pages describe their origins, even addressing [[Em (typography)#Obsolete alternative definition]].  — [[User:Kilvin the Futz-y Enterovirus|Kilvin the Futz-y Enterovirus]] ([[User talk:Kilvin the Futz-y Enterovirus|talk]]) 22:53, 30 December 2024 (UTC) |
|||
:::::::Oooh, @[[User:Kilvin the Futz-y Enterovirus|Kilvin the Futz-y Enterovirus]], you’re acknowledging “personal bias” towards “false etymology?” That requires a declaration of COI! 🙂 [[User:Augnablik|Augnablik]] ([[User talk:Augnablik|talk]]) 18:09, 31 December 2024 (UTC) |
|||
::::::::Aha! I found something! In the Historical thesaurus on the [https://www.oed.com/dictionary/em-dash_n?tab=meaning_and_use#1428570550100 OED] site. And I quote: |
|||
::::::::"'''[https://www.oed.com/dictionary/em-dash_n?tab=meaning_and_use#1428570550 em dash]1836-''' A long dash ''—'', originally and usually the width of one em (see ''em'', ''n.'')." [[User:Moonshane1933|Moonshane1933]] ([[User talk:Moonshane1933|talk]]) 13:13, 1 January 2025 (UTC) |
|||
== Add a page? == |
|||
:Resolved! thanks anway |
|||
[[User:Niccasey|Niccasey]] ([[User talk:Niccasey|talk]]) 13:26, 29 January 2017 (UTC) |
|||
Hello - How can somebody submit a page for a notable person? My husband has one of the country's worst wrongful convictions in the United States and I'd love to have somebody neutral put information up regarding his wrongful conviction case. We believe he will be exonerated someday. His name is Temujin Kensu and you can google search his name to learn more about this horrible case. Thank you! [[Special:Contributions/65.111.210.82|65.111.210.82]] ([[User talk:65.111.210.82|talk]]) 06:21, 28 December 2024 (UTC) |
|||
==page deleted== |
|||
:Based on my Google search, I consider it almost certain that Temujin Kensu is notable and that Wikipedia ought to have an article about him. [[User:Cullen328|Cullen328]] ([[User talk:Cullen328|talk]]) 07:39, 28 December 2024 (UTC) |
|||
hello my friend have created page name called Audrey D'Silva but page was deleted , i wanna recover that page |
|||
::For anyone interested in starting a draft [https://cse.google.com/cse?cx=007734830908295939403:galkqgoksq0#gsc.tab=0&gsc.q=Temujin%20Kensu some of these Google hits] could easily be used to pass [[WP:GNG]]. [[User:Michael D. Turnbull|Mike Turnbull]] ([[User talk:Michael D. Turnbull|talk]]) 12:21, 28 December 2024 (UTC) |
|||
is it possible ?[[User:Mehakdhavan|Mehakdhavan]] ([[User talk:Mehakdhavan|talk]]) 11:06, 29 January 2017 (UTC) |
|||
:::Realistically, you may not get a volunteer. Teahouse Hosts volunteer here to advise, not to be authors or co-authors. [[User:David notMD|David notMD]] ([[User talk:David notMD|talk]]) 20:18, 28 December 2024 (UTC) |
|||
:If your husband is [[Wikipedia:Notability|notable]] enough, he will be talked about. [[User:SimpleSubCubicGraph|SimpleSubCubicGraph]] ([[User talk:SimpleSubCubicGraph|talk]]) 05:37, 1 January 2025 (UTC) |
|||
:IP editor: I have now started to draft an article at [[Draft:Temujin Kensu]]. In view of your [[WP:PSCOI|conflict of interest]] it would be best if you did not edit it directly (although that's not forbidden while it remains a draft). I would welcome your suggestions for additions and corrections at [[Draft talk:Temujin Kensu]], especially where you can provide additional published sources I can use. [[User:Michael D. Turnbull|Mike Turnbull]] ([[User talk:Michael D. Turnbull|talk]]) 12:47, 1 January 2025 (UTC) |
|||
== Isotopes lists download == |
|||
: Welcome to the Teahouse. [[Audrey D'Silva]] has been deleted twice, the first at the author's request and the second after [[WP:Articles for deletion/Audrey D'Silva]]. The header of the latter page has a link to [[WP:Deletion review]]. --[[User:David Biddulph|David Biddulph]] ([[User talk:David Biddulph|talk]]) 11:31, 29 January 2017 (UTC) |
|||
Is there a to download these lists ? ( For example : the list in "<nowiki>https://en.wikipedia.org/wiki/Isotopes_of_sodium</nowiki>" ) |
|||
==How can I find my deleted article in the deletion log.== |
|||
Hi, |
|||
I have written a c# application that describes the relations between elements, isotopes, decays, fusions ... etc.(originating from the question "Where the carbon atoms in the cafeine in your coffee come from ?") |
|||
I wrote a page called 'Transcrypt' about a popular open source project I initiated. |
|||
It was deleted because it was considered promotional. |
|||
The reasons are said to be in the deletion log. |
|||
I tried to search for record of this deletion in the log, but couldn't find it. |
|||
I've filled in my user name and the name of the page, but there were zero hits. |
|||
When you make normal modifications to these lists, it takes me about 2 weeks to refresh my database for over 3000 isotopes and 5000 decays coming from 118 pages (and subject to typing errors...) |
|||
Kind regards |
|||
Jacques de Hooge |
|||
[[User:Jacdeh|Jacdeh]] ([[User talk:Jacdeh|talk]]) 08:57, 29 January 2017 (UTC) |
|||
I have tried to download one of these pages but I get one of these mumbo-jumbo network message ( about security and the correction looks like "set the web_client.Tchic_Tchac to Fling_Flang" ... and none of them works... ) |
|||
: Welcome to the Teahouse. It wasn't an article, it was a draft. The link is there from the notification on your user talk page. The link goes to [[Draft:Transcrypt]], which shows you the deletion log entry for that page. --[[User:David Biddulph|David Biddulph]] ([[User talk:David Biddulph|talk]]) 09:06, 29 January 2017 (UTC) |
|||
Do you have a suggestion ? |
|||
Thank you, David, for your reply. I have seen the entry you refer to. Only I wasn't aware that this was indeed a deletion log entry. The person deleting the page replied me to look in the deletion log to find the reasons why. However I only found the original remark about lacking references there. The point is that I've added those references in order to comply with quality and notability standards. I didn't get any reaction to that. Not that these references were good enough. Not that they were worthless. The page was just deleted, that's all. So I try to get into a conversation with someone experienced about how to improve this page. But I don't succeed in that, which I find frustrating. So at least I am glad to have obtained an helpful answer from a human being. I will for now not invest anymore time in this page. Not that I'm not willing to in principle, but there's no guarantee I'll get a decent reaction after even more effort. Still I consider Wikipedia very useful. Using it almost daily. |
|||
Thank you very much [[User:Michel Béliveau|Michel Béliveau]] ([[User talk:Michel Béliveau|talk]]) 17:49, 28 December 2024 (UTC) |
|||
==##1== |
|||
:Describing a message as a "mumbo-jumbo network message" is not very helpful in determining what your problem is. If you quote the error message exactly it might be more useful. In any case, I can successfully download articles using curl like this: <syntaxhighlight>curl -k https://en.wikipedia.org/wiki/Isotopes_of_sodium</syntaxhighlight> [[User:CodeTalker|CodeTalker]] ([[User talk:CodeTalker|talk]]) 19:52, 28 December 2024 (UTC) |
|||
how to delete our notifications or alerts? |
|||
::Thank you for this fast (and good) answer. |
|||
How to clear some unwanted msgs on our user talk page ? |
|||
::The mumbo-jumbo error message was : "The request was aborted: Could not create SSL/TLS secure channel." I was not using the good approach to download the content of the web page. |
|||
kindly let me know about this by discussing |
|||
::>>> However the CURL function does what I need. |
|||
thank you. |
|||
::Here is want to do In my application : |
|||
[[User:Jordanben|jordan]] ([[User talk:Jordanben|talk]]) 07:56, 29 January 2017 (UTC) |
|||
::For each Element ( 118 !!! ) get the "List of Isotopes" for this Element. Then for each Isotope : get its mass, half-life, decay mode(s) and decay product(s). This yields for over 3500 isotopes and over 4500 decays. Refreshing the data took quite a long time. |
|||
==How to delete a file. Should be simple and straightforward but not working out that way.== |
|||
::Analyzing the results of the curl command is not so hard and will eliminate typing mistakes. Even if I need a few days to program the analysis, it will be faster than re-typing the data. |
|||
I am trying to delete a duplicate file. The HELP sections says that files are deleted in the same manner as pages and that the process is very simple. Go tot he file (or page) and there is a delete button at the top of the page and you click that and a dialog box will ask for the underlying reason. |
|||
::I will take a look at Wikidata. |
|||
::Thanks again. [[User:Michel Béliveau|Michel Béliveau]] ([[User talk:Michel Béliveau|talk]]) 23:02, 28 December 2024 (UTC) |
|||
:Without really understanding what you are trying to do, I would suspect that [[Wikidata]] was a more useful resource than Wikipedia for your purpose, as it is a database which contains relations between its elements. [[User:ColinFine|ColinFine]] ([[User talk:ColinFine|talk]]) 21:34, 28 December 2024 (UTC) |
|||
::@[[User:Michel Béliveau|Michel Béliveau]] Wouldn't it be easier to download from the original sources, for example [https://www-nds.iaea.org/relnsd/nubase/nubase_min.html NUBASE]? [[User:Michael D. Turnbull|Mike Turnbull]] ([[User talk:Michael D. Turnbull|talk]]) 15:48, 29 December 2024 (UTC) |
|||
:::Thank you Mr. Turnbull. |
|||
:::You are correct. It would be easier to download from the original sources. |
|||
:::I have found (and used) a NUBASE file (namely for nucleus values) . So far, I have found only 1 NUBASE file that I could use ( coming from "The Ame2020 atomic mass evaluation (I)" by W.J.Huang, M.Wang, F.G.Kondev, G.Audi and S.Naimi - Chinese Physics C45, 030002, March 2021) . |
|||
:::The purpose of my request to Wikipedia is to avoid re-typing the values. The NUBASE file allowed this. |
|||
:::Do you know other NUBASE files ? Or other sources ? (I also found some data in PeriodicTable.com) |
|||
:::Thanks again for your interest [[User:Michel Béliveau|Michel Béliveau]] ([[User talk:Michel Béliveau|talk]]) 19:19, 29 December 2024 (UTC) |
|||
{{to|Michel Béliveau}} Hello! This kind of question is something you might want to take to [[WP:RD/C|the Computing reference desk]], where you can get people with computing expertise to assist. --[[User:Slowking Man|Slowking Man]] ([[User talk:Slowking Man|talk]]) 20:39, 31 December 2024 (UTC) |
|||
== Locked out of account == |
|||
Of course when I open the file there is no delete button anywhere in sight and so I am unable to delete the duplicate file. |
|||
I got locked out of my DooplissTTYD account because I forgot the complex password and didn’t have an email address linked to it. Is there any way that account can be renamed to something else and I change this one to DooplissTTYD? [[User:TTYDDoopliss|TTYDDoopliss]] ([[User talk:TTYDDoopliss|talk]]) 21:50, 28 December 2024 (UTC) |
|||
Please help! [[Special:Contributions/75.155.187.94|75.155.187.94]] ([[User talk:75.155.187.94|talk]]) 06:28, 29 January 2017 (UTC) |
|||
:You would have to request a renaming from the account you want renamed. It can't be requested by in essence a third party(as we have no way to know who is on the other end of the computer). The best you can do is post on your current and previous user pages that you lost access to your old account and have a new one. [[User:331dot|331dot]] ([[User talk:331dot|talk]]) 00:06, 29 December 2024 (UTC) |
|||
:{{ping|75.155.187.94}} Which file are you trying to delete? --[[User:Gronk Oz|Gronk Oz]] ([[User talk:Gronk Oz|talk]]) 06:37, 29 January 2017 (UTC) |
|||
:This is why the first thing you want to ensure you do right now is verify an e-mail address for your account, so you can restore it: [[Special:Preferences|follow this link if you haven't]]. Step 2: I highly recommend using a [[password manager]] to save passwords—as well as generate secure passwords. I suggest trying out [[Bitwarden]]. |
|||
::Hello, IP editor. Please take a look at [[Wikipedia:Files for discussion]], where you can nominate any file you wish for deletion. Though I am not familiar with the "delete button" you mention, please be aware that not all functions are available to unregistered editors. Please consider opening an optional Wikipedia account, which offers many benefits and no negatives. [[User:Cullen328|<b style="color:#070">Cullen</b><sup style="color:#707">328</sup>]] [[User talk:Cullen328|<span style="color:#00F">''Let's discuss it''</span>]] 08:04, 29 January 2017 (UTC) |
|||
:{{br}} |
|||
:Now in this particular case, [[Special:Contributions/DooplissTTYD|your prior account has only a few edits]]; this means if you wait a bit, say 6 months, you can [[WP:USURPNAME|request to "usurp" it]] which will accomplish what you want. ([[Paper Mario TTYD|Also, a very good game.]]) --[[User:Slowking Man|Slowking Man]] ([[User talk:Slowking Man|talk]]) 18:06, 31 December 2024 (UTC) |
|||
::I mean that’s what I did. I generated a password using a feature on my phone, and it kept saying the one I had on my phone was incorrect. |
|||
::and yes I have an email linked to this account now. 6 months is a very long time though… [[User:TTYDDoopliss|TTYDDoopliss]] ([[User talk:TTYDDoopliss|talk]]) 19:13, 31 December 2024 (UTC) |
|||
== donating - would like to donate == |
|||
:Hi person editing from 75.155.187.94. It sounds as if maybe you read an instruction page directed at administrators—who do have a deletion button; non-administrators do not. In addition to the instructions above, it's possible that if it is truly a duplicate separate file, or a duplicate ''version'' of a single file, then tagging it for speedy deletion might be possible. It must strictly meet the criterion though, which are set out at [[Wikipedia:Criteria for speedy deletion#Files]]. The deletion tags that you would place on the file page, if a speedy deletion criterion applies, is given for each criterion at the page I linked. A second possibility occurs to me. If the file is at the [[Commons:|Wikimedia Commons]], rather than on Wikipedia (people often don't realize images they see are not actually hosted here but there), than there is a "nominate for Deletion" link under the tools menu on the left hand side of the page. None of the deletion methods anyone has discussed here will apply to the Commons, though they have equivalent procedures. See [[Commons:Commons:Deletion policy|Commons:Deletion policy]]. By the way, if you had told us the name of the actual file, we all could have crafted far more targeted answers for you. Best regards--[[User:Fuhghettaboutit|Fuhghettaboutit]] ([[User talk:Fuhghettaboutit|talk]]) 14:11, 29 January 2017 (UTC) |
|||
Hail... |
|||
==How to locate and edit a new Catgegory for a recently uploaded picture.== |
|||
Would like to donate 50 quid to the cause but stop at the name and address |
|||
I had to create a new "category" as requested by the wizard while uploading my first photo for my first draft article. Of course I made a typing error in describing the category and now I need to edit it. I have looked all over the HELP feature and I can find no way to locate or edit my new category! Please provide some clear instructions on how to first locate and then edit my category. All this is related to my article: Jean Jepson, Dancer, Teacher, Choreographer. |
|||
part. Don't really see the need for full name and address. |
|||
Just old and not particularly wise. Any suggestions? [[Special:Contributions/81.96.25.61|81.96.25.61]] ([[User talk:81.96.25.61|talk]]) 12:28, 29 December 2024 (UTC) |
|||
:Hi IP editor, the Wikimedia Foundation deals with all donation issues and questions - editors here at the Teahouse don't have any input. Please direct your query to the email address at the bottom of [[donate:Ways_to_Give|donate.wikimedia.org/wiki/Ways_to_Give]] <span style="background-color: RoyalBlue; border-radius: 1em; padding: 3px 3px 3px 3px;">'''[[User:Qcne|<span style="color: GhostWhite">qcne</span>]]''' <small>[[User talk:Qcne|<span style="color: GhostWhite">(talk)</span>]]</small></span> 12:49, 29 December 2024 (UTC) |
|||
Teahouse has been very helpful in providing answers to my previous questions. It is not clear to me though if sending email replies would be helpful if I have a follow-up question to your initial reply. Please comment on this.[[User:CableHut|CableHut]] ([[User talk:CableHut|talk]]) 04:19, 29 January 2017 (UTC) |
|||
::Just to elaborate slightly: all editors here give their time entirely voluntarily and gain absolutely no financial benefit from any contributions made to keep the broader Wikipedia projects going. So we have little knowledge of how the donation systems work - despite being grateful for everyone's contributions. The advice above is sound. [[User:Nick Moyes|Nick Moyes]] ([[User talk:Nick Moyes|talk]]) 18:21, 29 December 2024 (UTC) |
|||
:Welcome to the Teahouse, {{u|CableHut}}. The Teahouse is for answering questions about editing Wikipedia, and I think you are talking about an image uploaded to our separate sister project [[Wikimedia Commons]] instead. No matter. I went over there and clicked the "edit" button on the image you uploaded, and changed "Category:Tap" to "Category: Tap dancer". That is a very easy thing to do. The categories are near the bottom of the image file. |
|||
:Also I'm not certain (not WMF personnel), but I am pretty confident the reason for requiring that info is fraud protection and banking and [[money laundering]] laws; it's not something the Foundation has any control over. WMF is a registered [[501(c)(3) organization]] in the US, meaning it has to comply with a bunch of laws and regulations. (For ''furrin types'', this is near-synonymous with "[[non-profit]]/[[Charity organization|charity]]" in the US, being simply the section of the [[Internal Revenue Code]] setting out the criteria said organizations have to meet to get tax exemption.) Note, a way that one can make semi-anonymous donations, is to pass them through what's called a "[[donor-advised fund]]" in the US, but this is a bit more involved and requires going through a bank or brokerage that will arrange your transfer. Alternatively, I suppose if one is accepting of the fact there are no guarantees in life one could always send cash to the WMF's front door with no return address and leave its final disposition up to them. --[[User:Slowking Man|Slowking Man]] ([[User talk:Slowking Man|talk]]) 18:38, 31 December 2024 (UTC) |
|||
== I don't find the ressource to add books wrote by someone == |
|||
:As for email replies, that is very rarely necessary. Just return to the thread you started, click "edit", and add a follow-up comment to the bottom of the thread. [[User:Cullen328|<b style="color:#070">Cullen</b><sup style="color:#707">328</sup>]] [[User talk:Cullen328|<span style="color:#00F">''Let's discuss it''</span>]] 08:23, 29 January 2017 (UTC) |
|||
Hello ! I'd like to add a book on the article about "[[David Murphy (CIA)]]". <br /> <br /> |
|||
==how long== |
|||
My [[Draft:List of highest-grossing animated films in Canada and the United States|article]] has be waiting for 3 weeks+ can someone get round to it soon please. [[Special:Contributions/82.38.157.176|82.38.157.176]] ([[User talk:82.38.157.176|talk]]) 00:31, 29 January 2017 (UTC) |
|||
:Hi, 86. You are currently working on two draft articles. Neither are currently submitted for review. So in short no one is going to get around to either anytime. And I cannot help further because I have no way of knowing which one you want to submit for review. [[User:John from Idegon|John from Idegon]] ([[User talk:John from Idegon|talk]]) 01:36, 29 January 2017 (UTC) |
|||
::Oh I didn't realise the one I want to be submit is [[Draft:List of highest-grossing animated films in Canada and the United States]] I thought I did send that off i do it now the other one my next project I will work on that after, i will try to send it off now if I have anymore problems I will come back.[[Special:Contributions/82.38.157.176|82.38.157.176]] ([[User talk:82.38.157.176|talk]]) 09:22, 29 January 2017 (UTC) |
|||
:::Back it telling me both articles have been submit[[Special:Contributions/82.38.157.176|82.38.157.176]] ([[User talk:82.38.157.176|talk]]) 09:24, 29 January 2017 (UTC) |
|||
::: I don't know why [[User:John from Idegon|John from Idegon]] thought that [[Draft:List of highest-grossing animated films in Canada and the United States]] (which was indeed the draft you linked to in your original question) had not been re-submitted for review. It was submitted on 1 January. There are 435 drafts awaiting review; about a quarter of them have been waiting more than 3 weeks. --[[User:David Biddulph|David Biddulph]] ([[User talk:David Biddulph|talk]]) 11:27, 29 January 2017 (UTC) |
|||
This is a book wrote by him not mentionned in the article. <br /> |
|||
==Is it appropriate for one editor to put a command in a comment when no such consensus exists or indeed has ever been discussed on the talk page? == |
|||
I don't find the ressource explaining how to add the bibliography of someone. [[User:Anatole-berthe|Anatole-berthe]] ([[User talk:Anatole-berthe|talk]]) 16:17, 29 December 2024 (UTC) |
|||
For example, [https://en.wikipedia.org/enwiki/w/index.php?title=National_Repertory_Orchestra&diff=761255881&oldid=761150119 here] I had added Trumpeter Michael Sachs to the list of notable alumni for the [[National Repertory Orchestra]]. Michael is the long time principal trumpeter of the Cleveland Orchestra, a world famous trumpeter and educator, and I provided a reference that demonstrated his notability and connection to the NRO. Immediately following that, another editor [https://en.wikipedia.org/enwiki/w/index.php?title=National_Repertory_Orchestra&diff=next&oldid=761255881 here] removed my entry, and put in a comment stating "Do NOT add names with now WP article". There was no consensus at this time, the other editor simply issued a command. Now according to [[WP:CSC]], "Red-linked entries are acceptable if the entry is verifiably a member of the listed group, and it is reasonable to expect an article could be forthcoming in the future" so (1) wasn't my entry perfectly reasonable, and (2) weren't the actions of the other editor outside of WP policy and borderline rude? --[[User:Cohler|<b>''TheClarinetGuy''</b> <sub>''talk''</sub>]] 18:56, 28 January 2017 (UTC) |
|||
:{{u|Cohler}}, there exists a project wide consensus that in order to be listed in a notable person's list, that person's notability needs to be shown by the existence of their biography here on Wikipedia or sourcing that shows that beyond a reasonable doubt. See [[WP:NLIST]]. If the person you want to add meets article qualification guidelines (either the [[WP:GNG|general]] guideline or the [[WP:NMUSIC|musician - specific]] guideline), then by all means write their biography then add them to the list. [[User:John from Idegon|John from Idegon]] ([[User talk:John from Idegon|talk]]) 19:08, 28 January 2017 (UTC) |
|||
::{{u|John from Idegon}} that is very different from what {{u|Marchjuly}} told me [https://en.wikipedia.org/enwiki/w/index.php?title=Wikipedia:Teahouse&diff=762361533&oldid=762360928 here] so now I am confused. And certainly it does not say that on [[WP:CSC]] which I quoted above. Why is there so much disagreement on this issue? Perhaps the policies surrounding this need to be clarified? Also, [[WP:Notability]] says that notability is established ''not'' by existence of a WP article but by the existence of acceptable sources. --[[User:Cohler|<b>''TheClarinetGuy''</b> <sub>''talk''</sub>]] 19:15, 28 January 2017 (UTC) |
|||
:::{{u|Cohler}}, you did not provide an independent source showing that this musician is "world famous" but rather a link to an orchestra website that lists all members of that orchestra. Because of your topic ban, you would be well-advised to avoid any type of editing that can be perceived as disruptive or misleading. Please desist. [[User:Cullen328|<b style="color:#070">Cullen</b><sup style="color:#707">328</sup>]] [[User talk:Cullen328|<span style="color:#00F">''Let's discuss it''</span>]] 19:22, 28 January 2017 (UTC) |
|||
::::Taking a closer look, {{u|Cohler}}, I see that you were editing a list that includes your own name, and you are topic banned from writing anything autobiographical. This is a ''really bad idea''. [[User:Cullen328|<b style="color:#070">Cullen</b><sup style="color:#707">328</sup>]] [[User talk:Cullen328|<span style="color:#00F">''Let's discuss it''</span>]] 19:27, 28 January 2017 (UTC) |
|||
:::::This has nothing to do with my topic ban. I am simply trying to get an answer to my question. Why do you only respond with commands and veiled threats? Forget about the particulars, I am asking a general question, why are people giving totally different answers? Some are saying notability does not require a WP article, others are saying that it does, and [[WP:Notability]] clearly does ''not'' require a WP article. Here for example, {{u|John from Idegon}} wrote "there exists a project wide consensus that in order to be listed in a notable person's list, that person's notability needs to be shown by the existence of their biography here on Wikipedia" which is a direct contradiction of what {{u|Marchjuly}} wrote when he said "Individual entries of such a list do not have to have a stand-alone article written about them". I'm just looking for an answer to the policy question. Please answer the question and stop the veiled threats. I thought this was the place for asking questions in a friendly environment? --[[User:Cohler|<b>''TheClarinetGuy''</b> <sub>''talk''</sub>]] 19:34, 28 January 2017 (UTC) |
|||
::::::[[User:Doc James]] clarified the terms of your topic ban to include "your work or the people you work with" here [[Wikipedia:Teahouse#Notability_requirement_for_a_list_that_is_inside_another_article]]. That would seem to include the [[National Repertory Orchestra]] article. Once again you seem to be tendentiously editing very close to your ban. [[User:Meters|Meters]] ([[User talk:Meters|talk]]) 19:46, 28 January 2017 (UTC) |
|||
:@[[User:Anatole-berthe|Anatole-berthe]] The standard template to use for book citations is {{t|cite book}}. It is usual, but not essential, to use its |URL= parameter to link to Google Books for the convenience of our readers. In this case that would be [https://www.google.co.uk/books/edition/Battleground_Berlin/3USTBk2dulQC?hl=en&gbpv=1&dq=david%20murphy%20battleground&pg=PR4&printsec=frontcover this link], from which you can also find the ISBN and full list of authors. [[User:Michael D. Turnbull|Mike Turnbull]] ([[User talk:Michael D. Turnbull|talk]]) 16:27, 29 December 2024 (UTC) |
|||
{{Od}}Read it again. I said they had to have a bio or you had to prove it (probably could have said that better...you have to prove they would qualify to have a bio). You are violating your topic ban. So, to put things in simple terms, you cannot edit that list. You cannot edit any article that contains your name, you cannot edit any article about any organization you have ever had any association with. That's what a topic ban is. You have been given a topic ban in order for you to show you have the capacity to internalized our requirements. You show that by editing in areas you have no connection with (e.g. [[Bread]]) successfully. The alternative is you will simply be blocked. Sorry....that's as friendly and supportive as it can be phrased. [[User:John from Idegon|John from Idegon]] ([[User talk:John from Idegon|talk]]) 19:48, 28 January 2017 (UTC) |
|||
:Hello, @[[User:Anatole-berthe|Anatole-berthe]]. Why do you want to add that book to that article? Has the book been discussed by independent sources? If not, why is it significant ednough to feature in a Wikipedia article? |
|||
:First, I am asking a theoretical question that has nothing to do with any specifics. I simply gave an example. Second, I have never worked at the National Repertory Orchestra, it is a training orchestra for young musicians and I did receive a fellowship to be in the orchestra for one summer in 1979. That hardly qualifies as some place that I work. What are you talking about? Could you please answer my question and stop with the attacks. I am not and have not violated my topic ban. Boy you admins seem to love attacking people who ask simple questions. Very frustrating indeed. I guess the Teahouse is not so friendly after all. --[[User:Cohler|<b>''TheClarinetGuy''</b> <sub>''talk''</sub>]] 20:02, 28 January 2017 (UTC) |
|||
:More to the point, that article is woefully short of sources, and does not establish that Murphy meets the criteria for [[WP:NBIO|notability]]. [[User:ColinFine|ColinFine]] ([[User talk:ColinFine|talk]]) 17:42, 29 December 2024 (UTC) |
|||
::The book was wrote by himself as I said. <br /> <br /> |
|||
::The ISBN-10 is '''"0300107803"''' and the ISBN-13 is '''"978-0300107807"''' for the first edition. |
|||
::The title is '''"What Stalin Knew: The Enigma of Barbarossa"''' , it was published by "[[Yale University Press]]" in 2005. <br /> <br /> |
|||
::The ISBN-10 is '''"030011981X"''' and the ISBN-13 is '''"978-0300119817"''' for the second edition. |
|||
::The second edition was published in 2006 by the same publisher. <br /> <br /> |
|||
::In this edition , it's mentionned the review of the first edition by "[[Niall Ferguson]]" for '''"New York Time Book Review"'''. |
|||
::The first edition was also reviewed by "[[Andrew Nagorski]]" for "[[The Weekly Standard]]". <br /> <br /> |
|||
::There are also a review by "[[Henry Kissinger]]" but I consider it doesn't count to add this book on Wikipedia. |
|||
::This person have to be considered as a reviewer among others even if he's notable accorded to criterias. <br /> <br /> |
|||
::He didn't wrote the review for a magazine or a journal and therefore the fact '''"Kissinger"''' wrote a review should not be taken into account. This is what I think. <br /> <br /> |
|||
::The same for the review by "[[Donald Kagan]]" for "[[Yale University]]" for a particular reason. This person worked for '''"Yale"''' and the book was published by "[[Yale University Press]]". Therefore , I consider it's like a review by the publisher itself. <br /> <br /> <br /> |
|||
::Conclusion : I think this book met the criteria n°1 for "[[Wikipedia:Notability_(books)]]" to create an article on this book. |
|||
::I don't want to create an article on the book itself. <br /> <br /> |
|||
::The '''"Threshold standards"''' is met because "[[Library of Congress]]" catalogued this book. |
|||
::[[LCCN]] is '''"2004065916"'''. <br /> <br /> |
|||
::If it is considered as notable to create an article , I consider it is notable enough to mention this book on the article "[[David Murphy (CIA)]]". [[User:Anatole-berthe|Anatole-berthe]] ([[User talk:Anatole-berthe|talk]]) 12:03, 1 January 2025 (UTC) |
|||
== can you give me a lnk of the Roblox page == |
|||
::::::(EC) What is required is that notability of the person be established. The easiest way to demonstrate this is by creating a well referenced article. Short of that, you would need to list multiple references (i.e. ''show'' the notability, show that the person ''could'' have an article) with the name in the list. While this is not forbidden; the former method is '''much''' preferred. A single link to a listing of all members of an orchestra does not prove that person meets the notability requirements. Does this help clarify matters for you? <span style="font-family: Lucida Calligraphy">[[User:LadyofShalott|<span style="color: #442288">Lady</span>]]<span style="color: #22aaaa">of</span>[[User_Talk:LadyofShalott|<span style="color: #cc2288">Shalott</span>]]</span> 19:49, 28 January 2017 (UTC) |
|||
:::::::{{u|LadyofShalott}}, {{u|Cohler}} can't write articles about people he knows and works with because he has a [[WP:COI|conflict of interest]]. [[User:Justlettersandnumbers|Justlettersandnumbers]] ([[User talk:Justlettersandnumbers|talk]]) 19:56, 28 January 2017 (UTC) |
|||
::::::::{{u|Justlettersandnumbers}} I don't work with any of the people in the list. As I said, I was a student member of the orchestra in 1979 nearly 40 years ago. Why do you jump to ridiculous conclusions and then publish them here as fact with no basis whatsoever? Please desist. Furthermore, I am just asking a question here. Answers as opposed to your totally unfounded accusations, would be appreciated. --[[User:Cohler|<b>''TheClarinetGuy''</b> <sub>''talk''</sub>]] 20:08, 28 January 2017 (UTC) |
|||
:::::::{{u|LadyofShalott}} Thank you for the first actual response to one of my questions here. --[[User:Cohler|<b>''TheClarinetGuy''</b> <sub>''talk''</sub>]] 20:08, 28 January 2017 (UTC) |
|||
i need Roblox link to sign in [[Special:Contributions/24.192.134.19|24.192.134.19]] ([[User talk:24.192.134.19|talk]]) 01:57, 30 December 2024 (UTC) |
|||
So getting back to my question, "Is it appropriate for one editor to put a command in a comment when no such consensus exists or indeed has ever been discussed on the talk page?" And this is a general question, not in relation to any particular circumstance. --[[User:Cohler|<b>''TheClarinetGuy''</b> <sub>''talk''</sub>]] 20:10, 28 January 2017 (UTC) |
|||
:{{welcometea}} Did you have a question about editing or using Wikipedia? —[[User:Tenryuu|<span style="color:#556B2F">Tenryuu 🐲</span>]] ( [[User talk:Tenryuu|💬]] • [[Special:Contributions/Tenryuu|📝]] ) 02:07, 30 December 2024 (UTC) |
|||
:Yes. If you want a more detailed explanation see above. You'll generally get a clearer response if you don't phrase your questions in a "Have you stopped beating your wife?" format. [[User:John from Idegon|John from Idegon]] ([[User talk:John from Idegon|talk]]) 20:43, 28 January 2017 (UTC) |
|||
::@[[User:Tenryuu|Tenryuu]] I think hes trolling. [[User:SimpleSubCubicGraph|SimpleSubCubicGraph]] ([[User talk:SimpleSubCubicGraph|talk]]) 02:50, 30 December 2024 (UTC) |
|||
::{{u|Cohler}}, yes, you have violated your topic ban when you wrote in an earlier Teahouse thread: "Why? The topic ban is on Jonathan Cohler. I'm allowed to edit other things right? Boston Conservatory is not related to "Jonathan Cohler" other than I am a notable faculty member there." You are not allowed to edit, mention or even allude to or hint at [[Jonathan Cohler]], except when appealing the topic ban. Please stop. [[User:Cullen328|<b style="color:#070">Cullen</b><sup style="color:#707">328</sup>]] [[User talk:Cullen328|<span style="color:#00F">''Let's discuss it''</span>]] 20:49, 28 January 2017 (UTC) |
|||
:::That's my standard reply to anyone who doesn't ask questions about Wikipedia here. —[[User:Tenryuu|<span style="color:#556B2F">Tenryuu 🐲</span>]] ( [[User talk:Tenryuu|💬]] • [[Special:Contributions/Tenryuu|📝]] ) 02:58, 30 December 2024 (UTC) |
|||
:[[User:Cohler]] - If you have a specific question that isn't related to your topic-ban and isn't confrontational, you may ask it. However, you don't appear to be asking a question so much as trying to start a quarrel. Please be aware that some of the editors here are administrators who can block you for disruptive editing, including for editing in violation of a topic-ban. [[User:Robert McClenon|Robert McClenon]] ([[User talk:Robert McClenon|talk]]) 21:00, 28 January 2017 (UTC) |
|||
:Does [[Roblox#External links|this link]] help at all? If you have general computing questions that aren't Wikipedia-specific, go to the [[WP:RD/Computing|Computing reference desk]]. --[[User:Slowking Man|Slowking Man]] ([[User talk:Slowking Man|talk]]) 18:46, 31 December 2024 (UTC) |
|||
::I was deliberately not getting into the topic ban question because I have not looked at either the ban or the article in question. {{U|Cohler}}, multiple editors who ''have'' looked at those think you are in violation. I would advise you to step away from this topic. <span style="font-family: Lucida Calligraphy">[[User:LadyofShalott|<span style="color: #442288">Lady</span>]]<span style="color: #22aaaa">of</span>[[User_Talk:LadyofShalott|<span style="color: #cc2288">Shalott</span>]]</span> 21:34, 28 January 2017 (UTC) |
|||
== Good Sources for Articles == |
|||
{{reply to|Cohler}} If my response to your previous Teahouse question was in error or created confusion than my apologies. However, you're quoting of me probably makes more sense if you quote the entire sentence and not just the first part. What I wrote (bold added by me) was {{tq|Individual entries of such a list do not have to have a stand-alone article written about them,<b>but that is typically the basic criterion for inclusion</b>}}. You left out that last part (which I think is kind of important) by mistake when you posted [[:Special:diff/Cohler/762429459|this]] above. The part you quoted seems to imply that I am in disagreement with {{u|John from Idegon}}, which is not the case at all. At the end of my post I wrote {{tq|There is no automatic inclusion for such names, and like other article content you may have to reach a consensus for it on the article's talk page}}, which means that the citeria for conclusion of a particular article is something that is often decided on the talk page of that article. Some articles such as [[:University of Oxford]] could have an "Notable alumni" list 1000s of entries long if inclusion was pretty much automatic, but the consensus doesn't seem to be to allow that. That is why another article [[:List of University of Oxford people]] where those not suitable for mentioning in the main article was created, but even that article would be huge and unwieldy if everyone was included, so its further broken up in to other stand-alone articles such as [[:List of University of Oxford people in the law]]. This why I also wrote in my that it's OK to be bold and add a name to the list, but you should follow [[:WP:BRD]] if you're reverted. |
|||
G-Day |
|||
Fnally, I wrote at the very beginning of my post {{tq|there was probably something about your editing at "Jonathan Cohler" which led to the community deciding to topic ban you from editing that article. So, if you try and use the same approach on another article, then your editing their will probably come under scrutiny as well}}. In general, editors who try to turn Wikipedia into a [[:WP:BATTLEGROUND]] have very little success achieving their goal(s) even in cases where they may have a point. If other editors posting above are telling you that adding the names to a list i a violation of your topic band, then you need to either (1) request clarification of that from the admin who closed the relevant discusion which lead to the topic ban or (2) accept that and [[:WP:DTS|move on to editing something else]]. The Teahouse is supposed to be a friendly place to try and help others with editing questions. It's not really set up to work like [[:WP:VP/P]], [[:WP:AN]], or other [[:WP:PNB|community noticeboards]] which deal with more specific issues. Perhaps one of those would be a better place for you to address your concerns. -- [[User:Marchjuly|Marchjuly]] ([[User talk:Marchjuly|talk]]) 00:05, 29 January 2017 (UTC) |
|||
I wanted to ask if there is any reliable source for Articles. I cannot afford my personal favourite, The "Britannica" Encyclopedia, since they seem to be rare and expensive. Thank you. (I know this isn't about a specific Article but I need to know this) [[User:PizzaFrank|PizzaFrank]] ([[User talk:PizzaFrank|talk]]) 13:51, 30 December 2024 (UTC) |
|||
==Should quotes or parenthesis be included in articles when you're not directly quoting someone?== |
|||
:@[[User:PizzaFrank|PizzaFrank]] there are all sorts of reliable sources, not just Encyclopædia Britannica, and most don't require a purchase of any kind. See [[Wikipedia:Reliable sources]] for a detailed overview of RSes here on Wikipedia. <span style="white-space:nowrap"><span style="font-family:monospace">'''<nowiki>'''[[</nowiki>[[User:CanonNi]]<nowiki>]]'''</nowiki>'''</span> ([[User talk:CanonNi|talk]] • [[Special:Contributions/CanonNi|contribs]])</span> 13:55, 30 December 2024 (UTC) |
|||
Example: The "autobahn" (German Train) is a very popular German method of transport. |
|||
::Thank you, I will check that out. [[User:PizzaFrank|PizzaFrank]] ([[User talk:PizzaFrank|talk]]) 14:03, 30 December 2024 (UTC) |
|||
:{{u|PizzaFrank}} Note that you don't have to personally own the source- it just needs to be publicly accessible, like online, or in a library. [[User:331dot|331dot]] ([[User talk:331dot|talk]]) 14:01, 30 December 2024 (UTC) |
|||
::Understood Thank you for the help. [[User:PizzaFrank|PizzaFrank]] ([[User talk:PizzaFrank|talk]]) 14:04, 30 December 2024 (UTC) |
|||
:::[[User:PizzaFrank|PizzaFrank]] I have used https://www.britannica.com a few times, and there is no fee to use the online version of Encyclopedia Britannica. [[User:Karenthewriter|Karenthewriter]] ([[User talk:Karenthewriter|talk]]) 18:27, 30 December 2024 (UTC) |
|||
:And besides [[public libraries]] which are great, Wikipedia editors with some experience can access [[WP:The Wikipedia Library|The Wikipedia Library]] which grants ''free'' access to tons of "paywalled" "digital content"! Pretty neat! Also see there: even if your account is not "seasoned" enough yet, you can request experienced editors assist you with researching sources through it. --[[User:Slowking Man|Slowking Man]] ([[User talk:Slowking Man|talk]]) 17:45, 1 January 2025 (UTC) |
|||
== Photograph attribution == |
|||
Should it be that, or this: |
|||
I want to use a portrait of my grandfather taken in 1915 by a company that no longer exists. I can show an attribution in the caption, but there is nobody I can seek permission from. How do I proceed, please? [[User:Gangnam Woodford|Gangnam Woodford]] ([[User talk:Gangnam Woodford|talk]]) 16:52, 30 December 2024 (UTC) |
|||
The autobahn is a very popular German method of transport. |
|||
:Hello. It depends on the laws of your country, but a photo taken in 1915 is likely in the public domain. According to https://commons.wikimedia.org/wiki/Commons:Copyright_rules_by_territory/United_States in the US anything published before 1929 is public domain(other than sound recordings), so it would just depend on your own country. [[User:331dot|331dot]] ([[User talk:331dot|talk]]) 16:56, 30 December 2024 (UTC) |
|||
Which should be used?[[User:ZachAttrax|ZaxAttrach]] ([[User talk:ZachAttrax|talk]]) 18:23, 28 January 2017 (UTC) |
|||
::Yes '''BUT''' (big but!), "publication" in the context of US copyright law has a specifical technical, legal definition. It doesn't mean "creation" or even "giving a copy to a particular person". [[commons:COM:Hirtle chart|Hirtle chart]]. A photograph of a private individual is almost certainly an "unpublished work" under the US copyright meaning of that term, which matters significantly for the length of its copyright term especially if an "older" work. Copyright is a subject that gets quite technical, so if not familiar and experienced with copyright issues, it is best to hesitate to advise others before researching things in detail and double-checking, and if in doubt, direct others to forums like [[commons:COM:VP/C|Commons]] where experts can be found. --[[User:Slowking Man|Slowking Man]] ([[User talk:Slowking Man|talk]]) 20:02, 31 December 2024 (UTC) |
|||
:Hello {{u|ZachAttrax}} and welcome to the Teahouse. Definitely the second one, as per [[MOS:QUOTEMARKS]]. Much more style advice is available at [[WP:MOS]]. [[User:Psiĥedelisto|Psiĥedelisto]] ([[User talk:Psiĥedelisto|talk]]) 18:29, 28 January 2017 (UTC) |
|||
:@[[User:Gangnam Woodford|Gangnam Woodford]] Is this an American picture? If so, you can upload it as public domain, see [https://commons.wikimedia.org/wiki/Commons:Hirtle_chart]. [[User:Gråbergs Gråa Sång|Gråbergs Gråa Sång]] ([[User talk:Gråbergs Gråa Sång|talk]]) 16:57, 30 December 2024 (UTC) |
|||
::Sorry, should have said: UK. [[User:Gangnam Woodford|Gangnam Woodford]] ([[User talk:Gangnam Woodford|talk]]) 16:58, 30 December 2024 (UTC) |
|||
:::@[[User:Gangnam Woodford|Gangnam Woodford]] Still good, see this example: [https://commons.wikimedia.org/wiki/File:Archibald_Joyce.jpg File:Archibald Joyce.jpg]. [[User:Gråbergs Gråa Sång|Gråbergs Gråa Sång]] ([[User talk:Gråbergs Gråa Sång|talk]]) 17:02, 30 December 2024 (UTC) |
|||
::::Thank you! [[User:Gangnam Woodford|Gangnam Woodford]] ([[User talk:Gangnam Woodford|talk]]) 17:06, 30 December 2024 (UTC) |
|||
:::::Please also see [[MOS:CREDITS]] ".... do not credit the image author or copyright holder in the article. .... as long as the appropriate credit is on the image description page." - [[User:Arjayay|Arjayay]] ([[User talk:Arjayay|talk]]) 17:16, 30 December 2024 (UTC) |
|||
::::::Thanks. [[User:Gangnam Woodford|Gangnam Woodford]] ([[User talk:Gangnam Woodford|talk]]) 17:29, 30 December 2024 (UTC) |
|||
:{{to|Gangnam Woodford}} "Congratulations", you've just tripped onto one of the fun little copyright law landmines that are out there (alternately: "job security for lawyers"). Everything on [[commons:|Wikimedia Commons]] must be public domain in '''''both''''' the US, '''and''' origin country. This means you have to care about US copyright law and all its nuances: see [[commons:COM:Hirtle chart|Hirtle chart]]. Unless the picture was like put on display in an art gallery or something, it is an "unpublished" work and therefore ''still under copyright in the US until 2035!'' (120 yrs from creation date) Really! Welcome to copyright law! |
|||
:{{br}} |
|||
:Here are the practical implications for you in this case: Is this image going to be used in an article? If so, upload it ''here on enwp'' (not Commons) by [[Special:Upload|following this link]], and tag it {{tl|PD-UK-unknown}}, as well as adding a [[WP:fair use rationale|fair use rationale]] since it's still under US copyright. |
|||
:Add it to the article(s), nothing more needed—some bot should automatically transport it over to Commons when it finally lapses into PD-US. However, if you just, say, want to use the image on your [[WP:user page|user page]], I'm afraid you aren't going to be able to for 10 more years. So stick a pin in the calendar for that. --[[User:Slowking Man|Slowking Man]] ([[User talk:Slowking Man|talk]]) 19:54, 31 December 2024 (UTC) |
|||
== An Article About CK == |
|||
:: ... and, of course, an autobahn is a motorway, not a train. --[[User:David Biddulph|David Biddulph]] ([[User talk:David Biddulph|talk]]) 18:36, 28 January 2017 (UTC) |
|||
{{resolved|1=Young person goofing off --[[User:Slowking Man|Slowking Man]] ([[User talk:Slowking Man|talk]]) 20:14, 31 December 2024 (UTC)}} |
|||
{{collapse top}} |
|||
I was asked by christen kuikoua representation to write an article about him and it was declined saying I didn't correctly reference it. Please if anyone wishes to jump in and help me with Will appreciate it [[Draft:Christen Kuikoua]] |
|||
[[User:Silvernet123|Silvernet123]] ([[User talk:Silvernet123|talk]]) 18:27, 30 December 2024 (UTC) |
|||
:Thank you [[User:Psiĥedelisto|ZaxAttrach]]! If you're wondering, the example I found this in was [[Loharu fort]]. Zachary G. 18:33, 28 January 2017 (UTC) |
|||
::Hi {{u|ZachAttrax}}. Use quotes (italics are preferred) only when talking about a word as a word. See [[MOS:WORDSASWORDS]]. Parenthesis are OK, but in the case you show, you would wikilink to [[autobahn]] instead of giving a definition. By the way, an autobahn is a German federal highway, not a train. [[User:StarryGrandma|StarryGrandma]] ([[User talk:StarryGrandma|talk]]) 18:41, 28 January 2017 (UTC) |
|||
:Hello {{u|Silvernet123}}, I've left some instructions on your talk page about the [[WP:DISCLOSEPAY|mandatory paid editing disclosure]] that is required by Wikimedia's terms of service. Please follow those instructions before making any other edits. -- [[User:DandelionAndBurdock|D'n'B]]-''[[User_talk:DandelionAndBurdock|📞]]'' -- 19:03, 30 December 2024 (UTC) |
|||
==Assessing an article== |
|||
::Okay thank you [[User:Silvernet123|Silvernet123]] ([[User talk:Silvernet123|talk]]) 19:05, 30 December 2024 (UTC) |
|||
I'm a new user (about 800 edits) and I've been working on the article [[Guccio di Mannaia]] which is categorized as "Start-Class", but I'd like to ask someone to take a look at it to advise me how it might be improved. Is there a particular place to ask? |
|||
:::{{u|Silvernet123}}, are you actually being paid to write complete garbage like {{tpq|He is known as one of the most philosophical minds in the 21st century with a record of 200+ Quotations and 89+ Poetry highlighting themes of love, patience, self-worth, discipline, and Christ-like values}} about an unknown 17 year old? That is the complete opposite of how an encyclopedia article should be written. Would you take a job as a chef if you don't know how to cook? [[User:Cullen328|Cullen328]] ([[User talk:Cullen328|talk]]) |
|||
::::{{u|Silvernet123}} I would add that our incentive to help you is low......if you're being paid to be here, it's up to you to learn our standards and what we're looking for. We're here for free. [[User:331dot|331dot]] ([[User talk:331dot|talk]]) 19:18, 30 December 2024 (UTC) |
|||
::::That's fair. I appreciate your feedback and will take the time to learn and meet Wikipedia's standards. Thank you for pointing me in the right direction. but still if you can assist I will appreciate [[User:Silvernet123|Silvernet123]] ([[User talk:Silvernet123|talk]]) 19:25, 30 December 2024 (UTC) |
|||
:::::Hello, @[[User:Silvernet123|Silvernet123]]. This is probably not what you want to hear, but: {{User:ColinFine/PractiseFirst}} [[User:ColinFine|ColinFine]] ([[User talk:ColinFine|talk]]) 20:59, 30 December 2024 (UTC) |
|||
:::::The entire article is written as self-promotion. One of your references is to fan writing on Medium claiming "(Account Not Own By Christen Kuikoua)", but appears to be copy/paste of material written by the subject. Promotional material does not belong in Wikipedia. Trying to create some fake noteriety by sneaking badly ghost-written articles into Wikipedia and elsewhere is unethical. If you wish to fake fame, then low-accountability social meda sites are probably more appropriate. [[User:Just Al|Just Al]] ([[User talk:Just Al|talk]]) 21:15, 30 December 2024 (UTC) |
|||
:Yo, I got here ‘cause I’ve been hearing you say I paid or my "representation" paid you to write an article. Like, not trying to be rude, but you gotta chill. I didn’t send you anywhere, and no one representing me sent you either. Bro, if you’re one of my friends messing around, you gotta stop. |
|||
:I looked at the draft, and while it’s kind of you to try writing something for me, you gotta do it the right way. Don’t go around saying I sent you somewhere—just saying, it’s not a good look. I’d like Wikipedia to delete the draft because I didn’t pay anyone, and I’m not planning on paying anyone either. [[User:Christenkofficial|Christenkofficial]] ([[User talk:Christenkofficial|talk]]) 23:54, 30 December 2024 (UTC) |
|||
::Hi Christenkofficial, |
|||
::Thank you for reaching out and clarifying your position. First, I want to say that I genuinely admire your work, which is why I was inspired to write about it. However, I now realize that my actions may have caused confusion or unintended trouble for you. I’m really sorry for using your name like that without your knowledge or permission it wasn’t my intention to create any issues. |
|||
::If you’d like the draft to be deleted, I completely understand and will fully support that. Thank you for bringing this to my attention, and I’ll be more careful moving forward. |
|||
::Best regards, |
|||
::Silvernet123 [[User:Silvernet123|Silvernet123]] ([[User talk:Silvernet123|talk]]) 00:09, 31 December 2024 (UTC) |
|||
{{collapse bottom}} |
|||
:Note that the account {{u|Christenkofficial}} has (correctly or not, I'm just reporting) now been blocked as a sockpuppet of {{u|Silvernet123}}, per [[Wikipedia:Sockpuppet investigations/Silvernet123]]. {The poster formerly known as 87.81.230.195} [[Special:Contributions/94.1.223.204|94.1.223.204]] ([[User talk:94.1.223.204|talk]]) 03:35, 31 December 2024 (UTC) |
|||
Thanks! |
|||
== How to add sources == |
|||
[[User:TimeForLunch|TimeForLunch]] ([[User talk:TimeForLunch|talk]]) 13:29, 28 January 2017 (UTC) |
|||
I need a simplified tutorial in how to add the number reference and the cite the source. [[User:M. Chris Tucker|M. Chris Tucker]] ([[User talk:M. Chris Tucker|talk]]) 19:02, 30 December 2024 (UTC) |
|||
::Hello, {{u|TimeForLunch}}. I think that the most obvious area for improvement would be to add a photo of the Chalice of Nicholas IV, which seems to be his best known work. It may take some effort to find (or take) a freely licensed photo, but adding one would certainly enhance the reader's understanding. [[User:Cullen328|<b style="color:#070">Cullen</b><sup style="color:#707">328</sup>]] [[User talk:Cullen328|<span style="color:#00F">''Let's discuss it''</span>]] 18:10, 28 January 2017 (UTC) |
|||
:::{{ec}}That's an interesting topic, {{u|TimeForLunch}}, and an interesting article – nice work! My first stop for info on that sort of figure is always the remarkable ''Dizionario Biografico degli Italiani'', which I see you've already consulted; those articles tend to have a very full bibliography, and following up those leads might perhaps lead to some further information. Many of them are likely to be quite hard to track down. On the article ''qua'' article, some quick changes you could make might be: (1) more wikilinks – not everyone knows what a [[chalice]] is, for example; (2) cut out "deceased" and any other euphemisms, replace with "dead" or other plain English word; (3) be ruthless with anything that might appear as [[WP:peacock|peacock]]ery or [[WP:puff|puff]]ery – "widely considered a masterpiece" is almost certainly true, but is not something you need to say here, it should be obvious from the description; (4) expand general statements where it's easy to do so – even if the source says "the papal court", it should be pretty straightforward to look up who the actual popes were that Pace di Valentino worked for; (5) build round your article – an article on Pace, even if brief, would add context (and a wikilink) to Guccio; (6) (should probably have been #1) do what {{u|Cullen328}} suggests; (7) use human-readable dates! – we aren't machines, ordinary mdy or dmy works fine here. Regards, [[User:Justlettersandnumbers|Justlettersandnumbers]] ([[User talk:Justlettersandnumbers|talk]]) 19:51, 28 January 2017 (UTC) |
|||
:Hello {{user|M. Chris Tucker}}, might I suggest ''[[Help:Referencing for beginners|Referencing For Beginners]]''. -- [[User:DandelionAndBurdock|D'n'B]]-''[[User_talk:DandelionAndBurdock|📞]]'' -- 19:06, 30 December 2024 (UTC) |
|||
==Template to tag inadequate ref?== |
|||
:Hi @[[User:M. Chris Tucker|M. Chris Tucker]] and welcome to Wikipedia! The simplest way to add references is, directly after the sentence or paragraph that your source supports, add <nowiki><ref></nowiki>, followed by the text of your reference, followed by <nowiki></ref></nowiki>. The software will sort out the numbering for you. Others here should be able to give you more detailed advice, and the guide that D'n'B linked above looks like a great place to learn more. Thanks for your additions to [[:Edward Dickson (Canadian politician)]]! Best, [[User:Wham2001|Wham2001]] ([[User talk:Wham2001|talk]]) 19:07, 30 December 2024 (UTC) |
|||
I was looking at [[Alphabetic_principle]], doing a bit of copyediting, and found several refs that were badly formatted— "cite" template without "ref" tags, so the ref was expanded in the text itself; URL and title with no tagging at all— and was able to fix them up at least somewhat. But this one, in § [[Alphabetic_principle#Role_in_beginning_reading|Role in beginning reading]] (second paragraph, second sentence), looks hopeless: |
|||
:: [[Special:Contributions/2607:FEA8:7D00:95A0:FDD3:6EB9:68D5:D6D7|2607:FEA8:7D00:95A0:FDD3:6EB9:68D5:D6D7]] ([[User talk:2607:FEA8:7D00:95A0:FDD3:6EB9:68D5:D6D7|talk]]) 00:38, 31 December 2024 (UTC) |
|||
: <code><nowiki><ref>Chall</ref></nowiki></code> |
|||
:::Thanks very much! I will follow up on your suggestion and wish you a Happy New Year. [[User:M. Chris Tucker|M. Chris Tucker]] ([[User talk:M. Chris Tucker|talk]]) 00:42, 31 December 2024 (UTC) |
|||
It wouldn't be so bad if there were a Reference or External link or such with some such name, but there's no such beast. |
|||
I looked for a template to tag it with, but couldn't find any such; the best I could do was |
|||
: <code><nowiki>{{clarify|reason=reference is a single name, totally inadequate}}</nowiki></code> |
|||
before the "/ref" tag. Is there anything more suitable? Please ping me to answer. --[[User:Thnidu|Thnidu]] ([[User talk:Thnidu|talk]]) 08:46, 28 January 2017 (UTC) |
|||
== How do I add a "Main Article" to Wikipedia == |
|||
:{{Ping|Thnidu}} I'd have thought {{tlx|better source|reason=}} would be appropriate...but that just may reduce to the same thing as {{tl|clarify}}. Any sort of remotely likely tag with a reason= parameter may be sufficient for the purpose: leave an in-line reason why some more editing work is required, get the page listed in a cleanup category, then drop it back into the vast lake to be picked up at some future date by you or someone else who has time and interest in fixing the problem more permanently. [[User:jmcgnh|<b><span style="color:#248F7D"> —jmcgnh</span></b>]]<sup><small><b>[[User_talk:jmcgnh|<span style="color:#58D582">(talk)</span>]] [[Special:Contributions/jmcgnh|<span style="color:#8F7D24">(contribs)</span>]]</b></small></sup> 08:57, 28 January 2017 (UTC) |
|||
::{{Ping|Thnidu}} That ref was introduced in [https://en.wikipedia.org/enwiki/w/index.php?title=Alphabetic_principle&direction=next&oldid=367613859 this edit] on 1 July 2010 by an IP editor who has never made any other edits. There is nothing in the context at the time to help with filling out the details. --[[User:Gronk Oz|Gronk Oz]] ([[User talk:Gronk Oz|talk]]) 09:21, 28 January 2017 (UTC) |
|||
:::{{Ping|Thnidu}} There is an author of that name who wrote about the development of reading skills, so seems to be appropriate - for example, {{cite journal |last1=Chall |first1=Jeanne S. |last2=Jacobs |first2=Vicki A. |title=Writing and Reading in the Elementary Grades: Developmental Trends Among Low SES Children |journal=Language Arts |date=1 January 1983 |volume=60 |issue=5 |pages=617–626 |url= http://www.jstor.org/stable/41961511}}. --[[User:Gronk Oz|Gronk Oz]] ([[User talk:Gronk Oz|talk]]) 09:27, 28 January 2017 (UTC) |
|||
::::The book in question is probably Chall, Jeanne S., [https://books.google.com/books?id=oCFhQgAACAAJ&dq=Jeanne+Chall%27s+(1967)+Learning+to+Read:+The+Great+Debate.&hl=en&sa=X&ved=0ahUKEwiTsoPywuTRAhUEKWMKHUDnCXgQ6AEILTAA ''Learning to Read: The Great Debate''], McGraw-Hill, 1967. This book is widely cited in other books that discuss the alphabetic principle. [[User:Cullen328|<b style="color:#070">Cullen</b><sup style="color:#707">328</sup>]] [[User talk:Cullen328|<span style="color:#00F">''Let's discuss it''</span>]] 09:34, 28 January 2017 (UTC) |
|||
:::::{{Ping|jmcgnh}} Thanks. There are so many templates, including about refs, and though I searched I just ''could not'' find an appropriate one. Your suggestions are helpful. |
|||
:::::{{ping| Gronk Oz |Cullen328}} Thank you both, comrades. With this I can put in a close-to-proper cite. --[[User:Thnidu|Thnidu]] ([[User talk:Thnidu|talk]]) 07:25, 29 January 2017 (UTC) |
|||
::::::{{ping| Gronk Oz |Cullen328|Thnidu}} The correct template would be {{tl|Full citation needed}} <span style="font-family: serif; letter-spacing: 0.1em">– [[User:Finnusertop|Finnusertop]]</span> ([[User talk:Finnusertop|talk]] ⋅ [[Special:Contributions/Finnusertop|contribs]]) 07:56, 29 January 2017 (UTC) |
|||
{{Od}} |
|||
Thank you, {{u|Finnusertop}}! <br> Respected more-experienced colleagues {{u| Gronk Oz }} & {{u|Cullen328}}, now I don't feel so bad about not finding it. ;-) <br>And in searching the Web for more info about the book, I found -- whaddayaknow? -- [[Jeanne Chall]], with a fuller citation for the ''1996'' edition! |
|||
--[[User:Thnidu|Thnidu]] ([[User talk:Thnidu|talk]]) 08:14, 29 January 2017 (UTC) |
|||
I updated our "Cornhusker Council" section under Boy Scouts of America, Nebraska Scouting. and would like to create a main article for our council. how do I do that? |
|||
==Notability requirement for a list that is inside another article == |
|||
If an article about a school, for example, has a section titled "Notable alumni" or "Notable faculty", is it required that each person listed have their own WP article? My understanding is that while [[WP:NOTABILITY|notability]] is a requirement to qualify for a stand-alone article on WP, the existence of a WP article does not necessarily confer notability, and furthermore, there are many notable people who do not have WP articles. So if I as an editor, add a person to such a list, ''with proper sourcing proving notability'', my entry should not be deleted solely on the basis that the person does not have a WP article correct? That's my understanding of [[WP:Source list]]. Am I wrong? --[[User:Cohler|<b>''TheClarinetGuy''</b> <sub>''talk''</sub>]] 07:12, 28 January 2017 (UTC) |
|||
:Welcome to the Teahouse, {{u|Cohler|TheClarinetGuy}}. You are correct that not every entry on every single list must have its own Wikipedia article. However, those entries must be verifiable. In the case of those lists designated as lists of "notable" people, though, the standard for inclusion is the existence of an acceptable Wikipedia article. If you believe that you have "proof" that a person is notable, and want to add them to such a list, then the solution is obvious: Write an acceptable Wikipedia article about that person and then add them to the list. There should be no Wikipedia articles about non-notable people. If such articles exist, then they should be deleted and removed from such lists. |
|||
[[User:Cornhusker324|Cornhusker324]] ([[User talk:Cornhusker324|talk]]) 21:15, 30 December 2024 (UTC) |
|||
:This standard prevents these lists of alumni and faculty members from growing to a bloated state, which they most certainly would it we did not limit inclusion on these lists to people who are already the subjects of acceptable Wikipedia biographies. [[User:Cullen328|<b style="color:#070">Cullen</b><sup style="color:#707">328</sup>]] [[User talk:Cullen328|<span style="color:#00F">''Let's discuss it''</span>]] 07:33, 28 January 2017 (UTC) |
|||
:{{u|Cornhusker324}} Hello and welcome to the Teahouse. First, please see [[WP:COI|conflict of interest]], as that needs to be formally disclosed. For a standalone article about your specific council, you would need to show with significant coverage in independent [[WP:RS|reliable sources]] that your council meets [[WP:ORG|the special Wikipedia definition of a notable organization]]. That coverage can't just be the reporting of its routine activities, but in depth coverage as to what makes your council important/significant/influential. Be aware that writing a new article is the most difficult task to attempt on Wikipedia, and it's even harder with a conflict of interest. Also be aware that an article is [[WP:PROUD|not necessarily desirable]]. [[User:331dot|331dot]] ([[User talk:331dot|talk]]) 21:23, 30 December 2024 (UTC) |
|||
::Hello @[[User:Cornhusker324|Cornhusker324]]. I have just reverted your edits in accordance with our conflict of interest policy. Please request specific edits on the article's respective [[help:Talk pages|talk page]]. Thanks. [[User:Tarlby|<span style="color:cyan;font-family:Comic Sans MS;">''Tarl''</span><span style="color:orange;font-family:Comic Sans MS;">''by''</span>]] <sup>([[User talk:Tarlby|''t'']]) ([[Special:Contributions/Tarlby|''c'']])</sup> 21:25, 30 December 2024 (UTC) |
|||
:::And I'm not sure why the council itself would merit a standalone article from the article about scouting in Nebraska. [[User:331dot|331dot]] ([[User talk:331dot|talk]]) 21:31, 30 December 2024 (UTC) |
|||
::::Many other Councils already have a standalone article. I would think this would be supported for consistency sake. [[User:Cornhusker324|Cornhusker324]] ([[User talk:Cornhusker324|talk]]) 21:35, 30 December 2024 (UTC) |
|||
:::::@[[User:Cornhusker324|Cornhusker324]], please see [[WP:42]]. Some council have articles. Others don't, and maybe they shouldn't. It all depends on the [[wp:rs|reliable sources]] that we need to use so that an article can be made. An article on Cornhusker Council won't be made if there are no such sources. [[User:Tarlby|<span style="color:cyan;font-family:Comic Sans MS;">''Tarl''</span><span style="color:orange;font-family:Comic Sans MS;">''by''</span>]] <sup>([[User talk:Tarlby|''t'']]) ([[Special:Contributions/Tarlby|''c'']])</sup> 21:40, 30 December 2024 (UTC) |
|||
::::::Noted [[User:Cornhusker324|Cornhusker324]] ([[User talk:Cornhusker324|talk]]) 21:43, 30 December 2024 (UTC) |
|||
:::I have changed my user name in hopes of complying with your COI policy. The stand-alone article seems too difficult at this point. Perhaps in the future. |
|||
:::As such, I would simply like to update our section, "Cornhusker Council," in "Scouting in Nebraska." and will attempt that in the 'talk page,' as you suggested. [[User:Cornhusker324|Cornhusker324]] ([[User talk:Cornhusker324|talk]]) 21:42, 30 December 2024 (UTC) |
|||
Your name change does not show up yet, and regardless of a name change, you still have a COI. I see that you are proposing changes on the Talk page of the article, but much of what you wrote is not referenced. See [[Help:Referencing for beginners]] to understand how to insert inline refs. [[User:David notMD|David notMD]] ([[User talk:David notMD|talk]]) 05:30, 31 December 2024 (UTC) |
|||
::{{re|Cullen328}} Thank you for your quick response. Can you point me to what policy states that a WP article is the requirement for inclusion in "notable" lists? Also, then as a specific example, are you saying that all red-linked people on this [[List of Boston Conservatory people]] should be removed? Before, I go ahead and do that, I would like to make sure that I cite the correct policy. --[[User:Cohler|<b>''TheClarinetGuy''</b> <sub>''talk''</sub>]] 07:40, 28 January 2017 (UTC) |
|||
:::Hello again, {{u|Cohler}}. Our editing is guided not only by formal policy, but also by behavioral guidelines, consensus and good sense. I have just read the discussion that led to your recent topic ban. I therefore feel obligated to warn you to refrain from any form of tendentious or disruptive editing. In my opinion, you are skating on very thin ice here, and would be well advised to stay away from this area. [[User:Cullen328|<b style="color:#070">Cullen</b><sup style="color:#707">328</sup>]] [[User talk:Cullen328|<span style="color:#00F">''Let's discuss it''</span>]] 07:54, 28 January 2017 (UTC) |
|||
::::Why? The topic ban is on [[Jonathan Cohler]]. I'm allowed to edit other things right? Boston Conservatory is not related to "Jonathan Cohler" other than I am a notable faculty member there. As long as I don't edit anything about myself there, isn't that allowed? --[[User:Cohler|<b>''TheClarinetGuy''</b> <sub>''talk''</sub>]] 08:24, 28 January 2017 (UTC) |
|||
:::::Writing about the faculty where you work is a conflict of interest. Since you have been topic banned from writing about yourself for COI reasons, this seems like a particularly bad idea. Pinging [[User:Doc James]] who should have the definitive answer on this. [[User:Meters|Meters]] ([[User talk:Meters|talk]]) 08:34, 28 January 2017 (UTC) |
|||
:::::::Hum. Yes do not edit about your work or the people you work with either. [[User:Doc James|<span style="color:#0000f1">'''Doc James'''</span>]] ([[User talk:Doc James|talk]] · [[Special:Contributions/Doc James|contribs]] · [[Special:EmailUser/Doc James|email]]) 08:49, 28 January 2017 (UTC) |
|||
::::::::{{u|Cohler}}, I agree with Doc James. In my opinion, you should stay a long way away from any article or list where you have even the slightest trace of a conflict of interest, even if they are outside the narrow confines of your topic ban. Your edits are subject to heightened scrutiny because of your history. You should defer to the opinions of the many experienced editors lacking any conflict of interest about these articles. [[User:Cullen328|<b style="color:#070">Cullen</b><sup style="color:#707">328</sup>]] [[User talk:Cullen328|<span style="color:#00F">''Let's discuss it''</span>]] 08:52, 28 January 2017 (UTC) |
|||
:::::{{ec}}Hi {{u|Cohler|TheClarinetGuy}}. I think what {{u|Cullen328}} is trying to say is that there was probably something about your editing at "Jonathan Cohler" which led to the community deciding to topic ban you from editing that article. So, if you try and use the same approach on another article, then your editing their will probably come under scrutiny as well. |
|||
:::::As for "Notable alumini", I think what you're looking for is [[:WP:LSC]], in particular [[:WP:CSC]]. Individual entries of such a list do not have to have a stand-alone article written about them, but that is typically the basic criterion for inclusion. If all the entries in the list have stand-alone articles, then you can assume that is probably the consensus for inclusion established for that particular article. If some of the entries don't have stand-alone articles but are supported by a citation to a reliable source, then you can assume that is probably what is needed in such cases. There is no one "common selection criteria" that applies to all article across Wikipedia, so often times you have to check the article's talk page to see it the subject has been discussed before. If not, then you can start a discussion yourself, or you can simply be bold and at the name to the list. If you're bold and reverted, then follow [[:WP:BRD]]. There is no automatic inclusion for such names, and like other article content you may have to reach a consensus for it on the article's talk page. -- [[User:Marchjuly|Marchjuly]] ([[User talk:Marchjuly|talk]]) 09:08, 28 January 2017 (UTC) |
|||
::::::Understood, and thank you for all the clear, informative and substantive answers. --[[User:Cohler|<b>''TheClarinetGuy''</b> <sub>''talk''</sub>]] 14:11, 28 January 2017 (UTC) |
|||
:I provided four references which is more than what is currently listed. Those currently used are not authoritative and factually incorrect. [[Special:Contributions/209.92.187.50|209.92.187.50]] ([[User talk:209.92.187.50|talk]]) 15:34, 31 December 2024 (UTC) |
|||
==How do I add (copy, insert, paste) a photo (picture) into my draft article?[[User:CableHut|CableHut]] ([[User talk:CableHut|talk]]) 06:23, 28 January 2017 (UTC)== |
|||
::How exactly did you change your username? It should be done via either [[Special:GlobalRenameRequest]] or [[WP:CHUS]]. You should continue to use your original username until it is changed, at which time you can begin using the new name. [[User:331dot|331dot]] ([[User talk:331dot|talk]]) 15:48, 31 December 2024 (UTC) |
|||
I have looked all over the Help features but cannot find any instructions on how to do this![[User:CableHut|CableHut]] ([[User talk:CableHut|talk]]) 06:23, 28 January 2017 (UTC) |
|||
::Never mind, I see it is changed now. [[User:331dot|331dot]] ([[User talk:331dot|talk]]) 15:49, 31 December 2024 (UTC) |
|||
:Hello, {{u|CableHut}}, and welcome to the Teahouse. Image must be uploaded first (usually on [[c:Special:UploadWizard|Wikimedia Commons]]). The most important thing to keep in mind is copyrights. Most images you'll find on the Internet, in books, and magazines do not meet the strict [[c:Commons:Licensing|licensing requirement]]: the copyright holder needs to have explicitly stated that they allow anyone to use the photo for any purpose, including commercially. Images are not the most important component of Wikipedia articles; they are the icing on the cake. I'd suggest you improve your draft [[Draft:Jean Jepson: Dancer; Choreographer; Teacher.]] in other ways first. You should, for instance, put the references right after the content they support (the list at the end will be created automatically): [[Help:Referencing for beginners#Test it out]]. <span style="font-family: serif; letter-spacing: 0.1em">– [[User:Finnusertop|Finnusertop]]</span> ([[User talk:Finnusertop|talk]] ⋅ [[Special:Contributions/Finnusertop|contribs]]) 06:44, 28 January 2017 (UTC) |
|||
:::That is not how references are created. Also, you now appear to be editing as an IP address, i.e., not logged into an account versus a name-change. [[User:David notMD|David notMD]] ([[User talk:David notMD|talk]]) 16:54, 31 December 2024 (UTC) |
|||
:Hi {{u|CableHut}}, [[Help:Files]] gives you the basics of adding images to articles, images and sound files are both types of file so it is almost logical to hide those instructions there. I have a training module on this at [[User:WereSpielChequers/image_adding]] which is ready for beta testers, feel free to check it out. ''[[User:WereSpielChequers|<span style="color:DarkGreen">Ϣere</span>]][[User talk:WereSpielChequers|<span style="color:DarkRed">Spiel</span>]]<span style="color:#CC5500">Chequers</span>'' 11:53, 28 January 2017 (UTC) |
|||
::::I've used two different means to source the information I provided, both of which were recommended by this site and/or a moderator. [[User:CCJLJ|CCJLJ]] ([[User talk:CCJLJ|talk]]) 17:04, 31 December 2024 (UTC) |
|||
:::::Again, see Help:Referencing for beginners to understand how to insert inline refs. What you posted on the Talk page of the article (below) are not properly formatted references. [[User:David notMD|David notMD]] ([[User talk:David notMD|talk]]) 21:22, 31 December 2024 (UTC) |
|||
References supporting change: |
|||
*Salistean, John, "A History of the Cornhusker Council 1940-1975," Houchen Bindery LTD of Utica, Nebraska, 1st ed., 2011. |
|||
*Golden Sun Lodge Website, www.goldensunlodge.org |
|||
*Cornhusker Council Website, www.cornhuskercouncil.org |
|||
*Brown, Elinor L., "History of Lancaster County, Then and Now," ASIN B0006CJTC2, Jan 1971. |
|||
== Notability and Independence. == |
|||
==removing template messages in an user space draft == |
|||
I've been adding citations, they are numbered, yet don't show on the list. I'm afraid to leave the draft's page (superstition...) in order to read the related info. Can you please help? |
|||
שוחרת[[User:שוחרת|שוחרת]] ([[User talk:שוחרת|talk]]) 01:36, 28 January 2017 (UTC) |
|||
:Hello, {{u|שוחרת}}, and welcome to the Teahouse. Without saving the draft, it's really difficult for us to guess what's wrong. Here are the instructions for citations anyhow: [[Help:Referencing for beginners]] <span style="font-family: serif; letter-spacing: 0.1em">– [[User:Finnusertop|Finnusertop]]</span> ([[User talk:Finnusertop|talk]] ⋅ [[Special:Contributions/Finnusertop|contribs]]) 06:47, 28 January 2017 (UTC) |
|||
::Hello, {{u|שוחרת}}. I agree with Finnusertop. Other editors cannot evaluate any unsaved changes that you may have made. Based on the last saved version of your draft, those tags should not be removed, because the problems with the draft have not yet been resolved. [[User:Cullen328|<b style="color:#070">Cullen</b><sup style="color:#707">328</sup>]] [[User talk:Cullen328|<span style="color:#00F">''Let's discuss it''</span>]] 06:52, 28 January 2017 (UTC) |
|||
:: Hi {{u|שוחרת}}. Unfortunately I do not read Hebrew, so I am not sure how to read your name. It's OK to use non-Latin script characters in your username per [[:WP:UN#Usernames with non-Latin characters]], but it might make it a bit easier for others to communicate with you and help you if you added some easy to type characters to your signature as explained in [[:WP:NLS]]. Not every editor is using a keyboard which can type Hebrew script, so they may not be able to address you by your username. You can customize your signature so that the Hebrew characters show when you sign your posts per [[:WP:CUSTOMSIG]], and then change your name to something a bit easier for more editors to understand if you want. |
|||
::Finally about your draft, its seem from [[:User talk:Kudpung#Dear Kudpung]] you are trying to writie an [[:WP:AUTO|autobiography]]. If you are the same [[:he:קורינה_הסופרת]], who is the subject of your draft [[:User:שוחרת/Corinna Hasofferett]], then I suggest you read [[:Wikipedia:Plain and simple conflict of interest guide]], [[:WP:COI#Miscellaneous|Wikipedia's Law of Unexpected Consequences]] and [[:Wikipedia:An article about yourself isn't necessarily a good thing]]. Hebrew Wikipedia and English Wikipedia are part of the [[:Wikimedia Foundation]] family, but English Wikipedia probably has a lot more people from around the world editing it and sometimes [[:WP:VANDAL|some of these people do not do so with the best of intentions]]. Being the creator of an article or the subject of an article does not give you any final editorial control over any edits to the article per [[:Wikipedia:Ownership of content]] and [[:Wikipedia:Biographies of living persons]], so it's important to understand that right from the start. -- [[User:Marchjuly|Marchjuly]] ([[User talk:Marchjuly|talk]]) 09:44, 28 January 2017 (UTC) |
|||
My grandmother, Ethel Margaret Streit Harrison, was the first woman elected as Clerk of the Montana Supreme Court, one of the founders of the Montana Association of Female Executives and one of the original board members of the Holter Museum of Art. There are printed sources that talk about her achievements but according to wikipedia the only thing notable about her was that she was married to [[John C. Harrison (judge)]]. I understand that, as a relative, there is a potential conflict here, but I think it is important her contributions are documented on something other than microfiche. Any thoughts? / Thank you :) [[User:Mehap dwhx|Mehap dwhx]] ([[User talk:Mehap dwhx|talk]]) 22:40, 30 December 2024 (UTC) |
|||
Thank you all good people, Cullen, [[User:Marchjuly|Marchjuly]] and Finnusertop. I'm indeed working intenslynon this article, as the perfectionist I am in all I attempt to do. It is still work in progress and will for sure take me a while, but I love it and am sure will continue to write missing articles for wikipedia, now that thanks to your continuous help I'm starting to grasp the handles and the behind the scene dramas. What I like most is the transparency. I wish we could import it into our daily political life. |
|||
:[[User:Mehap dwhx|Mehap dwhx]], none of those three roles that you describe automatically confers [[WP:N|Wikipedia-defined "notability"]] on a person. And the sum of the three doesn't either. But it's possible that she's "notable" all the same. Regardless of your conflict of interest, you're free to create [[Draft:Ethel Margaret Streit Harrison]]. If this demonstrates her notability and follows Wikipedia's other policies, it will be promoted to an article (possibly with a slightly different title), whereupon you shouldn't continue to edit it but would be free to make suggestions and requests on its talk page. -- [[User:Hoary|Hoary]] ([[User talk:Hoary|talk]]) 23:12, 30 December 2024 (UTC) |
|||
I will relate in good time to some of the issues you raised. Right now what baffles me is the fact that while inserting citations, they got numbered alright at the specific location, yet they do not add up to the single citation at the bottom list for citations. My question is why is that so and could it be remediated? |
|||
In Thanks, |
|||
[[User:שוחרת|שוחרת]] ([[User talk:שוחרת|talk]]) 13:56, 28 January 2017 (UTC)שוחרת |
|||
[[User:שוחרת|שוחרת]] ([[User talk:שוחרת|talk]]) 14:00, 28 January 2017 (UTC)שוחרת |
|||
:Hi again [[User:שוחרת|שוחרת]]. I am a bit confused. The draft you've been working on has no citations included in it at all. In a way, this gets back to the questions I answered yesterday, where I prefaced one of my comments by saying "[i]f the latest saved version is the version you are talking about..." Based on the above, it seems it is not. If you don't save your edits to the draft, then (still on the question from yesterday), you will lose that material if you log off and don't save (unless, of course, you save it offline, in a Word document or the like). Anyway, because it seems you have not saved the edits where the citation issue has come up, it's a bit opaque what the issue actually is. Have you included a dedicated references section with a {{tlx|reflist}} template in it? ''Viz'': |
|||
::<nowiki>==References==</nowiki> |
|||
::<code><nowiki>{{Reflist|30 em}}</nowiki></code> |
|||
:Here's some links to pages that might help: [[Help:Referencing for beginners]] and [[Help:Introduction to referencing/1]]; more involved: [[Wikipedia:Citing sources]]; there are numerous others, and each I've linked contains see also sections linking to additional help, guides and tutorials. Also, there's a visual guide to placing inline citations through <code><nowiki><ref> ... </ref></nowiki></code> tags that I am posting below (just click show). Best regards--[[User:Fuhghettaboutit|Fuhghettaboutit]] ([[User talk:Fuhghettaboutit|talk]]) 14:42, 28 January 2017 (UTC) |
|||
{{hidden begin |
|||
|title = Visual inline citation guide |
|||
|titlestyle = background: LightSteelBlue; text-align: center; |
|||
}} |
|||
{| class="wikitable" |
|||
|+ Formatting references using inline citations |
|||
|- style="vertical-align: top;" |
|||
| colspan=2 | <small>All information in Wikipedia articles should be [[Wikipedia:Verification|verified]] by [[Wikipedia:Citing sources|citations]] to [[Wikipedia:Reliable sources|reliable sources]]. Our preferred method of citation is using the <span class="plainlinks">"[http://www.mediawiki.org/wiki/Extension:Cite/Cite.php cite.php]"</span> form of inline citations, using the <nowiki><ref></ref></nowiki> elements. Using this method, each time a particular source is mined for information (''don't copy word-for-word''!), a footnote is placed in the text ("inline"), that takes one to the detail of the source when clicked, set forth in a references [[Help:Section|section]] after the text of the article.<p> |
|||
In brief, anywhere you want a footnote to appear in a piece of text, you place an opening <nowiki><ref></nowiki> tag followed by the text of the citation which you want to appear at the bottom of the article, and close with a <nowiki></ref></nowiki> tag. Note the closing slash ("/"). For multiple use of a single reference, the opening ref tag is given a name, like so: <nowiki><ref name="name"></nowiki> followed by the citation text and a closing <nowiki></ref></nowiki> tag. Each time you want to use that footnote again, you simply use the first element with a slash, like so: <nowiki><ref name="name" /></nowiki>.<p>In order for these references to appear, you must tell the software where to display them, using either the code <nowiki><references/></nowiki> or, most commonly, the template, {{tl|Reflist}} which can be modified to display the references in columns using <nowiki>{{Reflist|colwidth=30em}}</nowiki>. Per our [[Wikipedia:Manual of Style|style guidelines]], the references should be displayed in a separate section denominated "References" located after the body of the article.</small> |
|||
|- style="vertical-align: top; text-align: center;" |
|||
| <small>'''''[[Wikipedia:Footnotes|Inline citation]] code; what you type in 'edit mode''''''</small> |
|||
| <small>'''''What it produces when you save'''''</small> |
|||
|- style="vertical-align: top;" |
|||
| style="width: 400px;"|Two separate citations.<span style="color: #002BB8;"><nowiki><ref></nowiki></span><code>{{xt|Citation text.}}</code><span style="color: #002BB8;"><nowiki></ref><ref></nowiki></span><code>{{xt|Citation text2.}}</code><span style="color: #002BB8;"><nowiki></ref></nowiki></span><p><br />Multiple<span style="color: #002BB8;"><nowiki><ref name="multiple"></nowiki></span><code>{{Xt|Citation text3.}}</code><span style="color: #002BB8;"><nowiki></ref></nowiki></span>citation<span style="color: #002BB8;"><nowiki><ref name="multiple" /></nowiki></span> use.<span style="color: #002BB8;"><nowiki><ref name="multiple" /></nowiki></span><p><br />== References ==<p><nowiki>{{Reflist}}</nowiki> |
|||
| style="width: 400px;"|Two separate citations.<ref>Citation text.</ref><ref>Citation text2.</ref><p><br /><br /> Multiple<ref name="multiple">Citation text3.</ref> citation<ref name="multiple" /> use.<ref name="multiple" /><p><u style="font-size: larger;"><br /><br /><br />'''References'''_________________</u><p>{{Reflist}} |
|||
|- |
|||
| style="text-align: center;" colspan="2" | <small>Templates that can be used between <nowiki><ref></ref></nowiki> tags to format references</small><p>{{tl|Citation}} • {{tl|Cite web}} • {{tl|Cite book}} • {{tl|Cite news}} • {{tl|Cite journal}} • [[:Category:Citation templates|Others]] • [[Wikipedia:Citation templates|Examples]] |
|||
|- |
|||
|}{{Z3}}<!-- Template:Refref --> |
|||
{{Hidden end}} |
|||
Thanks dear Fuhghettaboutit. I have so much more to study - tomorrow - as it is 2 am in Tel Aviv. |
|||
As for the draft I'm working on, can you see it indeed? It does have already 3 citations, more to come. I wish I could send a screen pic via email as I'm not sure it is possible to post a link here, or allowed. I've just discovered that the citation links appear at the very bottom of the draft format, not where they should. I'll try to decipher the problem tomorrow. In short, I'm working on this draft that upon completion should be approved for publication/saving. It might take me a while. Right now it is online and shown. |
|||
Thanks a lot, Good Night, |
|||
שוחרת/Renica 00:04, 29 January 2017 (UTC)שוחרת |
|||
שוחרת/Renica 00:08, 29 January 2017 (UTC) שוחרת |
|||
:To expand an aspect of Hoary's excellent advice above: in Wikipedia, '[[WP:Notability|Notability]]' boils down to "is there ''enough'' substantial material, published [[Wikipedia:Independent sources|independently]] of the subject, in multiple [[WP:Reliable sources|Reliable sources]], to form the basis of an article about the subject. It doesn't necessarily require that the subject is 'famous', or in one of many possible senses 'important'. |
|||
==how does one insert photos and can photos be considered a source reference?== |
|||
:Also, the fact that your grandmother's achievements are ''not yet'' in Wikipedia doesn't mean Wikipedia asserts they ''aren't'', or she as a whole ''isn't'', notable; merely that no volunteer editor has yet gathered the necessary sources and added the information (with [[WP:citations|citations]]), whether in her husband's article or in a draft for her own. I encourage you to try. Good luck! |
|||
how does one insert photos and can photos be considered a source reference? AvaAva Bianca 01:35, 28 January 2017 (UTC) |
|||
:While we're here, I notice that [[John C. Harrison (judge)|John C. Harrison's]] article is on shaky ground, because it's entirely cited to a single (though reliable) source: we usually prefer a minimum of three, so if you could flesh out ''that'' article with further referenced material, it would be a good thing. {The poster formerly known as 87.81.230.195} [[Special:Contributions/94.1.223.204|94.1.223.204]] ([[User talk:94.1.223.204|talk]]) 03:56, 31 December 2024 (UTC) |
|||
::Add '''Ethel Margaret Streit Harrison''' to: [[John_C._Harrison_(judge)#Personal_life]] |
|||
::[[Special:Contributions/69.181.17.113|69.181.17.113]] ([[User talk:69.181.17.113|talk]]) 04:45, 31 December 2024 (UTC) |
|||
:::can you find at least 4 [[WP:RS]] sources ? [[Special:Contributions/69.181.17.113|69.181.17.113]] ([[User talk:69.181.17.113|talk]]) 04:47, 31 December 2024 (UTC) |
|||
:::She's already mentioned in that section as Ethel Harrison, cited to the article's (only) reference. Even to add her middle names would require a further citation to a ''published reliable source'' that mentions them (the existing one does not: I've downloaded and searched it). {The poster formerly known as 87.81.230.195} [[Special:Contributions/94.1.223.204|94.1.223.204]] ([[User talk:94.1.223.204|talk]]) 06:56, 31 December 2024 (UTC) |
|||
::::Being married to JCH can be mentioned in her Personal life section, but does not contribute to establishing her notability. [[User:David notMD|David notMD]] ([[User talk:David notMD|talk]]) 05:34, 31 December 2024 (UTC) |
|||
== Restoring previously unmerged articles == |
|||
:Welcome, {{u|User:Comfortscience}}. Pictures are welcome, as mentioned in [[WP:Image]]. Difficulties exist, and the biggest one is usually [[WP:Copyright]]. If you snapped the photo, you probably own the copyright and can license it for Wikipedia and the world to use. If someone else made the picture, the photographer is usually the owner. Whoever the copyright owner is, must make a statement to give up the rights. After the legal part has been done properly, the technical parts are easier. |
|||
:Photos by themselves are usually not a reference for statements in the encyclopedia. They need to be backed up by a [[WP:Reliable source]] saying what they are, in which case those words are the source reference. [[User:Jim.henderson|Jim.henderson]] ([[User talk:Jim.henderson|talk]]) 02:05, 28 January 2017 (UTC) |
|||
I would like to create a new article but it [https://en.wikipedia.org/enwiki/w/index.php?title=Cyprus%E2%80%93Jordan%20relations&redirect=no once existed back in 2009] and got merged without consensus. Is it okay if I restore the article but with more and better sources from [[User:Underdwarf58/sandbox|my sandbox]] later on? Because one user tried to restore a merged article and it got immediately undone because a consensus existed to merge it. I'm not saying that I'll do it right now as it's still unfinished but I need to ask because if I can't then I'll just cancel. [[User:Underdwarf58|Underdwarf58]] ([[User talk:Underdwarf58|talk]]) 00:29, 31 December 2024 (UTC) |
|||
==how to edit== |
|||
People educated at North Sydney Boys High School. How do I edit that article. If I click edit source, the items on page do not appear![[User:Saki0710|Saki0710]] ([[User talk:Saki0710|talk]]) 00:51, 28 January 2017 (UTC) |
|||
:There's very little about the original subject 'Cyprus–Jordan relations' in [[Foreign relations of Cyprus]] that it was merged into, so ''I'' think that if ''you'' think you have enough [[Wikipedia:Reliable sources|Reliably sourced]] material (bearing in mind there's also been another 15 years of history) you should go ahead and create a new [[Wikipedia:Drafts|Draft]] on the subject, via the normal [[Wikipedia:Articles for creation]] process, which can be submitted for assessment. Good luck! {The poster formerly known as 87.81.230.195} [[Special:Contributions/94.1.223.204|94.1.223.204]] ([[User talk:94.1.223.204|talk]]) 04:11, 31 December 2024 (UTC) |
|||
:We have articles called [[North Sydney Boys High School]] and [[List of Old Falconians]]. I assume you're trying to edit the second of those? Make sure you click on the Edit Source link in the tab marker right at the top of the page. If you click on the edit link next to the title, you're only editing the lead section, before the first section subtitle. Please don't add names of random ex-pupils to the list. Only those with existing articles about them are eligible. [[User:Rojomoke|Rojomoke]] ([[User talk:Rojomoke|talk]]) 06:20, 28 January 2017 (UTC) |
|||
== template == |
|||
==Updating page with PCD/COI== |
|||
Hi all, |
|||
I'm an employee of an NGO, and the page about the org is really out of date with regards to basic information about where the headquarters are, who the CEO is etc. It would be great if that were updated, and I made the suggestions with citations on the Talk page (including full transparency about my affiliation with the org), but it seems none of the editors are active. Where to from here? Do I wait for someone to see it and update it, or is this the kind of information it's okay for an employee to update? |
|||
Hello, I'm need of some assistance with creating a Wikipedia page about a music artist. Can anyone advise which template to use? [[User:RATHOMP|RATHOMP]] ([[User talk:RATHOMP|talk]]) 02:45, 31 December 2024 (UTC) |
|||
I'm being very cautious and respectful of the whole PCD/COI thing so I don't want to barge in and make changes without knowing it's okay to do so, as it's a controversial page with controversial topics. |
|||
:look at other Wikipedia pages about music artists ... [[Special:Contributions/69.181.17.113|69.181.17.113]] ([[User talk:69.181.17.113|talk]]) 04:40, 31 December 2024 (UTC) |
|||
Here's the related talk page: |
|||
::Hello, IP user, and welcome to the Teahouse. From your question, I suspect that you are focussing on the layout of your proposed article (I'm guessing that that's what you mean by a "template" - we use the word a bit differently here). But while the layout of an article is important, it is MUCH less important than the quality of the sources used. Until you have found adequate sources to establish that the artist meets Wikipedia's criteria for [[WP:notability|}], it's pointless spending any time thinking about the content or the layout. To use a house-building analogy, you may have an idea for what you want your house to look like, and even a plan; but until you've surveyed the site to make sure it's fit to build on, and checked that your plans meet local building regulations, it would be a waste of effort to start building. |
|||
::More generally, {{User:ColinFine/PractiseFirst}} [[User:ColinFine|ColinFine]] ([[User talk:ColinFine|talk]]) 10:36, 31 December 2024 (UTC) |
|||
:Is there a specific place you're currently working on the article at? I can't seem to find it on your userpage as a subpage. For advice, I'd recommend looking at other music artist articles (specifically various quality articles from [[The Beatles]] [FA] to [[Sepultura]] [C-class]) for general outlines on how to write it. For infoboxes, use <nowiki>{{infobox musical artist}}</nowiki> and fill out the template using it's template page at [[Template:Infobox musical artist]]. For general advice on writing, see Wikipedia's content policies and guidelines, I'd recommend WP:Nutshell as a starting point. |
|||
:Thanks, [[User: Sparkle & Fade|''Sparkle and Fade'']] <sup>[[User_talk:Sparkle & Fade|talk]]</sup><sub>[[Special:Contributions/Sparkle & Fade|edits]]</sub> 08:23, 31 December 2024 (UTC) |
|||
== how can I rigth a article in wikipedia == |
|||
https://en.wikipedia.org/wiki/Talk:Sea_Shepherd_Conservation_Society |
|||
how can I rigth [[User:Daniel Muanga|Daniel Muanga]] ([[User talk:Daniel Muanga|talk]]) 03:49, 31 December 2024 (UTC) |
|||
Thanks in advance for any advice! |
|||
:[[Wikipedia:Article wizard]] |
|||
[[User:Tuberose87|Tuberose87]] ([[User talk:Tuberose87|talk]]) 23:36, 27 January 2017 (UTC) |
|||
:[[Special:Contributions/69.181.17.113|69.181.17.113]] ([[User talk:69.181.17.113|talk]]) 04:39, 31 December 2024 (UTC) |
|||
:Welcome to the Teahouse, {{u|Tuberose87}}. Can you please format the references that you have added to the talk page to include full bibliographic information, as opposed to bare URLs? You can find instructions at [[WP:REFBEGIN|Referencing for beginners]]. In addition, you should write specific proposed language to be added to the encyclopedia. I will review your changes once you have done so. Leave a note on my talk page. [[User:Cullen328|<b style="color:#070">Cullen</b><sup style="color:#707">328</sup>]] [[User talk:Cullen328|<span style="color:#00F">''Let's discuss it''</span>]] 23:44, 27 January 2017 (UTC) |
|||
::New editors are strongly advised to first gain skills by doing time improving existing articles. References required. [[User:David notMD|David notMD]] ([[User talk:David notMD|talk]]) 05:39, 31 December 2024 (UTC) |
|||
== |
== Find a Grave = WP:RS ? == |
||
working on a draft for a long time and yet not ready but so tired. How do I save a draft without having it published, just so that it waits for me nicely? |
|||
שוחרת [[User:שוחרת|שוחרת]] ([[User talk:שוחרת|talk]]) 21:57, 27 January 2017 (UTC) |
|||
Dear Checkingfax, Teahouse host |
|||
Is [[Find a Grave]] = [[WP:RS]] ? |
|||
Please respond as soon as possible, I'm afraid to leave the draft so it won't misbehave and then so many hours of work might get lost.[[User:שוחרת|שוחרת]] ([[User talk:שוחרת|talk]]) 22:03, 27 January 2017 (UTC) שוחרת |
|||
:Hi [[User:שוחרת|שוחרת]]. If this is about [[User:שוחרת/Corinna Hasofferett]], it's already saved as an unsubmitted user space draft, and, barring out of the ordinary matters, should stay that way, undisturbed, for you to work on at your leisure. When you are ready to submit it, just click on the {{Font color|white|blue|Submit Your Draft for Review!}} button.<p>If this is about another draft, one you have not yet saved, so long as you do so as a [[Help:Userspace draft|user space draft]], or save it in the [[Wikipedia:Drafts|draft namespace]], the same will apply. To save it as the former, simply preview anywhere (or save at your userpage) a link in this form: <kbd><nowiki>[[User:שוחרת/Intuitive Name For Topic]]</nowiki></kbd> → click on the red link revealed → paste your content → save. For the latter, do the same, but instead of using "User:שוחרת/NAME", use "Draft:NAME" as the title. Best regards--[[User:Fuhghettaboutit|Fuhghettaboutit]] ([[User talk:Fuhghettaboutit|talk]]) 22:04, 27 January 2017 (UTC) |
|||
[[Special:Contributions/69.181.17.113|69.181.17.113]] ([[User talk:69.181.17.113|talk]]) 04:37, 31 December 2024 (UTC) |
|||
===May I ask for clarification?=== |
|||
:{{welcometea}} Please consult [[Wikipedia:FINDAGRAVE]]. —[[User:Tenryuu|<span style="color:#556B2F">Tenryuu 🐲</span>]] ( [[User talk:Tenryuu|💬]] • [[Special:Contributions/Tenryuu|📝]] ) 05:03, 31 December 2024 (UTC) |
|||
Thanks you for your response, Fuhghettaboutit. So, I have nothing to worry about? the draft will stays undisturbed, it won't disappear and I do not need to click any saving button, even if I close my computer for the night?[[User:שוחרת|שוחרת]] ([[User talk:שוחרת|talk]]) 22:29, 27 January 2017 (UTC) |
|||
:The answer is "no" because Find a Grave consists of user generated content, and is specifically mentioned in the [[WP:USERGENERATED]] section of the reliable sources guideline as a source that should not be used as a reference on Wikipedia. That does not mean that Find a Grave is of no value to Wikipedia editors. You may be able to find nuggets in those listings that will inform your searches about various people. Some but not all Find a Grave listings include references to reliable sources, and those sources may be useful as Wikipedia references. One thing that Find a Grave can teach editors is that many people down through the years share the same name, and we need to be very careful to avoid including biographical details about one Andrew Wilson in an article about another Andrew Wilson. That's just one of countless examples. I have been working on [[Andrew Stephen Wilson]] today, so that's why I chose that example.. [[User:Cullen328|Cullen328]] ([[User talk:Cullen328|talk]]) 06:22, 31 December 2024 (UTC) |
|||
שוחרת[[User:שוחרת|שוחרת]] ([[User talk:שוחרת|talk]]) 22:29, 27 January 2017 (UTC) |
|||
[[User:שוחרת|שוחרת]] ([[User talk:שוחרת|talk]]) 22:32, 27 January 2017 (UTC)שוחרת |
|||
:Hi again [[User:שוחרת|שוחרת]]. If the latest saved version is the version you are talking about, you may log off, shut down your computer, go to Tahiti for a week, and when you come back, there should be no problem ([[WP:CSD#G13|generally for six months]]). Of course, the reason the draft was userfied for you upon request was because of a copyright issue. One of the "out of the ordinary matters" is where a draft contains a copyright violation. That will result in deletion immediately once discovered "even" as a draft. But since that has already been addressed for this page, and assuming you added back no copied content, yes, go to bed!--[[User:Fuhghettaboutit|Fuhghettaboutit]] ([[User talk:Fuhghettaboutit|talk]]) 22:51, 27 January 2017 (UTC) |
|||
Thanks. Tahiti too far, almost as far as my orphaned bed of rose thorns... |
|||
Grateful,[[User:שוחרת|שוחרת]] ([[User talk:שוחרת|talk]]) 00:22, 28 January 2017 (UTC)שוחרת |
|||
[[User:שוחרת|שוחרת]] ([[User talk:שוחרת|talk]]) 00:22, 28 January 2017 (UTC) |
|||
== Hello -- I would like to start helping, but I'm having trouble finding what to do! == |
|||
==Not Wiki Specific: Proper nouns as adjectives== |
|||
So, I made [https://en.wikipedia.org/enwiki/w/index.php?title=Rusev_%28wrestler%29&type=revision&diff=762270168&oldid=762261419 an edit] where a previous anonymous editor changed the capitalization of a wrestling move called a "[[Rikishi Bomb|Samoan Drop]]" so that it read "samoan drop" instead of "Samoan drop." I restored the original capitalization (with good faith, of course), but now I'm wondering. |
|||
I see lots of support for ultra basics, but I know what the concept is, and how to edit, and how to make italics and hyperlinks, and that I should use a neutral voice, etc. I am trying to find some guidance on WHAT to contribute. I found the Typo Team (or at least, [[Wikipedia:Typo Team/moss/D|this typo team]]), but I haven't found guidance on interacting with it. (Do I delete entries if I resolve them? Yesterday I found many entries to check, but today none of the articles seem to HAVE the potential typo that was listed, or even a fixed version). I have found [[mediawikiwiki:Growth/FAQ#|this Growth page]], but can't get the features working. For example, it says to enable the Help panel in the Editing tab, but I don't see such a thing in the Editing tab. I also can't find "Display newcomer homepage" in my user preferences. Similar with Suggested Edits -- how can I "use <code>Special:NewcomerTasksInfo</code>"? Etc., etc. I must be missing some key piece of advice -- where can I figure out how to get things rolling? [[User:Skmccormick|SKM]] ([[User talk:Skmccormick|talk]]) 05:08, 31 December 2024 (UTC) |
|||
Obviously, "Samoan" is a proper noun, but when used to describe something else not directly relating to the original country, does it become non-proper? Kind of like "North Carolina" vs. "I'm heading north from here." I'm hard pressed for a more specific example, but I would assume something like a generic culinary dish named after a region like "New Mexico tacos" would preserve its capitalization instead of becoming "new mexico tacos." |
|||
:Hello @[[User:Skmccormick|Skmccormick]]. [[WP:volunteer|As a volunteer project]], you're free to do whatever you wanna do best. Want to copyedit a bunch of articles and bring our grammar up to shape? Join the [[WP:goce|Guild of Copy Editors]] and go wild. Wanna fight vandalism? Go patrol [[Special:RecentChanges]] and stop those dang vandals! Wanna go help out that typo team? Go right ahead. It's your choice. [[User:Tarlby|<span style="color:cyan;font-family:Comic Sans MS;">''Tarl''</span><span style="color:orange;font-family:Comic Sans MS;">''by''</span>]] <sup>([[User talk:Tarlby|''t'']]) ([[Special:Contributions/Tarlby|''c'']])</sup> 05:26, 31 December 2024 (UTC) |
|||
Any guidance? |
|||
::@[[User:Skmccormick|Skmccormick]] |
|||
::*You should find display user homepage at the bottom of [[Special:Preferences]], "User profile" tab. |
|||
::*If I wanted to interact with the typo team, my first place to try would be [[Wikipedia talk:Typo Team]]. |
|||
::*Have you found [[WP:TASKS]] and [[WP:REQUEST]]? [[User:Gråbergs Gråa Sång|Gråbergs Gråa Sång]] ([[User talk:Gråbergs Gråa Sång|talk]]) 07:28, 31 December 2024 (UTC) |
|||
:::Thank you! These links are helpful. I found "display user homepage" under User Profile. So maybe a silly question, but: how do I find this user homepage? I don't really go to Wikipedia generically, I usually jump straight to an article. Tasks and Requests seem like what I'm looking for. [[User:Skmccormick|SKM]] ([[User talk:Skmccormick|talk]]) 01:47, 1 January 2025 (UTC) |
|||
::::The link should be right at the top of every page when you're logged in, in the same place/menu as the link to your userpage. -- [[User:Asilvering|asilvering]] ([[User talk:Asilvering|talk]]) 02:05, 1 January 2025 (UTC) |
|||
:@[[User:Skmccormick|Skmccormick]], do you have any particular topics that you'd like to edit about? One way to find a lot of articles that need help is to go through our various maintenance backlogs (Gråbergs Gråa Sång has already linked you to [[WP:TASKS]]). Some people are content to plug away at a particular backlog chronologically, but if you prefer to edit on things you're generally interested in, it's helpful to filter these by wikiproject. Alternatively, do you have any particular skills or outside knowledge that might be helpful here? There's always demand for multilingual editors, the copyright folks are always backlogged, etc. -- [[User:Asilvering|asilvering]] ([[User talk:Asilvering|talk]]) 10:16, 31 December 2024 (UTC) |
|||
::Thank you. I would prefer to get into the groove with smaller contributions before I start throwing any weight around. Sadly, I cannot offer multilingual help; I am American. I'm mostly having trouble navigating the various pages and internal tools like TASKS or RecentChanges (anything labeled "Special:" is still new to me). [[User:Skmccormick|SKM]] ([[User talk:Skmccormick|talk]]) 01:50, 1 January 2025 (UTC) |
|||
:::I'm a fan of [[:Category:Wikipedia introduction cleanup]] as a newbie task - not ''small'', exactly, but hey, no time like the present to learn to [[WP:BEBOLD]]. You don't (or shouldn't) need to do any research to fix these articles - most of them are here because they are tagged with "lead too short". Find one of those, read the article, then rewrite the lead so it summarizes it accurately. Then remove the tag. All this requires is good English literacy, and since the lead is what most people read and what is used in the google knowledge box etc, it's a high-impact change that requires very little wiki-knowledge. -- [[User:Asilvering|asilvering]] ([[User talk:Asilvering|talk]]) 02:16, 1 January 2025 (UTC) |
|||
== Different images for Light/Dark mode == |
|||
Thanks, [[User:KNHaw|<font face="comic sans ms"><font color="SeaGreen"><i><b>KNHaw</b></i></font></font>]] <font color="SeaGreen">[[User talk:KNHaw|<sup>(talk)</sup>]]</font> 19:26, 27 January 2017 (UTC) |
|||
Is there any way to tell Wikipedia to display different images / media for Light vs Dark mode users? Due to transparency, some SVG and PNG images have bad contrast when viewed in Dark mode. [[User:CrushedAsian255|CrushedAsian255]] ([[User talk:CrushedAsian255|talk]]) 06:04, 31 December 2024 (UTC) |
|||
:{{reply|KNHaw}} Since "Samoan" is always a proper noun, it should always be capitalized. "North" is not always a proper noun, so it's case can vary. hope that helps! ···[[User:Nihonjoe|<font color="darkgreen">日本穣</font>]] · <small>[[Special:Contributions/Nihonjoe|<font color="blue">投稿</font>]] · [[User talk:Nihonjoe|Talk to Nihonjoe]] · [[WP:JA|<font color="maroon">Join WP Japan</font>]]!</small> 19:40, 27 January 2017 (UTC) |
|||
:@[[User:CrushedAsian255|CrushedAsian255]] This question is likely to have a better audience at [[WP:VPT]]. That board has a more technically oriented team than here. 🇺🇦 [[User:Timtrent|<span style="color:#800">Fiddle</span><sup><small>Timtrent</small></sup>]] [[User talk:Timtrent|<span style="color:#070">Faddle</span><sup><small>Talk to me</small></sup>]] 🇺🇦 09:26, 31 December 2024 (UTC) |
|||
:Fine by me! Thanks. [[User:KNHaw|<font face="comic sans ms"><font color="SeaGreen"><i><b>KNHaw</b></i></font></font>]] <font color="SeaGreen">[[User talk:KNHaw|<sup>(talk)</sup>]]</font> 20:03, 27 January 2017 (UTC) |
|||
::<span style="font-size:65%;">@[[:User:KNHaw|KNHaw]], [[:User:Nihonjoe|Nihonjoe]]:</span> Not always. We generally do not capitalize nouns/noun phrases that incorporate place names where the name has become highly generified as everyday English vocabulary: french fries, french toast, french doors, english (sidespin in billiards), plaster of paris, etc. But I don't think this has happened for Samoan drop. The issue occasionally arises at [[WP:RM|requested moves]] and the answer is always the same: check for usage across reliable sources—and don't use a web search but a search that tends to concentrate [[WP:IRS|reliable sources]], such as using Google Books—where [https://www.google.com/search?q=%22samoan+drop%22&biw=989&bih=889&noj=1&tbm=bks&source=lnms&sa=X&ved=0ahUKEwjG0dPysOPRAhUDw4MKHS1SAy8Q_AUIDSgG&dpr=1 here], the results strongly support using the uppercase ess. Best regards--[[User:Fuhghettaboutit|Fuhghettaboutit]] ([[User talk:Fuhghettaboutit|talk]]) 22:37, 27 January 2017 (UTC) |
|||
:::{{reply|Fuhghettaboutit}} So, in essence, "Samoan" is ''currently'' always a proper noun. {{smiley|wink}} ···[[User:Nihonjoe|<font color="darkgreen">日本穣</font>]] · <small>[[Special:Contributions/Nihonjoe|<font color="blue">投稿</font>]] · [[User talk:Nihonjoe|Talk to Nihonjoe]] · [[WP:JA|<font color="maroon">Join WP Japan</font>]]!</small> 22:50, 27 January 2017 (UTC) |
|||
== Wider vs specific consensus == |
|||
:Thanks again. I will keep my edit as capitalized. [[User:KNHaw|<font face="comic sans ms"><font color="SeaGreen"><i><b>KNHaw</b></i></font></font>]] <font color="SeaGreen">[[User talk:KNHaw|<sup>(talk)</sup>]]</font> 22:50, 27 January 2017 (UTC) |
|||
::::Yes, not "generified/genericized" yet.--[[User:Fuhghettaboutit|Fuhghettaboutit]] ([[User talk:Fuhghettaboutit|talk]]) 22:58, 27 January 2017 (UTC) |
|||
Can a case-specific consensus triumph a wider topic consensus already established? Not that consensus over wider topic is changed but maybe because that case is viewed from a different perspective. [[User:ExclusiveEditor|<span style="background:linear-gradient(to right,#FF7043,#FFCC80);color:#fff;padding:4px 8px;border-top-left-radius:6px;border-bottom-left-radius:6px;font-size:12px;font-weight:bold;box-shadow:0 2px 4px rgba(0,0,0,.2);text-transform:uppercase;">Exclusive</span><span style="background:linear-gradient(to right,#263238,#37474F);color:#fff;padding:4px 8px;border-top-right-radius:6px;border-bottom-right-radius:6px;font-size:12px;font-weight:bold;box-shadow:0 2px 4px rgba(0,0,0,.2);text-transform:uppercase;margin-left:-1px;">Editor</span>]] [[User talk:ExclusiveEditor|<sub style="font-size:11px;color:#757575;font-style:italic;letter-spacing:.5px;">🔔 Ping Me!</sub>]] 07:33, 31 December 2024 (UTC) |
|||
==How to handle reliable sources only available w/ UN/PW or by PDF== |
|||
For an entry I'm writing on an entrepreneur, I'd like to use a profile that Businessweek did of him. Unfortunately, it is not available online except as a PDF via Ebsco which I access through my library (NY Public). |
|||
:@[[User:ExclusiveEditor|ExclusiveEditor]] I doubt it. Were that to be the case we would not have consistency of operation. This question is likely to have a better audience at [[WP:VPP]]. That board has a more policy oriented team than here 🇺🇦 [[User:Timtrent|<span style="color:#800">Fiddle</span><sup><small>Timtrent</small></sup>]] [[User talk:Timtrent|<span style="color:#070">Faddle</span><sup><small>Talk to me</small></sup>]] 🇺🇦 09:28, 31 December 2024 (UTC) |
|||
The PDFs are scans of the hard copy magazine, so you see the magazine name and date in the footer. I can include link to it in Ebsco, but the url comes up as a "proxy" which is some kind of red flag to the editors, and probably won't work because you need a library card with a UN/PW. |
|||
:Hello, @[[User:ExclusiveEditor|ExclusiveEditor]], and welcome to the Teahouse. While I agree with FiddleFaddle's advice, I would also remark that general questions like yours are very frustrating for people who attempt to answer questions here. If you explain the specific issue you want guidance on, you are much more likely to get a useful answer (and also be less likely to be suspected of [[WP:wikilawyering|wikilawyering]]). |
|||
:I am aware of the possible irony in my answer, given your question. [[User:ColinFine|ColinFine]] ([[User talk:ColinFine|talk]]) 10:43, 31 December 2024 (UTC) |
|||
::Thanks for replying! Being a half-host on Teahouse myself I get you. I was inspired to ask this question by the discussion happening [[Wikipedia:In_the_news/Candidates#(Posted_blurb)_RD/Blurb_Jimmy_Carter|here]]. The latter part of the discussion specifically focuses on if 'Nobel prize' should be added or not in Jimmy Carter's death blurb on Main page's ITN section. It was initially proposed by the nominator and many supported it (albeit not mentioning specifically the Nobel prize) and it got posted without the mention of the prize. However later there was some more scrambling, this time with more regard to the Nobel prize and so it currently updated to include the prize in the blurb. The opposers are generally arguing that it is editorializing and other things. I may not be very good in summarizing discussions, so I left it in the question. --{{User:ExclusiveEditor/Signature}} 10:58, 31 December 2024 (UTC) |
|||
== How to upload a >100 MB file to Wikipedia? == |
|||
I have downloaded the PDF's and could archive them somewhere (archive.org, say), but I don't want to go the trouble only to find out that against the rules. |
|||
I've uploaded plenty of files to Commons before, but I've only uploaded a few (non-free) files to WP. I would like to upload a short film that will become public domain in the US at the start of 2025 (won't be PD in its country of origin for a few years, so no uploading it to commons), but the file is over 100 MB, the maximum file size listed on the [[Special:Upload|upload page]]. I don't want to compress it any more than it already is, so how to I get around this? I've seen several films large than 100 MB on WP already ([[:File:The Adventures of Prince Achmed (1926).webm|1]], [[:File:The Cabinet of Dr. Caligari (1920).webm|2]]), so it must be possible. |
|||
I'm sure this problem has come up before and am hoping someone can direct me to the right info -- or give it to me direct. |
|||
[[User: |
Any help with this is greatly appreciated. Thanks. — [[User:Toast for Teddy|Toast for Teddy]] ([[User talk:Toast for Teddy|talk]]) 07:48, 31 December 2024 (UTC) |
||
:Hi {{u|Sam Perkins}}. Sources do not have to be available online. See [[Wikipedia:Offline sources]]. [[User:StarryGrandma|StarryGrandma]] ([[User talk:StarryGrandma|talk]]) 19:13, 27 January 2017 (UTC) |
|||
:@[[User:Toast for Teddy|Toast for Teddy]] This question is likely to have a better audience at [[WP:VPT]]. That board has a more technically oriented team than here 🇺🇦 [[User:Timtrent|<span style="color:#800">Fiddle</span><sup><small>Timtrent</small></sup>]] [[User talk:Timtrent|<span style="color:#070">Faddle</span><sup><small>Talk to me</small></sup>]] 🇺🇦 09:26, 31 December 2024 (UTC) |
|||
:{{ec}} Hey [[User:Sam Perkins|Sam Perkins]]. [[Template:Cite]] includes a parameter <nowiki>|url-access=subscription</nowiki> which indicates to other readers that a paid subscription is required to access the material. Otherwise, sources behind paid subscriptions are allowed under Wikipedia's verifiability standards, even if they're less than ideal. [[User:Timothyjosephwood|<span style="color:#a56d3f;font-family:Impact;">Timothy</span><span style="color:#6f3800;font-family:Impact;">Joseph</span><span style="color:#422501;font-family:Impact;">Wood</span>]] 19:14, 27 January 2017 (UTC) |
|||
::@[[User:Toast for Teddy|Toast for Teddy]] That first file you linked was uploaded with the aid of a userscript discussed at [[:Commons:User talk:Rillke/bigChunkedUpload.js]]. I don't pretend to understand the details, but you may ;-) [[User:Michael D. Turnbull|Mike Turnbull]] ([[User talk:Michael D. Turnbull|talk]]) 14:27, 31 December 2024 (UTC) |
|||
:Even if you're uploading "locally", [[commons:Commons:Upload tools|Commons:Upload tools]] is likely helpful. All the projects use the same [[MediaWiki]] software, all that needs adjustment is the destination for the upload. --[[User:Slowking Man|Slowking Man]] ([[User talk:Slowking Man|talk]]) 20:54, 31 December 2024 (UTC) |
|||
== Editing a sandbox, references number is doubling == |
|||
WOW! That was fast. Thanks for such a ''speedy'' reply. Checking out your solutions now. |
|||
Hi there! I was editing my [https://en.wikipedia.org/enwiki/w/index.php?title=User:Andrea_Biographer/sandbox&action=submit#Gridspertise sandbox] and for some reasons all the different references I am adding are doubling the number of them and the previous ones are not disappeared. I was following your suggestions to add COI edit to my text, eliminating the internal sources and the bold words. Can you help me or it is just a matter of viewing and once I publish the sand box they will all disappear? [[User:Andrea Biographer|Andrea Biographer]] ([[User talk:Andrea Biographer|talk]]) 09:10, 31 December 2024 (UTC) |
|||
[[User:Sam Perkins|Sam Perkins]] ([[User talk:Sam Perkins|talk]]) 19:26, 27 January 2017 (UTC) |
|||
:And for completeness, {{U|Sam Perkins}}, I'll point out that uploading a PDF of a reference is hardly ever useful. Often it would be a copyright violation anyway, but even if it is freely licensed, it will hardly ever meet the requirement of [[WP:RS|reliability]]: even if it was downloaded from a reliable source, it could have been altered. --[[User:ColinFine|ColinFine]] ([[User talk:ColinFine|talk]]) 22:47, 27 January 2017 (UTC) |
|||
:Hi @[[User:Andrea Biographer|Andrea Biographer]]: it seems that with [https://en.wikipedia.org/enwiki/w/index.php?title=User:Andrea_Biographer/sandbox&diff=prev&oldid=1266218280 this edit] you've duplicated the contents by pasting an earlier edit into the page, thus embedding a copy of the entire page within the page (if that makes sense). You should undo your most recent edits up to and including that one. Or if you'd like me to do it, let me know. -- [[User:DoubleGrazing|DoubleGrazing]] ([[User talk:DoubleGrazing|talk]]) 09:16, 31 December 2024 (UTC) |
|||
== Any final checks recommended? == |
|||
::Hi @[[User:DoubleGrazing|DoubleGrazing]], thanks for your prompt answer! I think you made sense, could you check and fix this for me? I don't want to commit any further mistakes in the editing process or in the COI different templates, thanks in advance! Andrea [[User:Andrea Biographer|Andrea Biographer]] ([[User talk:Andrea Biographer|talk]]) 09:31, 31 December 2024 (UTC) |
|||
I'm working for some time now on an article on an international technical conference. I do my best to be as precise in terms of facts and neutral in tone as possible and to give ample references for my statements (mainly to the conference's web site). ProveIt shows no errors. Is there anything you would recommend to me before I shall submit my article? [[User:WolfgangSchi|WolfgangSchi]] ([[User talk:WolfgangSchi|talk]]) 17:55, 27 January 2017 (UTC) |
|||
:::User:Theroadislong has already sorted this out. Also, your draft is now located at [[Draft:Gridspertise]]. -- [[User:DoubleGrazing|DoubleGrazing]] ([[User talk:DoubleGrazing|talk]]) 10:30, 31 December 2024 (UTC) |
|||
:Hi {{u|WolfgangSchi}} and welcome to the Teahouse. Please read [[Wikipedia:Notability]] before going any further. Wikipedia requires that subjects already be well-known and written about in reliable sources independent of the subject before having an article here. None of your references show that about the conference. Look for independent sources about the conference itself or the sponsoring organization. [[User:StarryGrandma|StarryGrandma]] ([[User talk:StarryGrandma|talk]]) 18:07, 27 January 2017 (UTC) |
|||
::One reference does talk about the course in depth, but are there others? [[User:StarryGrandma|StarryGrandma]] ([[User talk:StarryGrandma|talk]]) 18:13, 27 January 2017 (UTC) |
|||
== how to give feedback == |
|||
==Should I put in an EPPO code into a Taxonomy box?== |
|||
Hello. Just wandering if I should put the EPPO code for the genus Kalaharituber on this page: https://en.wikipedia.org/wiki/Kalaharituber. And, how do I link articles on the same topic but in different languages? |
|||
[[User:AWearerOfScarves|AWearerOfScarves]] ([[User talk:AWearerOfScarves|talk]]) 17:39, 27 January 2017 (UTC) |
|||
i had a question on why an article was written and sent it to info@ wikipedia. they told me that I needed to engage in the "talk" feature and ask the editors, since wikipedia is only a platform. I did that and not only was the answer not given I was ridiculed, because I am not actually sure why. I am not interested in editing wikipedia, I am only interested in engaging with the editors, to understand inconsistencies. How exactly would I do that, if I should not be using the talk feature. |
|||
: Links to corresponding articles in other languages are made through Wikidata. See either the "Wikidata item" link on the left-hand toolbar, or the "Edit links" item at the foot of the "Languages" section at the bottom of that left-hand toolbar. There are two other languages there already. --[[User:David Biddulph|David Biddulph]] ([[User talk:David Biddulph|talk]]) 19:38, 27 January 2017 (UTC) |
|||
thanks [[User:Mommer264|Mommer264]] ([[User talk:Mommer264|talk]]) 12:34, 31 December 2024 (UTC) |
|||
:Hello. Discussion about an article usually takes place on its talk page. Discussion with an editor directly can occur on their user talk page. |
|||
==Help with article== |
|||
:Note that your only other edit was about the Israeli-Arab/Palestinian conflict, which is [[WP:CTOP|a topic area with special rules]] that I will notify you of on your user talk page. One of those is that you must be an experienced user in order to make edits to any type of page about it. Your account must be 30 days old with 500 edits. [[User:331dot|331dot]] ([[User talk:331dot|talk]]) 12:53, 31 December 2024 (UTC) |
|||
Hi, fellow Wikipedians. Earlier this month I created an article, [[Elisabeth Brichet case]]. I have been working on it and have managed to add a lot to it, but I'm a bit stuck. I think the article has potential to reach at least B-class, but there's a lot of info I still haven't added. Most of the references available are from [[Le Soir]] ([http://archives.lesoir.be/?action=search&firstHit=0&queryand=Brichet&queryor=&querynot=&nomau=&prenau=&when=-2&when=-1&begDay=01&begMonth=01&begYear=1989&endDay=26&endMonth=01&endYear=2017&sort=datedesc&by=10&rub=TOUT around 50 pages of news articles]; not all of them are usable, though), a French-language newspaper. I'm not fluent in French, and so there's an awful lot to read through and translate, and there are some parts I can't translate right. I've read through and translated many of them, and cited the ones I could use from those in the Wikipedia article. It would be helpful if others could help expand the article to include the missing info. It would be ideal if any of you know French, but don't worry if you can't; there are also links to news articles on the talk page, including a number of English ones, that you can use. I think this article has potential, but I can't do it all on my own. Any edits made to the article would be appreciated. Thank you in advance. [[User:Linguist111|<span style="color:lightpink">Linguist</span>]][[User talk:Linguist111|<sup><span style="color:green">talk</span></sup>]]|[[Special:Contributions/Linguist111|<span style="color:purple"><small>contribs</small></span>]] 17:06, 27 January 2017 (UTC) |
|||
::Also answering your query: :You asked a question that is now at [[Talk:State of Palestine/Archive 21]]. Talk pages of articles are not for 'open' questions. Instead, the proper method is to propose a specific change of text, as in replace A with B, and include a reference to support your proposal. Given that [[State of Palestine]] is a very controversial article, editors who participate there - at both article and talk - can be short on tact. [[User:David notMD|David notMD]] ([[User talk:David notMD|talk]]) 12:54, 31 December 2024 (UTC) |
|||
:{{reply|Linguist111}} You may try asking for help at [[WT:FRANCE|WikiProject France]]. It's likely there are fluent speakers there who can help. ···[[User:Nihonjoe|<font color="darkgreen">日本穣</font>]] · <small>[[Special:Contributions/Nihonjoe|<font color="blue">投稿</font>]] · [[User talk:Nihonjoe|Talk to Nihonjoe]] · [[WP:JA|<font color="maroon">Join WP Japan</font>]]!</small> 19:42, 27 January 2017 (UTC) |
|||
:{{tpq|I am not interested in editing wikipedia}} If you're not interested in contributing to the project, then you probably ought to go elsewhere. [[WP:PURPOSE|The purpose of Wikipedia]] is to produce an encyclopedia; notably, [[WP:TP|talk pages]] are for constructively discussing the project and its content, not general [[Internet forums]] for discussion. There are many many discussion forums elsewhere on the Internet, and Wikipedia can even [[:Category:Internet forums|help direct you to some of them]]. If you do wish to contribute to the encyclopedia, take a look at [[WP:Welcome]]. Thank you and I hope you have a good day. --[[User:Slowking Man|Slowking Man]] ([[User talk:Slowking Man|talk]]) 21:03, 31 December 2024 (UTC) |
|||
::{{ping|Nihonjoe}} Thank you! [[User:Linguist111|<span style="color:lightpink">Linguist</span>]][[User talk:Linguist111|<sup><span style="color:green">talk</span></sup>]]|[[Special:Contributions/Linguist111|<span style="color:purple"><small>contribs</small></span>]] 20:22, 27 January 2017 (UTC) |
|||
== VisualEditor == |
|||
==how to write well?== |
|||
How will I write a paragraph more structurally?I mean how I move from one paragraph to another by keeping a nice distance.how will I add picture[[User:Baruaanik|Baruaanik]] ([[User talk:Baruaanik|talk]]) 16:56, 27 January 2017 (UTC) |
|||
I've just changed my Preferences > Editing |
|||
:Hi {{u|Baruaanik}}, separate paragraphs with just one blank line. Read the [[WP:PIC|Picture tutorial]] and if you still have questions please come back here. [[User:Dodger67|Roger (Dodger67)]] ([[User talk:Dodger67|talk]]) 17:51, 27 January 2017 (UTC) |
|||
Turned on "Enable the visual editor" <small>(which, I think, it's on by default in Wikipedia in Italian language)</small> |
|||
==Hos to create an inset== |
|||
How to I create an inset such as the one you see on the right of this page: |
|||
But i can not find out how to edit with VisualEditor. |
|||
https://en.wikipedia.org/wiki/Alexander_Calder |
|||
I've looked at [[Help:VisualEditor]] and it says ([[Help:VisualEditor#Opening VisualEditor]] to click on the "Edit" tab (in the picture I can see a "drop down" menu to choose the editor). |
|||
[[User:Acelentano2016|Acelentano2016]] ([[User talk:Acelentano2016|talk]]) 15:35, 27 January 2017 (UTC) |
|||
But I've not this choice. My tab is named "Edit source". [[User:Centrodiurnomilano|Centrodiurnomilano]] ([[User talk:Centrodiurnomilano|talk]]) 14:47, 31 December 2024 (UTC) |
|||
:Hi Acelentano2016, and welcome. I'm guessing you mean the [[WP:INFOBOX|infobox]] - the code to create these varies depending on the subject matter. You can find the code and instructions for [[Francis Celentano|your article]] at [[Template:Infobox artist]], but let us know here if you need a hand. [[User:Yunshui|Yunshui]] [[User talk:Yunshui|<sup style="font-size:90%">雲</sup>]][[Special:Contributions/Yunshui|<sub style="font-size:90%">水</sub>]] 15:53, 27 January 2017 (UTC) |
|||
:I've found out, reading [[Help:VisualEditor#First step: enabling VE]] (which seems a bit outdated, by the way). |
|||
==Improve Content for a page that is labelled as deletion== |
|||
:After I've changed my Preferences > Editing > "Enable the visual editor" = ON |
|||
https://en.wikipedia.org/wiki/Anjusha_Chaughule |
|||
:and the press "save" button at the bottom. |
|||
:Only then, a "drop down menu" show up in the same section: "Editing mode", which is setted by default "Remember the last editor". I've changed it to "Show both editor tabs". |
|||
:It's not easy to find out. [[User:Centrodiurnomilano|Centrodiurnomilano]] ([[User talk:Centrodiurnomilano|talk]]) 15:04, 31 December 2024 (UTC) |
|||
::By the way, I've looked at the settings on Wikipedia in Italian language and "Enable the visual editor" it's enough there (there is no "Editing mode" option at all) [[User:Centrodiurnomilano|Centrodiurnomilano]] ([[User talk:Centrodiurnomilano|talk]]) 15:08, 31 December 2024 (UTC) |
|||
:Italian Wikipedia and English Wikipedia run on the same system, MediaWiki, but are two different Wikipedias. So if this is a question about itwiki, then you should go to a help board on itwiki. If this a question about enwiki, you should be able to switch between the source editor and visual editor by pressing the pencil button next to the preview button in the source editor. <span style="font-family:Arial;background-color:#fff;border:2px dashed#69c73e">[[User:Cowboygilbert|<span style="color:#3f6b39">'''Cowboygilbert'''</span>]] - [[User talk:Cowboygilbert|<span style="color:#d12667"> (talk) ♥</span>]]</span> 16:07, 31 December 2024 (UTC) |
|||
::But, @[[User:Centrodiurnomilano|Centrodiurnomilano]], do note that you can only use the VisualEditor is some namespaces but not all. You can in mainspace, userspace, draftspace but not in wikispace which is the space with the Wikipedia: prefix. <span style="font-family:Arial;background-color:#fff;border:2px dashed#69c73e">[[User:Cowboygilbert|<span style="color:#3f6b39">'''Cowboygilbert'''</span>]] - [[User talk:Cowboygilbert|<span style="color:#d12667"> (talk) ♥</span>]]</span> 16:09, 31 December 2024 (UTC) |
|||
:::Thank you, @[[User:Cowboygilbert|Cowboygilbert]] . I've tried in main space and the in user space. [[User:Centrodiurnomilano|Centrodiurnomilano]] ([[User talk:Centrodiurnomilano|talk]]) 16:29, 31 December 2024 (UTC) |
|||
::::Were you able to fix it using the instructions that I put, {{tq|you should be able to switch between the source editor and visual editor by pressing the pencil button next to the preview button in the source editor.}}? <span style="font-family:Arial;background-color:#fff;border:2px dashed#69c73e">[[User:Cowboygilbert|<span style="color:#3f6b39">'''Cowboygilbert'''</span>]] - [[User talk:Cowboygilbert|<span style="color:#d12667"> (talk) ♥</span>]]</span> 16:30, 31 December 2024 (UTC) |
|||
== "No Such Number tone" / "crybaby tone" (type of intercept error) == |
|||
Hello team, |
|||
Hello everyone. This is one of my first times writing a brand-new article, and I'm not sure if this is a notable enough topic for its own page or a subsection in the intercept message. |
|||
The above link created by me has got a tag for deletion. I would like to improve the content quality so as to better meet the wikipedia norms and regulations. Can you please tell me as to what are the errors on the above page so that I can help improve the article and avoid its deletion. |
|||
This technically isn't a message, but before the intercept message was used, a different type of tone could also be returned instead of connecting to an operator. This is known under many different names, such as the "no such number tone" or even the "crybaby tone." This tone would be returned if a caller attempts to dial a number that can't possibly exist according to the numbering plan. The call would not be allowed to go through because some of these numbers could be reserved for private use. The tone itself is continuous, sweeping from 200-400Hz back to 200Hz again over the course of one second. |
|||
[[User:Acpp555|Acpp555]] ([[User talk:Acpp555|talk]]) 13:23, 27 January 2017 (UTC) |
|||
The issue is that it's almost exclusively used in North America, if not, completely exclusively. It was introduced by the Bell System in 1941, but I've also heard of it in use by the 3CX Asterisk system. However, my intent is to preserve information about this tone, as next to no information seems to exist about it. An excerpt from the Bell Labs Record describes it here. It also seems to be exclusive to crossbar systems. |
|||
:Hey [[User:Acpp555|Acpp555]]. One option available is to request that the article be moved to a draft, where it will not be easily accessible to the general public, but will not be deleted, and so you can have additional time to work on it and seek advice from other editors. Since this is your first article, it would probably be helpful to consider reading through [[WP:YFA|our tutorial]], and (if you request it be made into a draft) consider submitting through our [[WP:AFC|Articles for Creation]] board, where it can be reviewed by experienced volunteers prior to publishing, and who can hopefully offer helpful advice. |
|||
[https://www.telephonecollectors.info/index.php/browse/document-repository/catalogs-manuals/bell-system-we/bell-labs-record/4432-41apr-blr-no-such-number-tone-for-dial-systems] Bell Labs record |
|||
:Otherwise, the deletion discussion will progress normally, usually for about a week or so, and if the community decides to delete the article, you will have to ask for an administrator to restore at [[WP:REFUND]]. [[User:Timothyjosephwood|<span style="color:#a56d3f;font-family:Impact;">Timothy</span><span style="color:#6f3800;font-family:Impact;">Joseph</span><span style="color:#422501;font-family:Impact;">Wood</span>]] 13:30, 27 January 2017 (UTC) |
|||
::Hi, {{u|Acpp555}}. I've made a request of the editor that nominated the article for deletion to move it to draft for you and withdraw the nomination. This is possible because no one else has entered the deletion discussion. It's 100% up to him, but if he agrees it will save you some wait. [[User:John from Idegon|John from Idegon]] ([[User talk:John from Idegon|talk]]) 01:58, 28 January 2017 (UTC) |
|||
:::{{u|Acpp555}}, You're article is now at [[Draft:Anjusha Chaughule]]. I'd suggest you submit it via [[WP:AFC|Articles for Creation]] when you think it's ready so you can get a second, more experienced opinion before it is put back in the encyclopedia and at risk for deletion again. Best of luck. [[User:John from Idegon|John from Idegon]] ([[User talk:John from Idegon|talk]]) 17:37, 28 January 2017 (UTC) |
|||
[https://www.nature.com/articles/148193b0] Article about this record |
|||
==Please improve my draft!== |
|||
Hey, if any editor is vacant can you please help me wit improving the styling of my draft?? Please repair its structure, template. I know the content. I just want a good structure. I have basic knowledge. Please help in improving my articles here :- |
|||
[https://www.youtube.com/watch?v=Ipuruc9Ur_4] Sample of this sound |
|||
https://en.wikipedia.org/wiki/Draft:Clash-a-Rama |
|||
[[User: |
What do you think of this? [[User:ZetaformGames|ZetaformGames]] ([[User talk:ZetaformGames|talk]]) 19:01, 31 December 2024 (UTC) |
||
:Hi @[[User:ZetaformGames|ZetaformGames]], welcome to the teahouse and welcome to Wikipedia! I think that this could be a good idea as a subsection in the article for [[Intercept message]] or some other related article. I don't think that there is enough for a full article with the sources you've posted but could definitely be an interesting paragraph! [[User:Justiyaya|'''<span style="color:#1d556d">Just</span>''']][[Special:Contributions/Justiyaya|'''<span style="color:#000000">i</span>''']][[User talk:Justiyaya#top|'''<span style="color:#6d351d">yaya</span>''']] 07:12, 1 January 2025 (UTC) |
|||
:Hi {{u|HighnessAtharva}}, I have taken a look but I can't figure out why the episode table is being rendered ''after'' the references. While we wait for someone who knows how to fix it, you need to look for better sources to reference. The subject's own website, Youtube and a blog are no good for proving notability. I'm not sure of the reliability of toucharcade.com. Look for quality sources such as reputable websites that review games. You could also as [[WP:WikiProject Video games]], as that's where the topic specialists hang out. [[User:Dodger67|Roger (Dodger67)]] ([[User talk:Dodger67|talk]]) 12:39, 27 January 2017 (UTC) |
|||
::Alright, thank you! And thanks for the welcome. I made this account a while ago, but haven't felt confident enough in my editing skills until now to contribute. [[User:ZetaformGames|ZetaformGames]] ([[User talk:ZetaformGames|talk]]) 17:46, 1 January 2025 (UTC) |
|||
== Maryam Mirzakhani == |
|||
==Is their a prebuilt template that I can use== |
|||
Is their a template with a superset of attributes that I can fill in and edit to create a WIKI page more efficiently? [[User:Acelentano2016|Acelentano2016]] ([[User talk:Acelentano2016|talk]]) 02:56, 27 January 2017 (UTC) |
|||
:Welcome to the Teahouse, {{u|Acelentano2016}}. I am unaware of any such template. An acceptable Wikipedia article will be originally written prose, which does not really lend itself to formulas or recipes. I suggest that you read [[WP:YFA|Your first article]], and take a look at [[WP:GA|Good articles]] and [[WP:FA|Featured articles]] about topics similar to yours. Those will give you a good idea about how your planned article should be written and referenced. [[User:Cullen328|<b style="color:#070">Cullen</b><sup style="color:#707">328</sup>]] [[User talk:Cullen328|<span style="color:#00F">''Let's discuss it''</span>]] 04:06, 27 January 2017 (UTC) |
|||
Can someone fix the error in the [[Maryam Mirzakhani]] article? |
|||
==I have come across an old photo of the silent film actress Peggy Hyland. == |
|||
Photo of Peggy Hyland; has her name written on back and that of Campbell Studios, 5th ave embossed on the picture. Would this picture be of interest for inclusion on her bio page [[Special:Contributions/2601:346:404:4375:E83D:1827:B773:4AE2|2601:346:404:4375:E83D:1827:B773:4AE2]] ([[User talk:2601:346:404:4375:E83D:1827:B773:4AE2|talk]]) 21:07, 26 January 2017 (UTC) |
|||
:Hi there, and welcome to the Teahouse. There are a few questions that need to be answered before we know it's OK to add an image to a Wikipedia article (even if the photo looks good and would be useful): |
|||
:*Who is the owner of the photo? |
|||
:*Is the photo free to reuse for any purpose? Usually there is a license that permits this, and if there is none or it's not clear, it's usually assumed to be owned by the photographer or copyright holder and cannot be used. There are some exceptions to this based on the age of the photo, as it may have fallen out of copyright. |
|||
:If you can provide a link to the photo (if it's online) or provide answers to these questions, we can help you out. Thanks! [[User:I JethroBT|<b style="font-family:Candara;color:green">I JethroBT</b>]][[User talk:I JethroBT| <sup>drop me a line</sup>]] 22:51, 26 January 2017 (UTC) |
|||
::Hello, anonymous editor. [[Peggy Hyland]]'s film career took place so long ago that it is highly likely that the copyright on your photo has expired, ''if'' it was published even once back in the days when she was an actress. Please note that our article already has several photos of her, even though they are not very good photos. Therefore, we cannot use the photo under a claim of fair use, so you will need to do some research. I suggest that you ask at [[Wikimedia Commons]], which has a lot of volunteer experts in borderline copyright cases. [[User:Cullen328|<b style="color:#070">Cullen</b><sup style="color:#707">328</sup>]] [[User talk:Cullen328|<span style="color:#00F">''Let's discuss it''</span>]] 02:56, 27 January 2017 (UTC) |
|||
Mirzakhani solved this counting problem by relating it to the problem of computing volumes in moduli space—a space whose points correspond to different complex structures on a surface genus '''<span style="color:red">Failed to parse (SVG (MathML can be enabled via browser plugin): Invalid response ("Math extension cannot connect to Restbase.") from server "http://localhost:6011/en.wikipedia.org/v1/":): {\displaystyle g}</span> .''' In her thesis, Mirzakhani found a volume formula for the moduli space of bordered Riemann surfaces of genus g {\displaystyle g} with n {\displaystyle n} geodesic boundary components. From this formula followed the counting for simple closed geodesics mentioned above, as well as a number of other results. This led her to obtain a new proof for the formula discovered by Edward Witten and Maxim Kontsevich on the intersection numbers of tautological classes on moduli space. |
|||
==Notability of a woman architect to acheive publication, how do I know when I have added enough== |
|||
I have created a page for licensed California architect Violeta Autumn and I submitted it before I was completed so It was not apparent how notable she was and it was immediately declined. I have added a lot more information, including some photos of awards, and added references to magazines, publications, and other libraries. I have also included her government work and election to public office using her architecture and planning background. This is my first page. How soon will I know when I have done enough? Do the images help?[[User:Sausalitoarchitect|Sausalitoarchitect]] ([[User talk:Sausalitoarchitect|talk]]) 20:41, 26 January 2017 (UTC) |
|||
Thanks. [[Special:Contributions/76.14.122.5|76.14.122.5]] ([[User talk:76.14.122.5|talk]]) 19:28, 31 December 2024 (UTC) |
|||
: Welcome to the Teahouse. There are over 400 drafts awaiting review. Not many have been waiting more than 4 weeks since submission, and you might be lucky and get a response more rapidly. Images rarely have any influence on the acceptability of an article; what will matter most is the quality of the references. --[[User:David Biddulph|David Biddulph]] ([[User talk:David Biddulph|talk]]) 21:11, 26 January 2017 (UTC) |
|||
:<s>Hi IP 76.14.122.5. Generally, the best place to discuss something like this is on the talk page for the article in question, which in this case would be [[:Talk:Maryam Mirzakhani]]; however, if you truly believe there's an error in the article, you can be [[:WP:BOLD]] and fix it yourself if you think you can. Please understand though that "fix it" in this context means to correct the article in accordance with relevant Wikipedia policies and guidelines, and the policies/guidelines most likely applicable in this case are going to be [[:Wikipedia:Verifiability]] and [[:Wikipedia:No original research]]. Ideally, you're going to need to find [[:WP:SECONDARY]] [[:WP:RS|reliable sources]] (as defined by Wikipedia) to cite in support of the change you want to make; [[:WP:VNT|even if you know such changes to be true]], you're still going to need to cite reliable sources in support to allow others to verify the changes. If you just make a change without providing any citations to a reliable source in support, there's a good chance the change will be undone by another user. Given that this seems to be related to mathematics, you might argue that "proving" the information to be incorrect based on Mirakhani's academic thesis is more than sufficient in and of itself, but Wikipedia generally requires something more and a thesis is going to be, for the most part, considered a [[:WP:PRIMARY]] source and could have other issues as explained in [[:WP:SCHOLARSHIP]]. You could also try asking about this at [[:Wikipedia:WikiProject Mathematics]] since that's where you're going to have a better chance of finding someone sufficiently versed in mathematics who might be able to help sort this out. -- [[User:Marchjuly|Marchjuly]] ([[User talk:Marchjuly|talk]]) 19:52, 31 December 2024 (UTC)</s>;<ins>post edited. -- 22:29, 31 December 2024 (UTC)</ins> |
|||
::Hello, {{u|Sausalitoarchitect}}. The best of your sources is probably the six page article in ''Progressive Architecture'' about a home she designed for herself. By the way, that was a different publication and was unrelated to ''Architecture'' magazine. I have seen photos of this home online and agree that it is a stunning design. However, a single write-up of a single project is not really enough to establish the notability of an architect. We are looking for sustained, ongoing coverage of multiple projects in several publications. Then there is her service on the Sausalito City Council and planning commission. Your sources for that are the local Marin County daily paper which gives routine coverage of routine local government business to all elected officials in that county. Sausalito, though a beautiful tourist attraction, is a very small town of only 7000 people. I know it well as I lived there briefly 45 years ago and visit it often. There is no presumption that an elected official in such a small town is notable. Then, she wrote an unusual cookbook. I do not think that is enough to establish notability as an author. Photos of minor awards are of no use. If any award was a major one, and is discussed in reliable sources, that would be helpful. The bottom line is that, after reading your draft in its current state, I am not convinced that she is notable as Wikipedia defines that term. [[User:Cullen328|<b style="color:#070">Cullen</b><sup style="color:#707">328</sup>]] [[User talk:Cullen328|<span style="color:#00F">''Let's discuss it''</span>]] 03:53, 27 January 2017 (UTC) |
|||
::Please disregard my reply IP 76.14.122.5. I misunderstood what you were asking about. ColinFine's suggestion below seems to be the best course of action here. -- [[User:Marchjuly|Marchjuly]] ([[User talk:Marchjuly|talk]]) 22:29, 31 December 2024 (UTC) |
|||
:Hello, IP user. What you are reporting is a problem in the software, the network, or the user interface, and not in the content of the article. Generally, technical problems of this sort are better handled at [[WP:VPT]] than here. However, I'm not seeing that problem, either on the browser on my laptop, or on the Android app. Is it repeatable, or might it have been a temporary glitch? |
|||
:Actually, now I look at it, the URL above appears to be a local proxy, so it may be that whoever manages your local network has not configured the proxy in a way that Wikipedia requires. Again, [[WP:VPT]] is a better place to ask about this. |
|||
:@[[User:Marchjuly|Marchjuly]]. The problem that the IP is reporting is obviously a technical one, so your answer is entirely off the point. [[User:ColinFine|ColinFine]] ([[User talk:ColinFine|talk]]) 20:21, 31 December 2024 (UTC) |
|||
::Thanks for catching my mistake and pointing the OP in the right direction {{u|ColinFine}}. I've stricken my reply so as to not confuse the OP or anyone else. -- [[User:Marchjuly|Marchjuly]] ([[User talk:Marchjuly|talk]]) 22:29, 31 December 2024 (UTC) |
|||
== Stuck on Puerto Rico outages notability. == |
|||
==Missing Information== |
|||
I was wondering how to go about a situation where I feel an article is missing information about a topic, but I don't feel in the slightest bit confident in actually writing any details about the situation. I'd like to point out the specific omission, but not actually write up the wording for the article itself. |
|||
Hi, new to Wikipedia here and probably gonna create a draft on the Puerto Rico power outage crisis but wanted to ask: |
|||
Also, is this handled differently if the page is "semi-protected"? [[User:JQ1981|JQ1981]] ([[User talk:JQ1981|talk]]) 19:17, 26 January 2017 (UTC) |
|||
I’m stuck in the notability of this topic, so, is the Puerto Rico power outage crisis notable enough for Wikipedia? By power outage crisis I mean the beginning of the Puerto Rican outages from [[Hurricane Maria]] to [https://amp.cnn.com/cnn/2024/12/31/us/puerto-rico-power-outage now] since it has lasted multiple years with sustained media coverage when an outage does occur. |
|||
Cheers! [[Special:Contributions/66.50.50.222|66.50.50.222]] ([[User talk:66.50.50.222|talk]]) 20:29, 31 December 2024 (UTC) |
|||
:Rather than a new article, you could expand on the outages already documented in [[:Puerto Rico Electric Power Authority]] and in [[:LUMA Energy]]. [[User:Schazjmd|<span style="color:#066293;">'''Schazjmd'''</span>]] [[User talk:Schazjmd|<span style="color:#738276;">''(talk)''</span>]] 20:44, 31 December 2024 (UTC) |
|||
::A search within Wikipedia on "Puerto Rico power outage" yields a list of several articles, including [[:Puerto Rico Electric Power Authority]] and in [[:LUMA Energy]] [[User:David notMD|David notMD]] ([[User talk:David notMD|talk]]) 21:15, 31 December 2024 (UTC) |
|||
::Noted. Will expand those articles then instead. Thanks! [[Special:Contributions/66.50.50.222|66.50.50.222]] ([[User talk:66.50.50.222|talk]]) 21:40, 31 December 2024 (UTC) |
|||
:My first impression would be that an article is likely warranted. PR is an island of ~3 million people, in the same neighborhood as Los Angeles. If Los Angeles were having sustained multi-year-long Issues with electrical service, there would be an article about it. [[WP:NONENG|Non-English language sources are perfectly acceptable]] for citing in articles, as long as a little care is observed. |
|||
:Note, to the anonymous editor: if you [[WP:create an account|create an account]] you get your own shiny neato [[WP:userspace|userspace]] to use mostly at your leisure, where for instance you can work on draft articles with no hurry. I have one underway in mine in fact. And thank you again for being interested in contributing to Wikipedia! |
|||
:(Regarding LA: some may have had come to mind the [[California energy crisis]], but, neat fact, LA actually escaped impact from that because of having [[Los Angeles DWP|its own municipal utility]] with its own generation capacity!) --[[User:Slowking Man|Slowking Man]] ([[User talk:Slowking Man|talk]]) 01:38, 1 January 2025 (UTC) |
|||
::<small>Note to [[User:Slowking Man]]: The userspace is the wrong place to draft an article. Either use your Sandbox or else follow instructions at [[WP:YFA]] to create a draft. [[User:David notMD|David notMD]] ([[User talk:David notMD|talk]]) 03:33, 1 January 2025 (UTC)</small> |
|||
:::Is "userspace" not the colloquial term for "your user page and any and all subpages of it"? The "official" user sandbox link is [[Special:MyPage/sandbox]], which takes you to the /sandbox subpage of your user page. --[[User:Slowking Man|Slowking Man]] ([[User talk:Slowking Man|talk]]) 05:29, 1 January 2025 (UTC) |
|||
::Hello, the IP editor (66.xx) here, just created my account, and I mainly refrained from creating an article due to being worried about a potential conflict-of-interest (I live in Puerto Rico myself). I might work on a draft later today and collect sources (as @[[User:BusterD|BusterD]] suggested), thank you all btw for helping me clear up this question I’ve had for some time now! [[User:Atheions|Atheions]] ([[User talk:Atheions|talk]]) 04:11, 1 January 2025 (UTC) |
|||
:On the other hand, [[User:Slowking Man]] is correct that an article might be created on this newsworthy subject and that one shouldn't necessarily rely strictly on English-language sources. Looking at the existing material, it certainly seems a sequence of outages could be established from some of the reliable sources already applied to pagespace. I agree with [[User:David notMD]] that [[WP:YFA]] is a place to consider how to start a new page. I would start collecting sources, online and in print. [[User:BusterD|BusterD]] ([[User talk:BusterD|talk]]) 03:58, 1 January 2025 (UTC) |
|||
:Apologies if jumping the gun or creating a draft too early, but I have [https://en.m.wikipedia.org/wiki/Draft:2017-25_Puerto_Rico_power_outages created one]. Won’t be able to do much progress today but will def collect sources to use. [[User:Atheions|Atheions]] ([[User talk:Atheions|talk]]) 07:32, 1 January 2025 (UTC) |
|||
== Am I allowed to post something about a game I am making on Wikipedia? == |
|||
: Welcome to the Teahouse. You can start a new section on the article's talk page, and explain what change you propose, supporting it with references to published independent [[WP:reliable sources|reliable sources]]. --[[User:David Biddulph|David Biddulph]] ([[User talk:David Biddulph|talk]]) 19:31, 26 January 2017 (UTC) |
|||
Am i allowed to post somthing about a game i am making? On Wikipedia [[User:Aaronfart14|Aaronfart14]] ([[User talk:Aaronfart14|talk]]) 22:09, 31 December 2024 (UTC) |
|||
:Allowed? Yes. Good idea? No. Likely to remain on Wikipedia? No. Writing an article is difficult, particularly for new editors. And since this is about '''your''' game, you have a conflict of interest in writing about it. See [[H:YFA]] and [[WP:COI]]. Simply posting information about your game rather than writing an article would be promotion. See [[WP:PROMOTION]] [[User:Meters|Meters]] ([[User talk:Meters|talk]]) 22:20, 31 December 2024 (UTC) |
|||
::{{u|Aaronfart14}}, the relevant content guideline is [[WP:Wikipedia is not for things made up one day]]. [[User:Cullen328|Cullen328]] ([[User talk:Cullen328|talk]]) 02:36, 1 January 2025 (UTC) |
|||
:::If your game goes public, and if it becomes so popular that people are publishing about it, then there is a chance that someone other than you will create an article about it. [[User:David notMD|David notMD]] ([[User talk:David notMD|talk]]) 03:37, 1 January 2025 (UTC) |
|||
== Writing about Childhood and Early Life == |
|||
==How to upload a photo== |
|||
Well i am new to Wikipedia and i have found that some articles are lacking some photos. as photos enhances the beauty of the page so i want to upload photos but when i click ''Click here to start the Upload Wizard'' it says that i am new and can't upload the photo so what is the procedure[[User:Mian Moazam|Mian Moazam]] ([[User talk:Mian Moazam|talk]]) 15:24, 26 January 2017 (UTC) |
|||
I am writing an article about Andrea Sheridan Ordin, who is already included in two existing wikipedia articles entitled, "List of first women lawyers and judges in California" and "United States District Court for the Central District of California." I am interviewing her personally and have reputable sources about her career notability, but I'm not sure how to write her "early life" section, since there are not many sources describing her childhood aside from her firsthand account. How to I write about her early life without secondary sources? [[User:Aharten97|Aharten97]] ([[User talk:Aharten97|talk]]) 22:17, 31 December 2024 (UTC) |
|||
:Hi [[User:Mian Moazam|Mian Moazam]], and welcome to the Teahouse. According to [[Wikipedia:Uploading images#Procedure to upload]], only registered users with [[WP:AUTOCONFIRM|autoconfirmed rights]] can upload images. This means that your account has to be at least ''four days old'', and you must have made ''at least ten edits'' on Wikipedia. Before this time, I would like to suggest that you read Wikipedia's policy on [[WP:Image use policy|Image use policy]], especially the issues concerning copyright. Generally, only images that are part of the [[Public domain]] and works that you have created and that you are releasing into the public domain are permitted for use on Wikipedia (there are limited exceptions for images under certain free licenses and fair use of copyrighted images in Wikipedia under US law, but I would advise everyone against treading into that domain, except for really experienced Wikipedia editors or US lawyers). --'''<span style="color:maroon"><span class="smallcaps" style="font-variant:small-caps;">[[User:Talk2chun|talk2Chun]]</span></span>'''<sup>([[User_talk:Talk2chun|talk]]) ([[Special:Contributions/Talk2chun|contributions]])</sup> 15:50, 26 January 2017 (UTC) |
|||
:{{re|Aharten97}} It's not possible without reliable secondary published sources. For the policies, see [[WP:V]]. Personal notes from an interview are neither published nor reliable, and self-published statements are rarely reliable. If it's noteworthy it will have been published somewhere. If not then it isn't. -- [[user:zzuuzz|zzuuzz]] <sup>[[user_talk:zzuuzz|(talk)]]</sup> 22:45, 31 December 2024 (UTC) |
|||
::https://ethics.lacity.gov/news/murray-and-ordin-re-elected-as-ethics-commission-leaders/ provides some info for an Early life and education section. [[User:David notMD|David notMD]] ([[User talk:David notMD|talk]]) 04:24, 1 January 2025 (UTC) |
|||
== What to write on your talk page? == |
|||
:Thank You Talk2Chun[[User:Mian Moazam|Mian Moazam]] ([[User talk:Mian Moazam|talk]]) 16:22, 26 January 2017 (UTC) |
|||
Above question [[User:HELSINKI!233|HELSINKI!233]] ([[User talk:HELSINKI!233|talk]]) 22:32, 31 December 2024 (UTC) |
|||
==Meghwal == |
|||
:Hi and welcome, @[[User:HELSINKI!233|HELSINKI!233]]. Your talk page is where other editors can leave messages for you or begin conversations with you. You can learn more at [[WP:USERTALK|the guidelines for user pages]]. [[User:Schazjmd|<span style="color:#066293;">'''Schazjmd'''</span>]] [[User talk:Schazjmd|<span style="color:#738276;">''(talk)''</span>]] 22:37, 31 December 2024 (UTC) |
|||
I made an edit on meghwal page however this was deleted. I used the reference which already exists from the page? <!-- Template:Unsigned IP --><small class="autosigned">— Preceding [[Wikipedia:Signatures|unsigned]] comment added by [[Special:Contributions/2A02:C7D:5B71:E100:7D59:4492:CEB7:EB48|2A02:C7D:5B71:E100:7D59:4492:CEB7:EB48]] ([[User talk:2A02:C7D:5B71:E100:7D59:4492:CEB7:EB48#top|talk]]) 22:47, 25 January 2017 (UTC)</small> <!--Autosigned by SineBot--> |
|||
::It is not for chat, or your thoughts/opinions about stuff. Also, with a few exceptions, you are allowed to delete content from your Talk page, although some people prefer to archive older content instead. [[User:David notMD|David notMD]] ([[User talk:David notMD|talk]]) 04:19, 1 January 2025 (UTC) |
|||
== Citations about movies == |
|||
:Hey anon. If sources need to be repeated, this can easily be done by using ref names. So instead of typing: |
|||
I'm trying to improve [[The Lincoln Lawyer (film)]] |
|||
<nowiki><ref>Wood, T (2017) How to use ref names. Retrieved 25 January 2017</ref></nowiki> |
|||
1) Is IMDB considered a good citation for the cast list, producer name, etc? |
|||
:You would give the reference a name by typing: |
|||
2) How do I add a citation to an existing infobox? (Visual or Source editor) |
|||
<nowiki><ref</nowiki> '''name="wood"'''<nowiki>>Wood, T (2017) How to use ref names. Retrieved 25 January 2017</ref></nowiki> |
|||
Many thanks |
|||
:So that next time you need to use the reference, you simply include: |
|||
[[User:Littenberg|Ben]] ([[User talk:Littenberg|talk]]) 22:36, 31 December 2024 (UTC) |
|||
<nowiki><ref name="wood"/></nowiki> |
|||
:Hi and welcome, @[[User:Littenberg|Littenberg]]. Imdb is not considered a reliable source because much of its information is [[WP:UGC|user-generated]]. You can learn how to add citations at [[User:Nick Moyes/Easier Referencing for Beginners|Easier Referencing for Beginners]]. [[User:Schazjmd|<span style="color:#066293;">'''Schazjmd'''</span>]] [[User talk:Schazjmd|<span style="color:#738276;">''(talk)''</span>]] 22:39, 31 December 2024 (UTC) |
|||
::If you click on Edit at the top menu it allows you to edit the entire article, including the Infobox. [[User:David notMD|David notMD]] ([[User talk:David notMD|talk]]) 04:15, 1 January 2025 (UTC) |
|||
:and it will duplicate the reference without needing to type the whole thing over again, and link all inline citations to the same reference at the bottom of the page. Just make sure you include the "/" in the repeated references, (<nowiki><ref name="wood"/></nowiki>), or you will get an error. |
|||
:So when all is said and done, if you type this: |
|||
This is the first sentence.<nowiki><ref>Wood, T (2017) How to use ref names. Retrieved 25 January 2017</ref></nowiki>. This is the second sentence.<nowiki><ref name="wood"/></nowiki> Finally, this is the third sentence citing the same source.<nowiki><ref name="wood"/></nowiki> |
|||
:What you will get is this: |
|||
{{talkquote| |
|||
This is the first sentence.<ref name="wood">Wood, T (2017) How to use ref names. Retrieved 25 January 2017</ref> This is the second sentence.<ref name="wood"/> Finally, this is the third sentence citing the same source.<ref name="wood"/> |
|||
{{reflist talk}} |
|||
}} |
|||
== How do I create a Wikipedia article == |
|||
:Assuming {{u|Sitush}} made the revert because it looked unreferenced, using this technique should avoid similar mixups in the future. [[User:Timothyjosephwood|<span style="color:#a56d3f;font-family:Impact;">Timothy</span><span style="color:#6f3800;font-family:Impact;">Joseph</span><span style="color:#422501;font-family:Impact;">Wood</span>]] 13:09, 26 January 2017 (UTC) |
|||
How do I create an article on Wikipedia? [[User:Красный Октябрь|Красный Октябрь]] ([[User talk:Красный Октябрь|talk]]) 23:12, 31 December 2024 (UTC) |
|||
==Adding one organizations logo to my article== |
|||
Is it okey if I will add one organization's logo to my article with their permission[[User:Hatadurdyyev|Hatadurdyyev]] ([[User talk:Hatadurdyyev|talk]]) 10:19, 26 January 2017 (UTC) |
|||
:Hello {{u|Hatadurdyyev}}, and welcome to Wikipedia and the Teahouse! Not only is it OK, but you don't need their permission. Please see [[WP:LOGOS]] for information about how to proceed, and consider using the [[WP:FFU]] process as you are a new user unfamiliar with the [[WP:FAIRUSE]] guidelines, so that a more experienced contributor can help you upload your image and affix the proper fair use rationale. [[User:Psiĥedelisto|Psiĥedelisto]] ([[User talk:Psiĥedelisto|talk]]) 10:56, 26 January 2017 (UTC) |
|||
:Здравствуйте, @[[User:Красный Октябрь|Красный Октябрь]], and welcome to the Teahouse. {{User:ColinFine/PractiseFirst}} [[User:ColinFine|ColinFine]] ([[User talk:ColinFine|talk]]) 23:33, 31 December 2024 (UTC) |
|||
==What could be done about [[Democratic socialism]] and [[Social democracy]]?== |
|||
::Can you help me learn how to make one? I want to make an article on the Kazan bombing that happened about a 1.5 week ago. [[User:Красный Октябрь|Красный Октябрь]] ([[User talk:Красный Октябрь|talk]]) 23:50, 31 December 2024 (UTC) |
|||
:::{{u|Красный Октябрь}}, please be aware that the [[Russo-Ukrainian War]] is a designated contentious topic. You cannot write new content about that war until your account is [[WP:XC|Extended confirmed]], which means the account is over a month old (it is) ''and'' has made over 500 constractive edits (you have a long way to go). [[User:Cullen328|Cullen328]] ([[User talk:Cullen328|talk]]) 02:56, 1 January 2025 (UTC) |
|||
::::Then can someone else make an article about it? [[User:Красный Октябрь|Красный Октябрь]] ([[User talk:Красный Октябрь|talk]]) 04:08, 1 January 2025 (UTC) |
|||
:::::Teahouse Hosts are here to advise, but not to be authors or co-authors. [[User:David notMD|David notMD]] ([[User talk:David notMD|talk]]) 04:13, 1 January 2025 (UTC) |
|||
::::::Who do I ask about it? [[User:Красный Октябрь|Красный Октябрь]] ([[User talk:Красный Октябрь|talk]]) 08:59, 1 January 2025 (UTC) |
|||
:::::::{{u|Красный Октябрь}}, if you persist in discussing a designated contentious topic such as the [[Russo-Ukrainian War]] before you are extended confirmed, you may be blocked. So, please edit other topic areas until then. [[User:Cullen328|Cullen328]] ([[User talk:Cullen328|talk]]) 09:16, 1 January 2025 (UTC) |
|||
:::::::You could ask on [[Talk:Russo-Ukrainian War]]. Normally such talk pages are protected, but that one doesn't seem to be at this time. What might happen is that someone adds a paragraph about the incident to the existing article rather than create a new article. |
|||
:::::::I'll add that editors who aren't extended-confirmed generally aren't permitted to use the talk pages of such articles either, and I have always disagreed with this, mainly because constructive edit requests end up in [[WP:RFED]], making extra work for administrators. ~[[User:Anachronist|Anachronist]] <small>([[User talk:Anachronist|talk]])</small> 16:34, 1 January 2025 (UTC) |
|||
== Why are padlocks not automatically added when an article is protected? == |
|||
I've removed [[Social democracy]] sidebar on the [[Democratic socialism]] page on grounds that there is "not to be confused with" markup with respect to the former. |
|||
Sorry if this is the wrong place. [[User:Heyaaaaalol|Heyaaaaalol]] ([[User talk:Heyaaaaalol|talk]]) 02:38, 1 January 2025 (UTC) |
|||
Hopefully, this is ok and the confusion seems to extend to the contents of Democratic socialism even though [[Socialism]] in itself is considered as far left today and anticapitalist by definition. |
|||
:@[[User:Heyaaaaalol|Heyaaaaalol]] Because an administrator needs to add a protection notice (the padlocks) to a page in order to show the padlock on pages. Sometimes the padlocks do not get placed on a page, especially if it's a user page, unless if the protection notice is placed by an administrator. [[User:NicePrettyFlower|NicePrettyFlower]] ([[User talk:NicePrettyFlower|talk]]) 03:35, 1 January 2025 (UTC) |
|||
The socialism sidebar in itself would make sense on the social democracy page and the remaining socialist sidebar on democratic socialism has a link to social democracy under variants. |
|||
:[[User:Heyaaaaalol|Heyaaaaalol]], I have done some page protection myself. Usually when I apply protection, I wish to see the "padlock" icon applied. Sometimes when I use the protection script, I forget to click the toggle which leaves the padlock icon. In that case, there's a bot which usually fixes that mistake automatically. If you see a protected page without an icon, you might tell someone. Do our answers help? [[User:BusterD|BusterD]] ([[User talk:BusterD|talk]]) 03:44, 1 January 2025 (UTC) |
|||
As I'm not clear as to where the idea of democratic socialism comes from, who can one talk to about it? - [[User:JamesPoulson|JamesPoulson]] ([[User talk:JamesPoulson|talk]]) 03:34, 26 January 2017 (UTC) |
|||
:Hello, @[[User:Heyaaaaalol|Heyaaaaalol]]. To actually answer your question: because nobody has implemented that function in the software. I don't know whether there is a technical reason for that, or whether it's just that nobody's got round to it. Questions about the software and user interface are better asked at the Village pump: either [[WP:VPT]] or [[WP:VPR]] [[User:ColinFine|ColinFine]] ([[User talk:ColinFine|talk]]) 13:34, 1 January 2025 (UTC) |
|||
:{{ping|Heyaaaaalol}} Protecting a page is a log action in the MediaWiki software which powers Wikipedia and thousands of other wikis. It does not make an edit to the page. Logs are separate from edits. Displaying a padlock with a link is a Wikipedia practice. We do it by editing the page and adding special code which places a padlock in the corner instead of the normal text area. Other wikis may use no or other symbols for protected pages, place them in other places, and make no or other links on them. A MediaWiki feature to automatically display a symbol on protected pages was recently added at [[phab:T12347]] but it's disabled by default. I haven't examined how flexible it is but I guess it would be non-trivial for us to convert to using it when we already have a well-functioning system. There is so far only a single Wikimedia wiki which has set <code>wgEnableProtectionIndicators</code> to true in https://noc.wikimedia.org/conf/highlight.php?file=InitialiseSettings.php. [[User:PrimeHunter|PrimeHunter]] ([[User talk:PrimeHunter|talk]]) 15:08, 1 January 2025 (UTC) |
|||
== Multiple non-free album covers on one page == |
|||
:Welcome to the Teahouse, {{u|JamesPoulson}}. The proper places to discuss the similarities and differences between these two topics are the talk pages of the two articles. I disagree with the "not to be confused with" tag on the top of one of those articles, which in my opinion, should be used only in cases of articles entirely unrelated topics rather than closely related topics. These are closely related articles discussing very closely related political philosophies. I also disagree with the removal of the sidebar. Take a look at the list of members parties in the [[Socialist International]], the worldwide alliance of similar parties. Many call themselves "social democratic", many call themselves "socialist", and the U. S. affiliate is called the [[Democratic Socialists of America]]. Others use a variety of different wordings combining "democracy" and "socialism" in various ways. [[User:Cullen328|<b style="color:#070">Cullen</b><sup style="color:#707">328</sup>]] [[User talk:Cullen328|<span style="color:#00F">''Let's discuss it''</span>]] 04:40, 26 January 2017 (UTC) |
|||
For the album ''[[Breakfast with Girls]]'', there's an associated EP located at the section ''[[Breakfast with Girls#Brunch|Brunch]]''. The EP has an infobox but is currently without cover artwork, so I'm wondering if it's appropriate for the article to have another non-free image when one is already in use for the album's artwork. If not within free use, ''Brunch''{{'}}s artwork (seen [https://self.is/brunch here]) is mostly text on a black background, so would I be allowed to crop out the non-text part and use that as a [[Template:PD-textlogo|public domain text logo]] image in an infobox? [[User:Koopastar|Koopastar]] ([[User talk:Koopastar|talk]]) 04:50, 1 January 2025 (UTC) |
|||
::It is being discussed on the talk page and that you are of this opinion would indicate a specialist or two needs to have a look. |
|||
:Hi [[User:Koopastar|Koopastar]], I do think that another cover would be within fair use. There's also been instances of this in the past like [[In Rainbows#In Rainbows Disk 2]]. Happy editing! [[User:Justiyaya|'''<span style="color:#1d556d">Just</span>''']][[Special:Contributions/Justiyaya|'''<span style="color:#000000">i</span>''']][[User talk:Justiyaya#top|'''<span style="color:#6d351d">yaya</span>''']] 06:59, 1 January 2025 (UTC) |
|||
::The ideologies are derived from socialism and some political parties do have [[socialist]] origins but [[Democratic Socialism]] and [[Social democracy]] might as well be considered synonyms and merged if they have links to each other. |
|||
== afd == |
|||
::Obviously, a self-described social democrat would not want to [https://www.quora.com/What-is-the-meaning-of-the-phrase-seize-the-means-of-production seize the means of production]. |
|||
I really like a certain article, but it's afd, if it is deleted, is there a way to still view it [[User:Saarabout|🐢]] ([[User talk:Saarabout|talk]]) 05:24, 1 January 2025 (UTC) |
|||
::Here are some links to clarify their differences: |
|||
:History checker [[User:SimpleSubCubicGraph|SimpleSubCubicGraph]] ([[User talk:SimpleSubCubicGraph|talk]]) 05:26, 1 January 2025 (UTC) |
|||
::* [http://www.forbes.com/sites/timworstall/2015/11/21/whatever-the-heck-bernie-sanders-is-talking-about-its-not-socialism/ Whatever The Heck Bernie Sanders Is Talking About It's Not Socialism] |
|||
:If the article is still "public" currently, you can save a copy for yourself: [[Wikipedia:Download as PDF|Download as PDF]] (maybe see also [[mw:Help:Export|Help:Export]]). Otherwise go to: [[CAT:RESTORE]]. --[[User:Slowking Man|Slowking Man]] ([[User talk:Slowking Man|talk]]) 06:14, 1 January 2025 (UTC) |
|||
::* [https://www.cato.org/publications/commentary/bernie-not-socialist-america-not-capitalist Bernie Is Not a Socialist and America Is Not Capitalist] |
|||
::This appears to be about [[X11 color names]], which has been in existance since 2003 and has been edited more than 700 times since then. While it is unlikely that it will be deleted, as noted, you can save a copy to your computer. [[User:David notMD|David notMD]] ([[User talk:David notMD|talk]]) 13:55, 1 January 2025 (UTC) |
|||
::* [http://www.economist.com/blogs/democracyinamerica/2016/02/bernie-manifesto How much of a socialist is Sanders?] |
|||
::* [http://www.thelocal.dk/20151101/danish-pm-in-us-denmark-is-not-socialist Danish PM in US: Denmark is not socialist] |
|||
::* [http://www.nytimes.com/2010/02/20/opinion/20iht-edzimmermann.html Social Democracy in America?] |
|||
::* [http://billmoyers.com/2015/07/03/social-democracy-is-100-american/ Social Democracy Is 100-Percent American] |
|||
::* [[Social Democrats, USA]] |
|||
::* [[Welfare state]] |
|||
::* [[Otto_von_Bismarck#Social_legislation|Otto von Bismarck#Social_legislation]] |
|||
::* [[Ferdinand Lassalle]] |
|||
::* [[Social Democratic Party of Germany]] |
|||
::* [[Critique of the Gotha Program]] --[[User:JamesPoulson|JamesPoulson]] ([[User talk:JamesPoulson|talk]]) 05:53, 26 January 2017 (UTC) |
|||
== Hypothetically, could it be possible to write a netural autobiography? == |
|||
:::Hello again, {{u|JamesPoulson}}. Please sign your posts. Most of the links you posted are not reliable, independent sources. The Teahouse is not a place for lengthy discussion of article content or political ideology. It is instead a place to discuss the mechanics of editing Wikipedia. Please be aware that Wikipedia is not edited by "specialists" but by ''anyone'' with an interest in a topic, who is willing to summarize the content of reliable sources in accordance with our policies and guidelines. [[User:Cullen328|<b style="color:#070">Cullen</b><sup style="color:#707">328</sup>]] [[User talk:Cullen328|<span style="color:#00F">''Let's discuss it''</span>]] 05:49, 26 January 2017 (UTC) |
|||
Hypothetically, would it be possible for someone on Wikipedia to write a neutral autobiography of themselves? Showing the good and bad and making no favor to either side, providing proof and checking all of the boxes. How would that go and would it be accepted? What if say a Wikipedia administrator that is not really well known becomes for example the president of the US? Can they have oversight over their own article? Are they removed from their position? Can they no longer edit anything involving US politics due to their inherent bias? And finally has there been any real examples of this over the past 20+ years? [[User:SimpleSubCubicGraph|SimpleSubCubicGraph]] ([[User talk:SimpleSubCubicGraph|talk]]) 05:25, 1 January 2025 (UTC) |
|||
::::Ok, I'll make a note that [[The Economist]], [[The Local]] and the [[New York Times]] are not reliable sources and you're RIGHT this is not a place to discuss political ideology. Have a nice day. --[[User:JamesPoulson|JamesPoulson]] ([[User talk:JamesPoulson|talk]]) 05:53, 26 January 2017 (UTC) |
|||
:Hello, {{u|SimpleSubCubicGraph}}. Hypothetically, what you describe is possible but the president of the US example is implausible because credible candidates for that office would already be the subject of a Wikipedia article. More plausible would be a longtime editor elected to a state or provincial legislature. I see no reason why that person could not submit an autobiography through [[WP:AFC|Articles for Creation]] with full disclosure, have the article accepted and continue as an editor or even as an administrator. They should certainly recuse from the current legislative affairs of their state or province but otherwise I would not see a broader problem. As for whether anything like that has ever happened, I do not know. Maybe another editor does. [[User:Cullen328|Cullen328]] ([[User talk:Cullen328|talk]]) 06:49, 1 January 2025 (UTC) |
|||
:@[[User:SimpleSubCubicGraph|SimpleSubCubicGraph]] welcome to the Teahouse! Interesting questions. I would say that while it is hypothetically possible, guidelines still strongly discourage creations of autobiographies and I haven't seen an instance of it being successful outside of early Wikipedia. This would have to go through [[WP:AFC|articles for creation]] and the editor should [[Wikipedia:DISCLOSE|declare]] their conflict of interest. Subjects do not have oversight of their own article. |
|||
:I would argue that if someone is elected president of the US they would have a financial relationship with the federal government and should stay away from editing those topics. I think they would have potentially less of a conflict of interest with state governments and historical united states politics. I don't think one would be removed as an administrator because they are elected to a public office here. I don't know of any Wikipedian that has been elected. [[CongressEdits]] is probably the closest to an example of this. [[User:Justiyaya|'''<span style="color:#1d556d">Just</span>''']][[Special:Contributions/Justiyaya|'''<span style="color:#000000">i</span>''']][[User talk:Justiyaya#top|'''<span style="color:#6d351d">yaya</span>''']] 06:46, 1 January 2025 (UTC) |
|||
::Succeeding at autobiography is more likely to occur in an area such as [[WP:NACADEMIC]]. A senior professor at a university would have as models articles about other professors at their university. [[User:David notMD|David notMD]] ([[User talk:David notMD|talk]]) 14:02, 1 January 2025 (UTC) |
|||
:The only time I recall a good autobiography being written was when the CEO of a small business wrote one, submitted it for review, and it was accepted after some minor revisions. So it is possible. |
|||
:By now I've gained enough experience on Wikipedia that I could probably write a neutral biography about myself, but because I am not notable, there's no point. ~[[User:Anachronist|Anachronist]] <small>([[User talk:Anachronist|talk]])</small> 16:26, 1 January 2025 (UTC) |
|||
:a) It would probably go over reasonably well, presuming they followed the Correct way to do that kind of thing: don't create your own bio article (meaning here the literal "creating a new page in [[WP:Main namespace|Main namespace]]") and don't edit it directly but put up stuff on the [[WP:Talk page|Talk page]] for others to evaluate, revise, and put into the article if they decide. |
|||
:b) No, [[WP:OWN|no one "owns" articles]] and gets special "powers" over them. Since all Wikipedia content is "[[WP:Wikipedia is free content|free as in freedom]]" you and anyone else can copy it put it up elsewhere and do whatever with the copies, as long as you credit the original creators. |
|||
:c) Why would they get adminship removed if they haven't misused it? |
|||
:d) They probably ought to stay away from US politics content yes, being rather [[WP:INVOLVED]]. Also realistically the POTUS is not going to have ample free time to devote to Wikipedia contributing, or to be inclined to devote what little precious free time they get to, doing more work. |
|||
:e) Have a look at [[WP:List of Wikipedians with articles]]. --[[User:Slowking Man|Slowking Man]] ([[User talk:Slowking Man|talk]]) 17:26, 1 January 2025 (UTC) |
|||
== Wiki page written by subject or friend of subject? == |
|||
:::::{{u|JamesPoulson}}, opinion pieces are reliable sources ''only'' for the opinions of their authors, no matter how exalted the publication they appear in. Every source must be evaluated individually in context. Wikipedia articles in themselves are not reliable sources, although the references they contain ought to be to reliable sources. [[User:Cullen328|<b style="color:#070">Cullen</b><sup style="color:#707">328</sup>]] [[User talk:Cullen328|<span style="color:#00F">''Let's discuss it''</span>]] 06:16, 26 January 2017 (UTC) |
|||
Hello, I saw a page on Wikipedia that looks like it was, based on the way it's presented, mostly written by the person who is the subject of the page, or a close friend. What is the standard flag or way to raise this on the page's talk page? Thank you for your help. [[User:FireBatV|FireBatV]] ([[User talk:FireBatV|talk]]) 08:05, 1 January 2025 (UTC) |
|||
:Hello @[[User:FireBatV|FireBatV]]! Welcome to the Teahouse. The standard way is to tag the article with the [[Template:COI]] on the main article page. Alternatively, you can use [[Wikipedia:Twinkle|Twinkle]] to tag it as well. '''[[User:TNM101|<span style="color:red;">TNM</span><span style="color:black;">101</span>]]''' ([[User talk:TNM101|<span style="color:blue;">chat</span>]]) 12:17, 1 January 2025 (UTC) |
|||
==Creating a page for a company== |
|||
What is the proper way to create a page for a company I work for? What is the proper language etc? |
|||
:: Thank you for the help! I've added the {{tlx|Template:COI}} to the page in question ([[Rhett Ayers Butler]]) and as per the instructions on the template page I am have also added{{tlx|Connected contributor}} to the talk page, so editors can take it from there. Thank you again for the help and the friendly welcome, my issue has been resolved. [[User:FireBatV|FireBatV]] ([[User talk:FireBatV|talk]]) 21:13, 1 January 2025 (UTC) |
|||
[[User:Asnowstaff|Asnowstaff]] ([[User talk:Asnowstaff|talk]]) 01:21, 26 January 2017 (UTC) |
|||
:::No problem! Feel free to reach out if you need any more help '''[[User:TNM101|<span style="color:red;">TNM</span><span style="color:black;">101</span>]]''' ([[User talk:TNM101|<span style="color:blue;">chat</span>]]) 06:46, 2 January 2025 (UTC) |
|||
: It is strongly suggested that you don't create an article (this an encyclopedia not a promotional tool) for a company you work for, and instead use [[WP:AFC]] to suggest suggest someone else write it. If you want to go ahead anyway, then read [[WP:YFA]], [[WP:COI]], and [[WP:PAID]] for how to create your first article, how to handle conflict of interest, and how to disclose you are being paid to edit. [[User:RudolfRed|RudolfRed]] ([[User talk:RudolfRed|talk]]) 02:19, 26 January 2017 (UTC) |
|||
::{{ping|Asnowstaff}} {{ping|RudolfRed}} A small correction: [[Wikipedia:Requested articles]] is where one requests than an article be written. [[WP:AFC]] is for writing an article yourself which, to reiterate, you should not do about a company you work for. – [[User:Joe Roe|Joe]] <small>([[User talk:Joe Roe|talk]])</small> 09:10, 26 January 2017 (UTC) |
|||
::: [[WP:AFC]] says that it can be used if you have a conflict of interest. [[User:RudolfRed|RudolfRed]] ([[User talk:RudolfRed|talk]]) 15:16, 26 January 2017 (UTC) |
|||
:One further point, {{U|Asnowstaff}}, which might seem picky, but I think is important, especially for people in your position: We don't have "pages for" companies or anything else. We have "articles about" subjects, which are neither for nor against the subject, but just summarise what reliable independent sources say about it, favourable or unfavourable. (I realise that your "for" probably didn't mean "for" in that partisan sense, but articles here are not "on behalf of" the subject either). --[[User:ColinFine|ColinFine]] ([[User talk:ColinFine|talk]]) 18:58, 26 January 2017 (UTC) |
|||
== Editing the Wikipedia page of "The crown" == |
|||
==Error: Mary Tyler Moor Show..How fix?== |
|||
The page states "first show to..." The given footnote does not say that, and the That_Girl factually contradicts the assertion. |
|||
i want to edit the Wikipedia page of "the crown" on netflix why i wont add the seventh season of "the crown" if you cant tell the staff at netflix please add aditional information like the royal wedding of prince william and kate Middleton and the queen involvement in the 2012 summer olympics in london england alongside James bond!!! [[Special:Contributions/89.128.137.159|89.128.137.159]] ([[User talk:89.128.137.159|talk]]) 14:08, 1 January 2025 (UTC) |
|||
Would "an early show" or "at the start of the trend" or so,me such be appropriate edits? |
|||
:Hello IP Editor: there was no 7th season of The Crown, so your additions were inappropriate and were removed. Please don't add fake information to Wikipedia. <span style="background-color: RoyalBlue; border-radius: 1em; padding: 3px 3px 3px 3px;">'''[[User:Qcne|<span style="color: GhostWhite">qcne</span>]]''' <small>[[User talk:Qcne|<span style="color: GhostWhite">(talk)</span>]]</small></span> 14:18, 1 January 2025 (UTC) |
|||
TIA[[User:C.o.o.|C.o.o.]] ([[User talk:C.o.o.|talk]]) 00:54, 26 January 2017 (UTC) |
|||
::False information is vandalism, which if continued can lead to your IP address being blocked and any account your subsequently register being blocked. [[User:David notMD|David notMD]] ([[User talk:David notMD|talk]]) 15:20, 1 January 2025 (UTC) |
|||
:Welcome to the Teahouse, {{u|C.o.o.}}. Simply edit the article to more accurately summarize what the source says. If your edit is reverted, discuss the matter on the article's talk page. [[User:Cullen328|<b style="color:#070">Cullen</b><sup style="color:#707">328</sup>]] [[User talk:Cullen328|<span style="color:#00F">''Let's discuss it''</span>]] 05:05, 26 January 2017 (UTC) |
|||
== The show button doesn't work on phone == |
|||
:Thank you for the warm welcome. The sentence in question is above the editable portions of the page. (See: [[The Mary Tyler Moore Show]] ). |
|||
:This suggests to this newbie that such a change is a case of "above my pay grade". [[User:C.o.o.|C.o.o.]] ([[User talk:C.o.o.|talk]]) 22:43, 26 January 2017 (UTC) |
|||
Hi. Some articles have [https://s8.uupload.ir/files/screenshot_20250101_182319_chrome_rpkr.jpg this] dialouge box on top of them. But when we touch the show button on phone (Chrome for Android for me), [https://s8.uupload.ir/files/screenshot_20250101_182322_chrome_yxid.jpg nothing] happens. I don't know about desktop. [[User:Aminabzz|Aminabzz]] ([[User talk:Aminabzz|talk]]) 15:38, 1 January 2025 (UTC) |
|||
::{{yo|C.o.o.}} There are no [[WP:PROTECT|edit restrictions]] on the article at this time, so the entire page should be editable. The [[MOS:LEAD|introduction]] (above the table of contents) does not by default have its own edit button like the other sections do, which may be a source of confusion. To edit it, click either the "Edit" or "Edit source" button at the very top of the page, which will open up an edit page for the entire article. [[User:Clpo13|clpo13]]<sub>([[User_talk:Clpo13|talk]])</sub> 23:04, 26 January 2017 (UTC) |
|||
:What is an example of such an article? What happens when you use the Wikipedia app? Does it work then? ~[[User:Anachronist|Anachronist]] <small>([[User talk:Anachronist|talk]])</small> 16:21, 1 January 2025 (UTC) |
|||
:::{{u|Cullen328}} and {{u|Clpo13}} If the idea that the absence of a specific "edit" link implies that editing the lead is "above the pay grade" of newbies, is a common belief, then we should probably do something about it. I've [[WP:Village pump (idea lab)#Lack of a specific edit link for lead sections may be problematic.|opened a discussion at VPI]] about this issue. [[User:Dodger67|Roger (Dodger67)]] ([[User talk:Dodger67|talk]]) 09:28, 28 January 2017 (UTC) |
|||
:Hello, @[[User:Aminabzz|Aminabzz]], and welcome to the Teahouse. |
|||
:I agree. If I look at [[Amathus]], I get that box (from template {{tl|expand language}}) at the top, but on a browser on my Android phone, when I pick "Show" it replaces the word "Show" with "Hide", but doesn't expand the box, so I can't see additional information. |
|||
:However, the box has a "Learn more" button, and if I pick that it shows me a little more, but not the full information that I see on my browser on a computer. |
|||
:This looks like a bug in the user interface: [[WP:VPT]] is a better place to ask/report such things than here, and I suggest you post there. Thank you for pointing it out. [[User:ColinFine|ColinFine]] ([[User talk:ColinFine|talk]]) 18:04, 1 January 2025 (UTC) |
|||
== How to find the full form of abbreviations on phone? == |
|||
==My Article Has Been Problem Tagged, What Do I Do?== |
|||
I have made the article [[Grand Trunk Western 4070]] but it was tagged as having problems with the sources cited credibility. I cited five references and I don't know how to fix this problem since the references I cited were the only ones that I could find. What should I do?[[User:Dolothedolphin|DolotheDolphin]] ([[User talk:Dolothedolphin|talk]]) 00:16, 26 January 2017 (UTC) |
|||
:Hi, {{u|DolotheDolphin}}. Welcome to the Teahouse. The reason your article was tagged was not credibility, it was notability. Notability is what we call our standard for inclusion in the encyclopedia and it is based on whether the subject has been written about in detail in [[WP:RS|multiple reliable sources]]. The key in your article's case is the reliable part. I happen to be familiar with your subject matter here and I know this locomotive is notable, so I'm going to pull that tag. Sources do not have to be online and I'm certain the 4070 has been written up in both ''Trains'' and ''Railroad'' magazines. You should find them and cite them. [[WP:Referencing for beginners]] will show you how and if you have any further questions, feel free to come back and ask. I had a 4070 excursion go right by my home in SW Michigan in the late 60s. Good memories. Thanks. [[User:John from Idegon|John from Idegon]] ([[User talk:John from Idegon|talk]]) 02:10, 26 January 2017 (UTC) |
|||
::Hello, {{u|DolotheDolphin}}. Like John from Idegon, I am interested in historic steam locomotives. You may be interested in an article that I wrote about a famous example, [[Sierra No. 3]], often called the "Movie star locomotive". [[User:Cullen328|<b style="color:#070">Cullen</b><sup style="color:#707">328</sup>]] [[User talk:Cullen328|<span style="color:#00F">''Let's discuss it''</span>]] 05:25, 26 January 2017 (UTC) |
|||
Hi. On Wikipedia, some abbreviation words have dotline underlines. When we hover the mouse cursor on them on PC we can see the full form. For example, TBA reveals to be "to be announced". |
|||
== Draft Approval Timeline == |
|||
But on mobile phones (Chrome for Android for me) there is no mouse so that we hover the cursor on them! So how can we find the full form of abbreviations on these dotline-underlined words in cellphones? Holding the word doesn't work. |
|||
Hello, I have worked on creating a page for an author, Essel Pratt, who is an author of Horror, Fantasy, and Sci-fi. He has been published alongside Clive Barker, William F. Nolan, and H.P. Lovecraft, to name a few. In addition, one of his articles from the Inquisitr was used as a reference for a Wikipedia article on Vegan Cheese. He is a rather prolific author that has had his newest novel, Sharkantula, widely publicized on various Internet sites. However, the Essel Pratt draft has been in draft form for over a month now. I continue to update it as I can. However, I am worried that it may be lost in the sea of submissions. Can someone spread light on when I may be able to find out when it may be approved? <!-- Template:Unsigned --><small class="autosigned">— Preceding [[Wikipedia:Signatures|unsigned]] comment added by [[User:Immortalgaze|Immortalgaze]] ([[User talk:Immortalgaze#top|talk]] • [[Special:Contributions/Immortalgaze|contribs]]) 13:51, 25 January 2017 (UTC)</small> <!--Autosigned by SineBot--> |
|||
Look at [https://s8.uupload.ir/files/screenshot_20250101_182537_chrome_dbd2.jpg this] for seeing the dotlines. [[User:Aminabzz|Aminabzz]] ([[User talk:Aminabzz|talk]]) 15:40, 1 January 2025 (UTC) |
|||
:Hey [[User:Immortalgaze|Immortalgaze]]. AfC currently has a backlog of about 400 drafts, so it may take a while, but a volunteer will eventually review your submission. I know that isn't a very satisfying answer, but unfortunately there are many more editors submitting drafts than there are editors reviewing them. |
|||
:That's one of the limitations of using a smartphone as a browser. There is no notion of "hover". I have seen hovering implemented in some Samsung phones, in which holding your finger near to the screen without touching it is sensed as a hover, but this worked only in certain apps and wasn't a universal experience across all apps on the phone. Unless someone has a better answer, I'd say that features reliant on hovering are generally not accessible on smartphones. ~[[User:Anachronist|Anachronist]] <small>([[User talk:Anachronist|talk]])</small> 16:16, 1 January 2025 (UTC) |
|||
:In the meantime, I notice a couple of things looking over your submission: First, you include [[WP:EL|external links]] in the body, which isn't permitted. Either these should be formatted as refs, or they should simply be removed. Second, while there are a lot of sources provided, it looks like nearly all of them are works written by the subject. While this may establish that he is prolific, self-written works do not contribute to notability in the same way that pieces written by independent sources do. So it's probably a good idea to find some independent [[WP:RS|reliable sources]], and try to add them. As long as it may take to get a draft reviewed, if it fails the review, the whole wait just starts over again. [[User:Timothyjosephwood|<span style="color:#a56d3f;font-family:Impact;">Timothy</span><span style="color:#6f3800;font-family:Impact;">Joseph</span><span style="color:#422501;font-family:Impact;">Wood</span>]] 16:00, 25 January 2017 (UTC) |
|||
::Samsung devices with [[S Pen]] support actually support "hover" by well, hovering the S Pen close to the display. The display has an "active digitizer" that can sense the Pen with [[Near-field comms|NFC]]. Shows a little cursor and pops up stuff and you can hit the Pen's button to do things, very neat actually. ...Buuuut still doesn't work for the abbr stuff in browsers because it's handled differently in the system software (as it's not "really a mouse", it's handled by different code) and so doesn't "pass through" a mouse hover event down to the browser software. Shucks. --[[User:Slowking Man|Slowking Man]] ([[User talk:Slowking Man|talk]]) 17:07, 1 January 2025 (UTC) |
|||
:Oh yeah [[Template:abbr|this is a known problem for years]]. Abbr and friends use the standard [[HTML tags]] for such things, and browsers don't gaf apparently about making them "work" when there's no pointing device (mouse), and the attitude appears to be {{shrug}}. WP could implement some sort of JavaScript "workaround" that pops up a thing, but, that requires deploying something "globally" site-wide and that is a Big Deal so it needs Official approval: ask around at [[WP:VPT]] whether there's any effort in this direction. (FYI templates can't have JavaScript, it needs to be JS to "dynamically" add new page elements and display them) |
|||
:In the meantime the "workaround" is to hit edit and look at the wikitext. Or you could also view the page HTML source and use the browser's "find" to go to the abbreviation which will show its definition. Also also there might be some [[WP:userscript|userscript]] someone has made to make it pop up, which you can "install" to use while logged-in. (You could also always plug in a mouse/trackpad/etc or connect a wireless one {{;)}}) --[[User:Slowking Man|Slowking Man]] ([[User talk:Slowking Man|talk]]) 17:07, 1 January 2025 (UTC) |
|||
::@[[User:Aminabzz|Aminabzz]], [[User:Anachronist|Anachronist]], and [[User:Slowking Man|Slowking Man]]: I couldn't find any such script, so [[User:JJPMaster/abbrMobile.js|I made it]]. [[User:JJPMaster|JJP]]<sub>[[User talk:JJPMaster|Mas]]<sub>[[Special:Contributions/JJPMaster|ter]]</sub></sub> ([[She (pronoun)|she]]/[[Singular they|they]]) 05:08, 2 January 2025 (UTC) |
|||
== GA Spotcheck == |
|||
:In addition to the above, you should look for book reviews in mainstream news media or magazines, but avoid "reader reviews" such as on Amazon or goodreads, look only at professional reviews. Literary journals could also be good sources of information about the subject's background and career. [[User:Dodger67|Roger (Dodger67)]] ([[User talk:Dodger67|talk]]) 16:17, 25 January 2017 (UTC) |
|||
Apparently I might've misunderstood what is needed for a source spotcheck in a good article review. Could someone please explain what is needed? [[User:History6042|<span style="color:darkorange">History6042😊</span>]] '''([[User talk:History6042|<span style="color:blue">Contact me</span>]])''' 15:53, 1 January 2025 (UTC) |
|||
:{{u|Immortalgaze}}, I'm afraid in its current state [[Draft:Essel Pratt]] will never be reviewed, because you have not actually submitted it into the review queue yet. To do so please add {{tl|subst:submit}}, including the double pair of curly brackets, to the page. [[User:Dodger67|Roger (Dodger67)]] ([[User talk:Dodger67|talk]]) 16:25, 25 January 2017 (UTC) |
|||
:Appears that at [[Wikipedia talk:Good article nominations]], date 1/1/25, there has been a question raised about GA nomination reviews conducted by History60432. [[User:David notMD|David notMD]] ([[User talk:David notMD|talk]]) 17:07, 1 January 2025 (UTC) |
|||
::Yes, that is why I am asking. [[User:History6042|<span style="color:darkorange">History6042😊</span>]] '''([[User talk:History6042|<span style="color:blue">Contact me</span>]])''' 17:09, 1 January 2025 (UTC) |
|||
:@[[User:History6042|History6042]], note 2 on the [[Wikipedia:Good article criteria]] page explains source checking. If you're concerned that your interpretation isn't in line with the community's, you might find it helpful to discuss with other editors involved with the good article process at [[Wikipedia talk:Good article nominations]]. [[User:Schazjmd|<span style="color:#066293;">'''Schazjmd'''</span>]] [[User talk:Schazjmd|<span style="color:#738276;">''(talk)''</span>]] 17:33, 1 January 2025 (UTC) |
|||
::So it means Verifiable with no original research: |
|||
::it contains a list of all references (sources of information), presented in accordance with the layout style guideline; |
|||
::reliable sources are cited inline. All content that could reasonably be challenged, except for plot summaries and that which summarizes cited content elsewhere in the article, must be cited no later than the end of the paragraph (or line if the content is not in prose); |
|||
::it contains no original research; and |
|||
::it contains no copyright violations or plagiarism? [[User:History6042|<span style="color:darkorange">History6042😊</span>]] '''([[User talk:History6042|<span style="color:blue">Contact me</span>]])''' 19:25, 1 January 2025 (UTC) |
|||
:::The '''note''' says {{tq| "Ideally, a reviewer will have access to all of the source material, and sufficient expertise to verify that the article reflects the content of the sources; this ideal is not often attained. At a minimum, check that the sources used are reliable (for example, blogs are not usually reliable sources) and that those you can access support the content of the article (for example, inline citations lead to sources that agree with what the article says) and are not plagiarized (for example, close paraphrasing of source material should only be used where appropriate, with in-text attribution if necessary)."}} [[User:Schazjmd|<span style="color:#066293;">'''Schazjmd'''</span>]] [[User talk:Schazjmd|<span style="color:#738276;">''(talk)''</span>]] 20:40, 1 January 2025 (UTC) |
|||
::::Okay I think i did that but do I just need to like write it down? [[User:History6042|<span style="color:darkorange">History6042😊</span>]] '''([[User talk:History6042|<span style="color:blue">Contact me</span>]])''' 21:06, 1 January 2025 (UTC) |
|||
:::::Saying in your review what you checked is helpful. Perhaps read through [[Wikipedia_talk:Good_article_nominations/Archive_30#Spot_checks?|this discussion on spot checks]]. You might also find it useful to read through some GA reviews by experienced reviewers, see what they're doing in their written reviews that you can learn from. [[User:Schazjmd|<span style="color:#066293;">'''Schazjmd'''</span>]] [[User talk:Schazjmd|<span style="color:#738276;">''(talk)''</span>]] 21:47, 1 January 2025 (UTC) |
|||
::::::Thank you. [[User:History6042|<span style="color:darkorange">History6042😊</span>]] '''([[User talk:History6042|<span style="color:blue">Contact me</span>]])''' 00:39, 2 January 2025 (UTC) |
|||
== Creating an article heavily lacking sources :/ == |
|||
:What if an entire paragraph is plagiarized? Do I remove the all the material? [[User:Grace1701|Grace1701]] ([[User talk:Grace1701|talk]]) 02:56, 27 January 2017 (UTC) |
|||
I've been working on trying create an article ([[Draft:Millennium Force's effects]]) but I've run into some issues; including after initial submission. The only sources I can find are mainly from various YouTube videos (not tied to the subject) and very few separate links; so it's no wonder why it wasn't accepted. |
|||
==Foreign births in country year pages?== |
|||
During an unrelated cleanup I noticed a great deal of actress births on pages like [[1988 in Turkey]] are actually born in Germany and don't appear to have any presence/recognition in Turkey. |
|||
So I know what I've written down is true, but I don't have the secondary sources to prove it and that makes my info [[No original research|Original Research]]. What can I do? I was told I could try another Wiki (ex: Amusement Park Wiki) but is there anything I can do to keep it on Wikipedia? Thanks! [[User:Therguy10|<span style="color:blue;">'''Therguy10'''</span>]] ([[User talk:Therguy10|talk]]) 16:49, 1 January 2025 (UTC) |
|||
The biographies don't look notable and sometimes removing the birth makes the page an orphan. This is mainly affecting actresses and singers - other kinds of profiles look fine, which makes me think these biographies are being added by paid publicists. |
|||
:@[[User:Therguy10|Therguy10]] Welcome to theTeahouse. I'm afraid not. [[WP:RS|Reliable sources]] are fundamental to Wikipedia articles. See [[WP:42]]. [[User:Shantavira|Shantavira]]|[[User talk:Shantavira|<sup>feed me</sup>]] 18:01, 1 January 2025 (UTC) |
|||
Is there a policy on this? Are the edits below OK or is this overzealous? |
|||
:Hello, @[[User:Therguy10|Therguy10]]. I agree with Shantavira. Please see also [[WP:No amount of editing can overcome a lack of notability|No amount of editing can overcome a lack of notability]] [[User:ColinFine|ColinFine]] ([[User talk:ColinFine|talk]]) 18:07, 1 January 2025 (UTC) |
|||
:@[[User:Shantavira|Shantavira]] @[[User:ColinFine|ColinFine]] Thank you both! I do believe that the article has potential in the future, but I also understand it isn't notable enough as of now. With the article being a history of a subject, there is older information that I fear may not be written about. My hope is that the new changes made this year were important enough to spark some kind of interest for a source; I'll have to wait and see. [[User:Therguy10|<span style="color:blue;">'''Therguy10'''</span>]] ([[User talk:Therguy10|talk]]) 18:13, 1 January 2025 (UTC) |
|||
See: |
|||
::well i guess technically the changes were made last year...lol [[User:Therguy10|<span style="color:blue;">'''Therguy10'''</span>]] ([[User talk:Therguy10|talk]]) 18:14, 1 January 2025 (UTC) |
|||
[[https://en.wikipedia.org/enwiki/w/index.php?title=1988_in_Turkey&diff=prev&oldid=761868646]] |
|||
:::If you can find reliable sources, why not improve the '''Ride experience''' section of [[:Millennium Force]]? [[User:Schazjmd|<span style="color:#066293;">'''Schazjmd'''</span>]] [[User talk:Schazjmd|<span style="color:#738276;">''(talk)''</span>]] 18:27, 1 January 2025 (UTC) |
|||
[[https://en.wikipedia.org/enwiki/w/index.php?title=1986_in_Turkey&diff=prev&oldid=761868455]] |
|||
::::That was a strong consideration of mine. I certainly won't be able to fit ''everything'' inside of the main article but it might not hurt to add a little more than what I already have there. Once again my biggest issue would be collecting sources; this seems more doable. Thanks! [[User:Therguy10|<span style="color:blue;">'''Therguy10'''</span>]] ([[User talk:Therguy10|talk]]) 18:34, 1 January 2025 (UTC) |
|||
== How do i delete a Wikepedia page? == |
|||
[[User:Cybela|Cybela]] ([[User talk:Cybela|talk]]) 08:21, 25 January 2017 (UTC) |
|||
:Hello, {{u|Cybela}}, and welcome to the Teahouse! If they are not [[WP:N|notable]], you ought to pursue deletion and worry about other things only later. <span style="font-family: serif; letter-spacing: 0.1em">– [[User:Finnusertop|Finnusertop]]</span> ([[User talk:Finnusertop|talk]] ⋅ [[Special:Contributions/Finnusertop|contribs]]) 09:04, 25 January 2017 (UTC) |
|||
a school has a wikepedia page, (carlisle public schools) i have moved the page to inside the town page (carlisle masssachusetts) so how do i delete the old school page? it is useless (i moveed the page because it contains about only 1 paragraph, so why not make it a section) so yeah, if someone knows how to delete a page, please delete Carlisle Public Schools |
|||
:Thanks Finnusertop. The biographies I've checked aren't sourced and notability is questionable but I've found deletions a little confusing so I'm looking for smaller cleanups to help with for now. So I'm still interested to know if these edits are on the right track, if you put notability aside. The UK and US year pages look strict about this, but Turkey year pages not so much, so I couldn't be certain. [[User:Cybela|Cybela]] ([[User talk:Cybela|talk]]) 09:36, 25 January 2017 (UTC) |
|||
how to find page - go to the education part of carlisle, massachusetts, press carlisle public schools, and you will be there. [[User:Theawezomefriend12|Theawezomefriend12]] ([[User talk:Theawezomefriend12|talk]]) 19:39, 1 January 2025 (UTC) |
|||
:So, I did some more checking to see if there's a precedent but even the most notable biographies of people with international ancestry don't get listed this way (1) Donald Trump isn't in listed in births on [[1946 in Scotland]] and (2) Barack Obama isn't in 1961 births in Kenya for example. Based on this I'm feeling more confident to go ahead and clean up inconsistent births on year pages. [[User:Cybela|Cybela]] ([[User talk:Cybela|talk]]) 10:05, 25 January 2017 (UTC) |
|||
:[[Carlisle Public Schools]] is an article. If you believe it should be deleted, the process is to nominate it for deletion via the [[WP:AFD]] process. What you added to [[Carlisle, Massachusetts]] was rightfully deleted because you did not include any references. See [[Maynard, Massachusetts]] for an example of a referenced education section. I was going to point you to Acton or Sudbury, but those have unreferenced content. There is also [[Concord-Carlisle High School]], but that has its own problems. [[User:David notMD|David notMD]] ([[User talk:David notMD|talk]]) 20:02, 1 January 2025 (UTC) |
|||
::Okay, Thanks! [[User:Theawezomefriend12|Theawezomefriend12]] ([[User talk:Theawezomefriend12|talk]]) 20:30, 1 January 2025 (UTC) |
|||
:::{{re|Theawezomefriend12}} I'm going to offer some slightly different advice. For a small article in this situation: First copy the relevant text to the target article (the article about the town). If there are no references then be sure to add one or some. See [[WP:CWW]] for the correct process, if you are moving any text. Once that has been successfully done, turn the former article (the school district) into a redirect to the town as you did previously. There is no need to actually delete anything: redirects are useful for people trying to find things, such as information about Carlisle Public Schools, and can be safely left behind. -- [[user:zzuuzz|zzuuzz]] <sup>[[user_talk:zzuuzz|(talk)]]</sup> 21:12, 1 January 2025 (UTC) |
|||
::::Thanks for the advice, i will remove the deletion (i dont know if thats possible i am new) and make the page redierect to Carlisle, Massachusetts. [[User:Theawezomefriend12|Theawezomefriend12]] ([[User talk:Theawezomefriend12|talk]]) 21:30, 1 January 2025 (UTC) |
|||
:::::{{re|Theawezomefriend12}} I've helped a bit with the deletion part. The best way to learn is to just do things, so I'll leave you to continue the process (and will probably pop back in a bit). I'll just mention that it appears that the only reason your previous attempt was reverted was because you didn't include any references for the content you were adding/moving to the town article. Sufficient references probably exist in the district article so they can just be copied. If not, I'm sure they won't be difficult to find. -- [[user:zzuuzz|zzuuzz]] <sup>[[user_talk:zzuuzz|(talk)]]</sup> 21:38, 1 January 2025 (UTC) |
|||
::::::oh ok, i am propably going to be moving the main description and the new gallery part i added, i probably will not be added refrences and external links. [[User:Theawezomefriend12|Theawezomefriend12]] ([[User talk:Theawezomefriend12|talk]]) 21:45, 1 January 2025 (UTC) |
|||
== Hi! PARIS == |
|||
:: {{ping|Cybela}} I judge those edits to be legit and not at all zealous, since the information on these performers's pages clearly state that the births did not occur in Turkey. Notability is probably a concern, but it's quite likely that the entries are added by interested fans. |
|||
Good morning, good afternoon or good evening. Happy new year 2025. Please how to move to the main space the [[Draft:Hi! PARIS]]. This research center in AI is now one of the most important in France : https://www.lesechos.fr/tech-medias/intelligence-artificielle/une-dotation-de-70-millions-pour-la-formation-de-lelite-de-lia-en-france-2096150 and https://www.lemondeinformatique.fr/actualites/lire-hec-et-l-institut-polytechnique-remportent-l-appel-a-projets-cluster-ia-93818.html. Many thanks in advance for your help. Have a great day. [[Special:Contributions/2A01:CB00:B48:9900:1C67:231F:4C2F:9933|2A01:CB00:B48:9900:1C67:231F:4C2F:9933]] ([[User talk:2A01:CB00:B48:9900:1C67:231F:4C2F:9933|talk]]) 21:57, 1 January 2025 (UTC) |
|||
:: And what's going on with the styling of Kizil's page name? Within the article, it sometimes uses lower case "i" characters and sometimes uses whatever Unicode character that's used in the page name. Whether Kizil is popular/notable in Turkey is unclear from her article, but Evcen is shown as appearing in a couple of Turkey-based television shows. [[User:jmcgnh|<b><span style="color:#248F7D"> —jmcgnh</span></b>]]<sup><small><b>[[User_talk:jmcgnh|<span style="color:#58D582">(talk)</span>]] [[Special:Contributions/jmcgnh|<span style="color:#8F7D24">(contribs)</span>]]</b></small></sup> 10:14, 25 January 2017 (UTC) |
|||
:On the AoPS Wiki go to [[special:move]]. I don't know bout wikipedia tho [[Special:Contributions/73.31.42.97|73.31.42.97]] ([[User talk:73.31.42.97|talk]]) 22:22, 1 January 2025 (UTC) |
|||
::Sorry its [[Special:MovePage]] [[Special:Contributions/73.31.42.97|73.31.42.97]] ([[User talk:73.31.42.97|talk]]) 22:23, 1 January 2025 (UTC) |
|||
:Bonsoir, IP user. |
|||
:73. is right that the technique for moving pages is to use the [[WP:Move|Move]] function (though most people do it from their user-interface, rather than going to [[Special:MovePage]]. |
|||
:However, not logged in users do not have access to that function. |
|||
:I was going to advise you to submit it for review. However, I see that the draft was created in mainspace user BobVillars (since blocked for sockpuppetry) was then moved to Draft space by @[[User:Rosguill|Rosguill]], was submitted for review by an ipV6 user in the same range as you, then moved to main space again by McSyl, also blocked as a sockpuppet of BobVillars, and moved back to draft space ''again'' by @[[User:Janhrach|Janhrach]]. |
|||
:Neither you nor anybody else has since made any substantial edits to the draft. |
|||
:I am finding it very difficult to [[WP:assume good faith|assume good faith]]. I will ask you directly: are you BobVillars/McSyl? If so, you are evading your block, which is not permitted. [[User:ColinFine|ColinFine]] ([[User talk:ColinFine|talk]]) 22:59, 1 January 2025 (UTC) |
|||
::Hi Colin. Thanks for your answer. I don’t know what you are talking about. I am using 5G access so my Ip range is used by million of users. But if it is too complicated no worries, please cancel my request. This AI cluster is one of the most important in France and one of the biggest in Europe. So soon or later it will be back to mainspace. Have a great day. [[Special:Contributions/2A01:CB06:B064:82B5:2C9D:5034:3AC5:1BD4|2A01:CB06:B064:82B5:2C9D:5034:3AC5:1BD4]] ([[User talk:2A01:CB06:B064:82B5:2C9D:5034:3AC5:1BD4|talk]]) 06:21, 2 January 2025 (UTC) |
|||
== AoPS page == |
|||
:: {{ping|jmcgnh}}{{ping|Finnusertop}} Thanks jmcgnh. I went ahead and did some fixes for obvious cases, added a few tags for other pages with issues and have tried to find sources for the rest as you and Finnusertop suggest. My guess is that some of these pages were added when Wikipedia was less strict. I noticed that [[Wikipedia:WikiProject_Top_Model|the Top Model international franchise wikiproject]] may be responsible for generating a large number of biographies for people who appeared on TV approximately once or twice to participate in the contest's cycles and franchises across the world but otherwise have no discernible modelling career. [[User:Cybela|Cybela]] ([[User talk:Cybela|talk]]) 11:31, 28 January 2017 (UTC) |
|||
Why isn't there a page for [[Art of Problem Solving]]? [[Special:Contributions/73.31.42.97|73.31.42.97]] ([[User talk:73.31.42.97|talk]]) 22:15, 1 January 2025 (UTC) |
|||
==Being asked to rewrite but don't know why== |
|||
: Hi IP 73.31.42.97. There could be a couple reasons why: (1) nobody thought enough about the book/series to try and create one; or (2) somebody did try to create one, but the subject wasn't deemed to meet [[:Wikipedia:Notability (books)]] and was either deleted altogether or added to [[:Richard Rusczyk]]. ''{{no redirect|Art of Problem Solving}}'' current [[:WP:REDIRECT]]s to the "Rusczyk" article and its [[:H:PH|page history]] shows that there once was a stand-alone article about the book/series, but it was "redirected" per [[:Wikipedia:Articles for deletion/Art of Problem Solving (2nd nomination)]]. If you feel something has changed since then, you might want to first explain why to the administrator who closed the Articles for Deletion discussion. That administrator's name is {{u|Liz}}. Perhaps by asking at [[:User talk:Liz]], Liz can tell you what is needed for the article to be recreated or restored. -- [[User:Marchjuly|Marchjuly]] ([[User talk:Marchjuly|talk]]) 22:30, 1 January 2025 (UTC) |
|||
It would be helpful if someone could give me an example from my attempt, that indicates specifically why it needs a 'major rewrite' https://en.wikipedia.org/wiki/Draft:Digital_Literacy_Coach |
|||
eg, you have written this "...", which is unacceptable because "..."... please? |
|||
I've modelled my post on other Wikipedia articles and asked friends to proofread, and can't see how to move this forward. Having already committed many hours to this, this is now extremely frustrating. :( |
|||
[[User:Mistermchugh|Mistermchugh]] ([[User talk:Mistermchugh|talk]]) 06:46, 22 January 2017 (UTC) |
|||
:Hello {{u|Mistermchugh}} and welcome to Wikipedia! {{oldsmiley|10}} The reason your draft was rejected was primarily one of [[WP:TONE]]. As reviewer {{u|David.moreno72}} wrote: |
|||
:{{Quote|This submission reads more like an essay than an encyclopedia article. Submissions should summarise information in secondary, reliable sources and not contain opinions or original research. Please write about the topic from a neutral point of view in an encyclopedic manner.}} |
|||
:That is good advice. Following the advice on the pages [[WP:NOTESSAY]], [[WP:TONE]], [[WP:MFA]] and [[WP:SMOS]] will help you get your article approved. [[User:Psiĥedelisto|Psiĥedelisto]] ([[User talk:Psiĥedelisto|talk]]) 06:59, 22 January 2017 (UTC) |
|||
:: |
|||
:::Hello, {{u|Mistermchugh}}. Your draft does not read as a neutral encyclopedia article about the topic of a "Digital literacy coach" but rather a review of the literature discussing the topic and also as an advocacy piece more or less arguing that this job title is a good idea. It comes off almost as a manual for implementing the position within an an organization. There are stylistic issues as well. We do not use the formula "Smith and Jones (2005) argue that . . ." Instead, we provide full bibliographic information in footnotes (also called references), rather than shortened bibliographic information in the body of the article. [[User:Cullen328|<b style="color:#070">Cullen</b><sup style="color:#707">328</sup>]] [[User talk:Cullen328|<span style="color:#00F">''Let's discuss it''</span>]] 07:38, 22 January 2017 (UTC) |
|||
== Where is Wikipedia year in review for iOS? == |
|||
:I've already rewritten the article in a neutral tone, this seems like an extremely subjective matter, so I'm asking if someone could illustrate exactly how the tone of my article is inappropriate with a specific example. I've read the pages you've linked to, but I feel the text confirms these expectations. Specific feedback would be far more helpful than generic criticism. |
|||
[[User:Mistermchugh|Mistermchugh]] ([[User talk:Mistermchugh|talk]]) 07:41, 22 January 2017 (UTC) |
|||
I am looking for Wikipedia year in review in iOS where is it? [[Special:Contributions/172.59.25.192|172.59.25.192]] ([[User talk:172.59.25.192|talk]]) 22:18, 1 January 2025 (UTC) |
|||
:Thanks Cullen that's more helpful, I used the text in this Wikipedia article as guide, who use exactly the formula you've described. |
|||
https://en.wikipedia.org/wiki/Formative_assessment |
|||
For example where they say, "Kluger and DeNisi (1996)[26] reviewed over three thousand reports on feedback..." So it's acceptable in that post but not in mine? This is the subjectivity I'm struggling with. |
|||
Again, a specific example would be far less ambiguous and far more productive. |
|||
[[User:Mistermchugh|Mistermchugh]] ([[User talk:Mistermchugh|talk]]) 07:47, 22 January 2017 (UTC) |
|||
::Please see [[WP:OTHERSTUFF]]. The use is just as wrong in that article as it is in yours. Generally, if you want a guide from which to base your article off of, you should use a [[WP:FA|featured article]]. Some new users choose to create articles themselves after having become [[WP:AUTOCONFIRMED|auto confirmed]] rather than via the [[WP:AFC]] process, or articles are simply updated by newer users who don't know the proper way to [[WP:CITE]]; thus some sections of Wikipedia receive more scrutiny than others. But the existence of a problem with one page does not justify the existence of a problem on another. [[User:Psiĥedelisto|Psiĥedelisto]] ([[User talk:Psiĥedelisto|talk]]) 07:52, 22 January 2017 (UTC) |
|||
:::No, it is not acceptable in that article either, {{u|Mistermchugh}}. That article should be copy edited to remove that type of prose. We have well over five million articles and probably one to two million of them have glaring problems and need work. That does not mean that we should add new articles that also have glaring problems. Instead, we want well developed new articles that comply with our core content policies and at least approximate compliance with our [[WP:MOS|Manual of style]]. |
|||
:Welcome to the Teahouse, IP user. This isn't something I'm familiar with, but a Google search yields this link which I hope gives you what you seek: https://www.mediawiki.org/wiki/New_Engagement_Experiments/PES_1.3.1:_Wikipedia_insights. Regards, [[User:Nick Moyes|Nick Moyes]] ([[User talk:Nick Moyes|talk]]) 01:56, 2 January 2025 (UTC) |
|||
:::As for a specific example, consider this sentence: "Attempting to provide technical skills training onsite to teachers in the use of technology has been found to be insufficient, and teaching skills in isolation can be ineffective in ensuring that teachers develop this kind of knowledge, for example, how to use technology to teach content in differentiated ways according to students' learning needs, or technological pedagogical knowledge (TPK); how technology can be used to support the learning of specific curriculum content, or technological content knowledge (TCK); or how to help students meet particular curriculum content standards while using technologies appropriately, (technological pedagogical and content knowledge, or TPACK) in their learning; this is where employing the services of a dedicated DLC has been found to be useful." With all due respect, that sentence is ''excessively'' long and convoluted, uses excessive jargon and acronyms, and is confusing to me even after I have read it several times. Instead, we want neutral, descriptive prose that is concise and clear, and avoids any trace of advocacy. [[User:Cullen328|<b style="color:#070">Cullen</b><sup style="color:#707">328</sup>]] [[User talk:Cullen328|<span style="color:#00F">''Let's discuss it''</span>]] 08:00, 22 January 2017 (UTC) |
|||
== How to find vandalism on Wikipedia? == |
|||
:Thanks Jim, this is much more helpful. Trust me to pick an article as model that is flawed. This gives me the direction I needed, your time and specificity is much appreciated. |
|||
[[User:Mistermchugh|MisterMcHugh]] ([[User talk:Mistermchugh|talk]]) 08:08, 22 January 2017 (UTC) |
|||
The closest thing I have found to vandalism is on the page for the number 3, where someone changed references to “3” with “2.” I’m wondering how to find vandalism, so I can revert it. Most of my edits are fixing grammar. Thank you. [[User:Heyaaaaalol|Heyaaaaalol]] ([[User talk:Heyaaaaalol|talk]]) 00:18, 2 January 2025 (UTC) |
|||
:It seems to me that there's another issue, unrelated to those described above. The writer is too close to the subject, and "can't see the wood for the trees". The article appears to be addressed to readers who already know what a DLC is. many readers won't know, and the article doesn't tell them. For example: |
|||
:@[[User:Heyaaaaalol|Heyaaaaalol]] Welcome to the Teahouse. If you'd care to read [[WP:VANDALISM]], and especially the section linked from this shortcut: ([[WP:SPOTVAN]]), you should have all the answers you need. Monitoring 'Recent Changes' is the best way to spot ongoing vandalism, and you can choose to select only certain types of edits which highlight the most likely problematic edits for you to assess and respond to, and ignore all the honest ones. Thank you for your interest in helping out in this field. Regards, [[User:Nick Moyes|Nick Moyes]] ([[User talk:Nick Moyes|talk]]) 01:50, 2 January 2025 (UTC) |
|||
:The first sentence says that DLCs are employed at schools that have "a 1:1 provision of devices". It says nothing about what kind of device. A fountain pen? A cricket bat? I guess it means a personal computer; if so, the article ''needs to say so''. |
|||
== "Series of small edits" == |
|||
:The second paragraph preaches aboout how schools don't employ enough DLCS. But while the reader has no idea what a DLC does, it is unclear why they should employ any at all. There's a mention of "expensive hardware": maybe the DLC's job is to prevent the pupils from stealing the "devices"? [[User:Maproom|Maproom]] ([[User talk:Maproom|talk]]) 09:54, 22 January 2017 (UTC) |
|||
::I can understand your confusion. While I cannot say with certainty that the duty of a DLC is not to assure that devices aren't stolen, I believe that the main job of the DLC is to help teachers learn about and use the technology that the school is paying for. I agree that this absolutely should be stated in the article. [[User:Psiĥedelisto|Psiĥedelisto]] ([[User talk:Psiĥedelisto|talk]]) 11:41, 22 January 2017 (UTC) |
|||
I find it difficult to follow and review edits for valid constructive contributions when an editor makes a "series of small edits" (example: [https://en.wikipedia.org/enwiki/w/index.php?title=Marty_Makary&diff=prev&oldid=1266013859 1], [https://en.wikipedia.org/enwiki/w/index.php?title=Marty_Makary&diff=prev&oldid=1265997397 2], [https://en.wikipedia.org/enwiki/w/index.php?title=Marty_Makary&diff=prev&oldid=1265999775 3]) where several of the edits could, IMHO, easily and conceivably be made in a single edit with a single edit summary encompassing all of what is edited. Is there policy or guidelines about this practice that could help me in a [[User talk:Llll5032#Marty Makary|talk page discussion]] I am currently engaged in? [[User:Iljhgtn|Iljhgtn]] ([[User talk:Iljhgtn|talk]]) 00:23, 2 January 2025 (UTC) |
|||
Thanks [[User:Maproom|Maproom]] as hard as this is for me to admit, you've probably hit the nail on the head with the 'wood for the trees' comment, now there's way too many idioms in this thread. Part of the reason for wanting this post on Wikipedia is that I'm hoping it will open this up to a wider audience, but for now, despite my arboreal proximity, I'm the only one who is prepared to write this, and also knowledgeable about the role. My hope if it ever gets published is too put out to a wider community to invite a wider discussion. |
|||
:Well, there's [[Wikipedia:CAUTIOUS|WP:Cautious]], which touches on not making too large edits. There is probably better material than this, but I can't find it. |
|||
[[User:Psiĥedelisto|Psiĥedelisto]] you hit the nail on the head, so there must be something in the article that makes sense. [[User:Mistermchugh|MisterMcHugh]] ([[User talk:Mistermchugh|talk]]) 02:18, 28 January 2017 (UTC) |
|||
:The point has been discussed on the Help/Teahouse desks before, and various editors have said that more smaller edits are better than one all-encompassing one, because if only one or two of several small edits are objected to, they can be reverted individually, but if they're part of a larger edit the whole thing has to be reverted and then it becomes difficult to sort out which details are accepted and which disputed. |
|||
[[User:Mistermchugh|MisterMcHugh]] ([[User talk:Mistermchugh|talk]]) 05:09, 24 January 2017 (UTC) |
|||
:I myself, when reading an article for its content, sometimes make small edits as I go along on noticing typos, misspellings, incorrect grammar, etc. If I were to be deliberately setting out to copyedit the article (I used to be a professional editor) I would likely make them in larger batches, perhaps one per Section or Subsection (basically, as often as [edit] is present in the text), because that makes for a smaller and easier chunk (with all its confusing reference codings) to navigate in the edit box. |
|||
:Unfortunately, {{U|Mistermchugh}}, wanting to "open this up to a wider audience" is a bad place to start in writing for Wikipedia. This is a bit paradoxical, because obviously people do write articles specifically because they think they should be there. But [[WP:SOAP|Wikipedia is not a soapbox or means of promotion]]: it reports on topics which have ''already'' been written about. If your purpose is opening up a topic to a wider audience, there are other places which fit that purpose better. --[[User:ColinFine|ColinFine]] ([[User talk:ColinFine|talk]]) 17:25, 27 January 2017 (UTC) |
|||
:In general, I don't think an editor making good-faith edits need think primarily of the convenience of some hypothetical would-be reverter, and I wouldn't criticise anyone for making successive small edits. Were the edits to smell of some deliberate obfustication as a cover for vandalism, or edit-count inflation, it would be another matter. {The poster formerly known as 87.81.230.195} [[Special:Contributions/94.6.84.253|94.6.84.253]] ([[User talk:94.6.84.253|talk]]) 02:11, 2 January 2025 (UTC) |
|||
== Political Party Relations == |
|||
[[User:ColinFine|ColinFine]] We can nitpick about the semantics and sensibilities about what you may or may not feel are my possibly nefarious motivations for creating this post—but let me assure you, I have one motivation, to add to the existing body knowledge; that is it. I have no agenda, I do not stand to benefit personally from this article, I created it because quite frankly I was/am amazed that there isn't already an article dealing with this, which is also more than a little ironic given the nature of the role. So, to clarify, as I see the whole point of Wikipedia in large part about opening up knowledge in general 'to a wider audience' ie not just those who are have paid for access to an online encyclopedia. That's it, is that so hard to accept? [[User:Mistermchugh|MisterMcHugh]] ([[User talk:Mistermchugh|talk]]) 02:18, 28 January 2017 (UTC) |
|||
Just like how we have [[Category:Sign language family tree templates]], should we have that for political parties kind of like below but for whole countries? These would be based on the infobox political party preceded by/succeeded by/merger of/merged into. If so would these be their own article or would they be in the many "List of political parties in ___"? I am curious and want to know if these are wanted/needed. |
|||
== Semi-protected edit request on 26 January 2017 == |
|||
{{Tree chart/start|align=center}} |
|||
{{Tree chart| | | | | | | | | | |SPD ||SPD=[[Social Democratic Party of Germany|SPD]]}} |
|||
{{Tree chart| |,|- |- |- |- |- |- |- |-|-|(|SL=[[Spartacus League]]}} |
|||
{{Tree chart|SL| | | | | | | | |!|SL=[[Spartacus League]]}} |
|||
{{Tree chart| |!| | | | | | | | | |!|SPD=[[Social Democratic Party of Germany|SPD]]}} |
|||
{{Tree chart| KPD |v|-|-|-|-|-|-|-|(| |KPD=[[Communist Party of Germany|KPD]]}} |
|||
{{Tree chart| | | |!| | | | | | | |!| |}} |
|||
{{Tree chart| | | SED | | | | | | |!| |SED=[[Socialist Unity Party of Germany|SED]]}} |
|||
{{Tree chart| | | |!| | | | | | | |!| |}} |
|||
{{Tree chart| | | PDS |-|v|-| WAS |(|PDS=[[Party of Democratic Socialism (Germany)|PDS]]|WAS=[[Labour and Social Justice – The Electoral Alternative|WASG]]}} |
|||
{{Tree chart| | | | | | |!| | | | |!| |}} |
|||
{{Tree chart| | | | | | LIN | | | |!| |LIN=Die Linke}} |
|||
{{Tree chart| | | | | | |!| | | | |!| |}} |
|||
{{Tree chart| | | | BSW |(| | | | |!| |BSW=[[Bündnis Sahra Wagenknecht|BSW]]}} |
|||
{{Tree chart| | | | |!| |!| | | | |!| |}} |
|||
{{Tree chart/end}} [[User:History6042|<span style="color:darkorange">History6042😊</span>]] '''([[User talk:History6042|<span style="color:blue">Contact me</span>]])''' 00:39, 2 January 2025 (UTC) |
|||
:Hi [[User:History6042|History6042]], that's an interesting idea, you might want to pitch this at [[Wikipedia talk:WikiProject Politics]]. [[User:Justiyaya|'''<span style="color:#1d556d">Just</span>''']][[Special:Contributions/Justiyaya|'''<span style="color:#000000">i</span>''']][[User talk:Justiyaya#top|'''<span style="color:#6d351d">yaya</span>''']] 03:23, 2 January 2025 (UTC) |
|||
{{edit semi-protected|Wikipedia:Welcoming committee/Welcome to Wikipedia|answered=y}} |
|||
::Okay, I'll do that, thanks. [[User:History6042|<span style="color:darkorange">History6042😊</span>]] '''([[User talk:History6042|<span style="color:blue">Contact me</span>]])''' 03:27, 2 January 2025 (UTC) |
|||
== Freedom of Panorama Inquiries == |
|||
allee raza |
|||
This is regarding the article [[Liliget Feast House]]. The site where the restaurant once stood was torn down 2008, and I wanted to inquire regarding the best way to navigate [[Fair Use]] and [[Freedom of Panorama]] rules on Wikipedia in order to get photographs of the site on the page. This is probably best asked on Commons, but I'm not familiar with where I might be able to ask on there either. [[User:Ornithoptera|Ornithoptera]] ([[User talk:Ornithoptera|talk]]) 02:19, 2 January 2025 (UTC) |
|||
::{{u|Allee raza}}, please read David Biddulph's reply, just above this comment. |
|||
::{{ping|David Biddulph}} Allee raza won't have been notified of your reply, because you just typed their username without <code><nowiki>[[User:...]]</nowiki></code> or any of the templates, like <code>{{tl|u}}</code> or <code>{{tl|reply to}}</code> (a.k.a. <code>{{tl|ping}}</code>), that would trigger a notification. --[[User:Thnidu|Thnidu]] ([[User talk:Thnidu|talk]]) 08:59, 28 January 2017 (UTC) |
|||
::: {{ping|Thnidu}} If you look at who typed what here, you'll see that I didn't type Allee raza's username here, and you may wish to look at [[User talk:Allee raza]] to see the notification. --[[User:David Biddulph|David Biddulph]] ([[User talk:David Biddulph|talk]]) 18:16, 28 January 2017 (UTC) |
|||
::::{{ping|David Biddulph}} Ooops! I apologize for my misunderstanding here. Clearly, that "semi-protected" process has twists that I don't understand. --[[User:Thnidu|Thnidu]] ([[User talk:Thnidu|talk]]) 08:28, 29 January 2017 (UTC) |
|||
:Hi [[User:Ornithoptera|Ornithoptera]]. I think you can take a picture of the where the restaurant once stood and upload it here, which should be fine under [[Freedom of panorama#Canada]] which also allows incidental inclusion of copyrighted, non-architectural work (also [[commons:COM:FOP Canada]]). You would still be able to release a photograph that you took of publicly viewable buildings under a free license. |
|||
== Url for a citation linking to an archived copy of an article is not working. == |
|||
:If you are asking about whether an inclusion of a [[WP:NFCC|non free file]] is allowed, I'm not sure but I think that it meets all the criteria seeing that the building was demolished and if a suitable free photo cannot be located. [[User:Justiyaya|'''<span style="color:#1d556d">Just</span>''']][[Special:Contributions/Justiyaya|'''<span style="color:#000000">i</span>''']][[User talk:Justiyaya#top|'''<span style="color:#6d351d">yaya</span>''']] 05:48, 2 January 2025 (UTC) |
|||
::Hi {{u|Justiyaya}}, thank you so much for your time! Truthfully I'm uncertain about using a photograph of where the restaurant once stood because it would be irrelevant to the content of the article (or at least for the intended use within the infobox). I am more asking about if I am able to use a non free file, yes, is there a venue where I can have a conversation with individuals who are more familiar with the matter? I'm not entirely sure if there is a relevant noticeboard/talk page/etc where I can bring this matter up with, if you or one of the editors here know one do let me know! [[User:Ornithoptera|Ornithoptera]] ([[User talk:Ornithoptera|talk]]) 05:52, 2 January 2025 (UTC) |
|||
:::@[[User:Ornithoptera|Ornithoptera]] I think it would be a good use of an non-free file. [[Wikipedia:Media copyright questions]] would be a relevant noticeboard for asking these types of questions. Commons won't be the place to go because they don't deal with non-free stuff. Hope this helps and happy editing! :D [[User:Justiyaya|'''<span style="color:#1d556d">Just</span>''']][[Special:Contributions/Justiyaya|'''<span style="color:#000000">i</span>''']][[User talk:Justiyaya#top|'''<span style="color:#6d351d">yaya</span>''']] 06:06, 2 January 2025 (UTC) |
|||
== Why am I blocked from doing stuff on Russian Wikipedia? == |
|||
Source #31 on the Timeline of the Virginia Tech shooting leads to "page not found" when clicked. So I used an archived version from it (archived in 2014) and used the citation that is used for archives. However, it's still leading to "page not found"? |
|||
[[User:Depthburg|Depthburg]] ([[User talk:Depthburg|talk]]) 01:50, 28 January 2017 (UTC) |
|||
According to my global account information I've been indefinitely blocked from doing anything other than viewing the Russian Wikipedia because I'm "using multiple accounts or having others promote me (so what I say is seen more correct)", but I'm not? I don't know anyone else with a Wikipedia account and as far as I know I only have one. It's just on the Russian Wikipedia, none of the others. I haven't really done anything on the Russian Wikipedia anyways. I've only edited my own user page. Why did this happen and how do I fix it? |
|||
:: Hello {{u|Depthburg}} and welcome back to the Teahouse. I see you're still posting questions at the bottom of this page, but I'll answer it down here anyway. |
|||
More information: |
|||
:: It would be a good idea to clean up the URLs that you use in citations so they don't include unnecessary cruft. If the URL contains a question mark some parameters, try removing all of that and checking to see if the url will still work. Also, it's important to get the deadurl= parameter correct so the links will display appropriately. If "yes", then the archive link is presented first. If "no", then the archive-url is just a backup. |
|||
The exact stated reason was обход блокировки (bypass blocking) and it linked [https://ru.m.wikipedia.org/wiki/%D0%92%D0%B8%D0%BA%D0%B8%D0%BF%D0%B5%D0%B4%D0%B8%D1%8F:%D0%9C%D0%BD%D0%BE%D0%B3%D0%BE%D0%BB%D0%B8%D0%BA%D0%BE%D1%81%D1%82%D1%8C here]. It also gave me a link to [https://ru.m.wikipedia.org/enwiki/w/index.php?title=%D0%A1%D0%BB%D1%83%D0%B6%D0%B5%D0%B1%D0%BD%D0%B0%D1%8F:%D0%96%D1%83%D1%80%D0%BD%D0%B0%D0%BB%D1%8B/block&page=User%3A%D0%9A%D1%80%D0%B0%D1%81%D0%BD%D1%8B%D0%B9+%D0%9E%D0%BA%D1%82%D1%8F%D0%B1%D1%80%D1%8C here]. [[User:Красный Октябрь|Красный Октябрь]] ([[User talk:Красный Октябрь|talk]]) 04:25, 2 January 2025 (UTC) |
|||
:Hi @[[User:Красный Октябрь|Красный Октябрь]], welcome to the teahouse. English and Russian Wikipedia are moderated differently, so a block there should not affect editing here nor could the community here fix anything there. From what I've been able to see with google translate and all, it appears that you've been checkuser blocked by @{{u|Q-bit array}} without email or talk page access. The administrator appears to monitor cross wiki notifications so hopefully they would respond here. They also appear to monitor [[:meta:User talk:Q-bit array]]. Usually appeals without talk page access go through [[WP:UTRS]] but I can't seem to find a similar system in ru-wiki. [[User:Justiyaya|'''<span style="color:#1d556d">Just</span>''']][[Special:Contributions/Justiyaya|'''<span style="color:#000000">i</span>''']][[User talk:Justiyaya#top|'''<span style="color:#6d351d">yaya</span>''']] 05:34, 2 January 2025 (UTC) |
|||
:: As of your last [https://en.wikipedia.org/enwiki/w/index.php?title=Timeline_of_the_Virginia_Tech_shooting&diff=next&oldid=762321034 edit] to that article [[Timeline of the Virginia Tech shooting]], both the archiveurl and original url are the same, pointing to 404-space on Computerworld. The previous version looked to me like it was working, except for the extra url cruft and the wrong sense of dead-url. I suggest fixing these two things to see if you don't get something you're happy with. [[User:jmcgnh|<b><span style="color:#248F7D"> —jmcgnh</span></b>]]<sup><small><b>[[User_talk:jmcgnh|<span style="color:#58D582">(talk)</span>]] [[Special:Contributions/jmcgnh|<span style="color:#8F7D24">(contribs)</span>]]</b></small></sup> 09:15, 28 January 2017 (UTC) |
|||
== Notice of geological updates == |
|||
:::Removing the unnecessary parameters and providing "deadurl" a "yes" helped, thank you. [[User:Depthburg|Depthburg]] ([[User talk:Depthburg|talk]]) 09:43, 28 January 2017 (UTC) |
|||
Hi, in the past few months, I was working on pages regarding the periods, ages and other units of the geological timescale, and noticed a [https://stratigraphy.org/ICSchart/ChronostratChart2024-12.pdf recent update in the ICC Chronostratigraphic Chart (2024/12)], compared to the [https://stratigraphy.org/ICSchart/ChronostratChart2023-09.pdf previous version (2023/09)], that affects the time boundaries of a lot of ages. I didn't know which talk page or portal I could specifically notify of this change, so I came here to speak about it. |
|||
==Can I write an article about myself?== |
|||
I want to write an article about myself. <!-- Template:Unsigned --><small class="autosigned">— Preceding [[Wikipedia:Signatures|unsigned]] comment added by [[User:Prashant Raut|Prashant Raut]] ([[User talk:Prashant Raut#top|talk]] • [[Special:Contributions/Prashant Raut|contribs]]) 17:07, 28 January 2017 (UTC)</small> <!--Autosigned by SineBot--> |
|||
Now, I don't want to linger on a change like this here, since that feels like advertising which I KNOW Wikipedia is not about (see |
|||
: Welcome to the Teahouse, {{u|Prashant Raut}}. |
|||
[[WP:SOAPBOX]]). But I know a change like this is going to require a lot of edits to catch up with, and I for sure can't do it alone. So does anyone know a good project/portal page I can discuss this more on? — [[User:Alex26337|Alex26337]] ([[User talk:Alex26337|talk]]) 05:05, 2 January 2025 (UTC) |
|||
: |
:Hi [[User:Alex26337|Alex26337]], thank you for your question. [[Wikipedia talk:WikiProject Geology]] would be the best venue to discuss this. I don't think this is soapbox like if you are advertising on-Wiki collaboration. The policy refers to more so advertising on Wiki things off-wiki. Discussions with other community members that share the same interest is what these Wikiprojects are for :D [[User:Justiyaya|'''<span style="color:#1d556d">Just</span>''']][[Special:Contributions/Justiyaya|'''<span style="color:#000000">i</span>''']][[User talk:Justiyaya#top|'''<span style="color:#6d351d">yaya</span>''']] 05:58, 2 January 2025 (UTC) |
Latest revision as of 06:55, 2 January 2025
Stormy clouds, a Teahouse host
Your go-to place for friendly help with using and editing Wikipedia.
Note: Newer questions appear at the bottom of the Teahouse. Completed questions are archived within 2–3 days.
Technical question about the long hyphen
Hi!
I've been editing the timeline of Polermo where the long hyphen dominates, but I can't seem to generate one.Typing a regular hyphen, gives me just that - a regular hyphen, typing two hyphens gives me two hyphens (--) and trying to make one through the keboard shortcut which I found on internet forums (Alt+0151), just gives me one that's too long (—). So far I've been copying and pasting existing long hyphens which is kind of annoying, does anyone have any better solutions?
Thanks! Moonshane1933 (talk) 14:38, 22 December 2024 (UTC)
- Hello, @Moonshane1933. I think you're talking about an em-dash. See MOS:EMDASH ColinFine (talk) 14:52, 22 December 2024 (UTC)
- Yes! That's what I meant! Thank you! Moonshane1933 (talk) 15:15, 22 December 2024 (UTC)
- I don't think you could find a better character in "unicode table".
- This "article" is listing the most common characters.
- There are also the "Unicode block" entry on Wikipedia that can be maybe helpful. Anatole-berthe (talk) 14:54, 22 December 2024 (UTC)
- Excellent. Thank you too! Moonshane1933 (talk) 15:16, 22 December 2024 (UTC)
- I don't think ressources I shared with you will help you but I hope it will. Anatole-berthe (talk) 15:45, 22 December 2024 (UTC)
- Excellent. Thank you too! Moonshane1933 (talk) 15:16, 22 December 2024 (UTC)
- Ignoring the Minus sign, there are three 'horizontal line' characters most commonly used in text, the hyphen, the N-dash and the M-dash. There are various ways to insert the latter two; usually I do so with [alt]+0150 and [alt]+0151. Despite being a former professional book editor, I have not previously encountered a "long hyphen" (a term not found anywhere in Wikipedia). Note that the lengths of all these characters may look different in different typefaces: I suspect your "long hyphen" is an N-dash. [Apologies for semi-overlap with answers above.] {The poster formerly known as 897.81.230.195} 94.1.223.204 (talk) 17:00, 22 December 2024 (UTC)
- @Moonshane1933 If you use the source editor, which you can do even if you mainly edit with the visual editor, you'll find that the N-dash and M-dash appear at the foot of the editing window, where you can click on them to insert them into text. Other useful tags like <ref></ref> are also available with a single click. Mike Turnbull (talk) 14:11, 24 December 2024 (UTC)
- OOOOOOOHHHH... THANK YOU! That makes life easier! I hadn't even thought of looking at the source editor, because it always looks headache inducing to me. I'll give it a try. Thank you so much. Moonshane1933 (talk) 13:07, 25 December 2024 (UTC)
- Yes, well, the "long hyphen" is a term that I coined, simply because I lacked the knowledge of its correct name, So I would have been very surprised if it had appeared in Wikipedia. Anyway, thank you, oh mysterious IP poster, I hope our paths cross again! Moonshane1933 (talk) 13:03, 25 December 2024 (UTC)
- @Moonshane1933, some Christmas goodies for you:
- — Merriam-Webster Dictionary has a nice clear explanation about the both kinds of dashes and the hyphen, with good examples.
- — The way the two kindts of dashes is written is em-dash (for —) and en-dash ( for – ), even though we pronounce the terms "M dash" and "N dash."
- — Why these terns? Because the em-dash is exactly the width of capital M and the en-dash is exactly the width of capital N.
- — If you have a Macintosh, there's a real simple way to make the dashes: the em-dash by pressing Control Option Hyphen at the same time, and the en-dash by pressing Option Hyphen at the same time.
- —Did you notice how Nick Moyes creatively renamed Dasher, one of Santa Claus's eight reindeer, in his "Seasonal Greetings from all at the Teahouse" post to fellow editors below?
- —You may be pleased to know that I found an online reference to a "long hyphen." So, then, you weren't completely alone in doing that. But as 94.1.223.204 commented above, in professional editing we just don't use it. Like ColinFine, )I think anyone who did say "long hyphen" would probably be thinking of the em-dash; though I also think what 94.1.223.204 said above is also technically correct, that the term would have to refer to the en-dash (that's the next size up for a hyphen, after all). Augnablik (talk) 06:08, 27 December 2024 (UTC)
- @Augnablik, What a great reply! I thoroughly enjoyed every bit of it! And I learned a lot (not to detract from the other contributors, each of whom taught me something new - thanks, everybody) - a special thank you for the meanings of the em-dash and en-dash (I love that type of thing), and for drawing my attention to Nick Moyes' "Seasonal Greetings", and of course for finding me an ally in calling the en-dash a "Long hyphen" (don't worry, now that I know the correct terminology I will use it and hopefully amaze my friends...). Thank you again and Merry Christmas! Moonshane1933 (talk) 12:58, 29 December 2024 (UTC)
- @Augnablik Not to be a naysayer, but I think the bit about em dashes being named for being 'M' width is a false etymology. I too would have loved if it were true, but I think it's actually based on the em unit as described in Dash#Em dash. — Kilvin the Futz-y Enterovirus (talk) 10:23, 30 December 2024 (UTC)
- Well, @Kilvin the Futz-y Enterovirus, not to be a counter-naysayer (!) but there are many online sites with support for the width of the em-dash equal to M and of the en-dash equal to N. Here’s just one, offered by Grammarly. (Scroll down to What Is an En-Dash? and What Is an Em-Dash?) Augnablik (talk) 12:06, 30 December 2024 (UTC)
- Hm... not to be a counter- counter- naysayer I mean I don't really think 'many online sites' and Grammarly (unsourced, could just be mirroring untrustworthy sources) are WP:RS. But you're right; my searches show similar results and don't really yield great answers to this matter (ideally there'd be a page like "Many people say that an em dash is named for X, but actually it is Y, and this misconception came from Z" linking to many reliable sources). I guess my personal bias is towards the people going "false etymology!" that acknowledge both X and Y rather than one group who simply assert the "fun" explanation with seemingly no awareness of the other explanation. I mean, as linked previously, the em and en are typographical units whose pages describe their origins, even addressing Em (typography)#Obsolete alternative definition. — Kilvin the Futz-y Enterovirus (talk) 22:53, 30 December 2024 (UTC)
- Oooh, @Kilvin the Futz-y Enterovirus, you’re acknowledging “personal bias” towards “false etymology?” That requires a declaration of COI! 🙂 Augnablik (talk) 18:09, 31 December 2024 (UTC)
- Aha! I found something! In the Historical thesaurus on the OED site. And I quote:
- "em dash1836- A long dash —, originally and usually the width of one em (see em, n.)." Moonshane1933 (talk) 13:13, 1 January 2025 (UTC)
- Oooh, @Kilvin the Futz-y Enterovirus, you’re acknowledging “personal bias” towards “false etymology?” That requires a declaration of COI! 🙂 Augnablik (talk) 18:09, 31 December 2024 (UTC)
- Hm... not to be a counter- counter- naysayer I mean I don't really think 'many online sites' and Grammarly (unsourced, could just be mirroring untrustworthy sources) are WP:RS. But you're right; my searches show similar results and don't really yield great answers to this matter (ideally there'd be a page like "Many people say that an em dash is named for X, but actually it is Y, and this misconception came from Z" linking to many reliable sources). I guess my personal bias is towards the people going "false etymology!" that acknowledge both X and Y rather than one group who simply assert the "fun" explanation with seemingly no awareness of the other explanation. I mean, as linked previously, the em and en are typographical units whose pages describe their origins, even addressing Em (typography)#Obsolete alternative definition. — Kilvin the Futz-y Enterovirus (talk) 22:53, 30 December 2024 (UTC)
- Well, @Kilvin the Futz-y Enterovirus, not to be a counter-naysayer (!) but there are many online sites with support for the width of the em-dash equal to M and of the en-dash equal to N. Here’s just one, offered by Grammarly. (Scroll down to What Is an En-Dash? and What Is an Em-Dash?) Augnablik (talk) 12:06, 30 December 2024 (UTC)
- @Moonshane1933 If you use the source editor, which you can do even if you mainly edit with the visual editor, you'll find that the N-dash and M-dash appear at the foot of the editing window, where you can click on them to insert them into text. Other useful tags like <ref></ref> are also available with a single click. Mike Turnbull (talk) 14:11, 24 December 2024 (UTC)
Add a page?
Hello - How can somebody submit a page for a notable person? My husband has one of the country's worst wrongful convictions in the United States and I'd love to have somebody neutral put information up regarding his wrongful conviction case. We believe he will be exonerated someday. His name is Temujin Kensu and you can google search his name to learn more about this horrible case. Thank you! 65.111.210.82 (talk) 06:21, 28 December 2024 (UTC)
- Based on my Google search, I consider it almost certain that Temujin Kensu is notable and that Wikipedia ought to have an article about him. Cullen328 (talk) 07:39, 28 December 2024 (UTC)
- For anyone interested in starting a draft some of these Google hits could easily be used to pass WP:GNG. Mike Turnbull (talk) 12:21, 28 December 2024 (UTC)
- Realistically, you may not get a volunteer. Teahouse Hosts volunteer here to advise, not to be authors or co-authors. David notMD (talk) 20:18, 28 December 2024 (UTC)
- For anyone interested in starting a draft some of these Google hits could easily be used to pass WP:GNG. Mike Turnbull (talk) 12:21, 28 December 2024 (UTC)
- If your husband is notable enough, he will be talked about. SimpleSubCubicGraph (talk) 05:37, 1 January 2025 (UTC)
- IP editor: I have now started to draft an article at Draft:Temujin Kensu. In view of your conflict of interest it would be best if you did not edit it directly (although that's not forbidden while it remains a draft). I would welcome your suggestions for additions and corrections at Draft talk:Temujin Kensu, especially where you can provide additional published sources I can use. Mike Turnbull (talk) 12:47, 1 January 2025 (UTC)
Isotopes lists download
Is there a to download these lists ? ( For example : the list in "https://en.wikipedia.org/wiki/Isotopes_of_sodium" )
I have written a c# application that describes the relations between elements, isotopes, decays, fusions ... etc.(originating from the question "Where the carbon atoms in the cafeine in your coffee come from ?")
When you make normal modifications to these lists, it takes me about 2 weeks to refresh my database for over 3000 isotopes and 5000 decays coming from 118 pages (and subject to typing errors...)
I have tried to download one of these pages but I get one of these mumbo-jumbo network message ( about security and the correction looks like "set the web_client.Tchic_Tchac to Fling_Flang" ... and none of them works... )
Do you have a suggestion ?
Thank you very much Michel Béliveau (talk) 17:49, 28 December 2024 (UTC)
- Describing a message as a "mumbo-jumbo network message" is not very helpful in determining what your problem is. If you quote the error message exactly it might be more useful. In any case, I can successfully download articles using curl like this: CodeTalker (talk) 19:52, 28 December 2024 (UTC)
curl -k https://en.wikipedia.org/wiki/Isotopes_of_sodium
- Thank you for this fast (and good) answer.
- The mumbo-jumbo error message was : "The request was aborted: Could not create SSL/TLS secure channel." I was not using the good approach to download the content of the web page.
- >>> However the CURL function does what I need.
- Here is want to do In my application :
- For each Element ( 118 !!! ) get the "List of Isotopes" for this Element. Then for each Isotope : get its mass, half-life, decay mode(s) and decay product(s). This yields for over 3500 isotopes and over 4500 decays. Refreshing the data took quite a long time.
- Analyzing the results of the curl command is not so hard and will eliminate typing mistakes. Even if I need a few days to program the analysis, it will be faster than re-typing the data.
- I will take a look at Wikidata.
- Thanks again. Michel Béliveau (talk) 23:02, 28 December 2024 (UTC)
- Without really understanding what you are trying to do, I would suspect that Wikidata was a more useful resource than Wikipedia for your purpose, as it is a database which contains relations between its elements. ColinFine (talk) 21:34, 28 December 2024 (UTC)
- @Michel Béliveau Wouldn't it be easier to download from the original sources, for example NUBASE? Mike Turnbull (talk) 15:48, 29 December 2024 (UTC)
- Thank you Mr. Turnbull.
- You are correct. It would be easier to download from the original sources.
- I have found (and used) a NUBASE file (namely for nucleus values) . So far, I have found only 1 NUBASE file that I could use ( coming from "The Ame2020 atomic mass evaluation (I)" by W.J.Huang, M.Wang, F.G.Kondev, G.Audi and S.Naimi - Chinese Physics C45, 030002, March 2021) .
- The purpose of my request to Wikipedia is to avoid re-typing the values. The NUBASE file allowed this.
- Do you know other NUBASE files ? Or other sources ? (I also found some data in PeriodicTable.com)
- Thanks again for your interest Michel Béliveau (talk) 19:19, 29 December 2024 (UTC)
- @Michel Béliveau Wouldn't it be easier to download from the original sources, for example NUBASE? Mike Turnbull (talk) 15:48, 29 December 2024 (UTC)
To editor Michel Béliveau: Hello! This kind of question is something you might want to take to the Computing reference desk, where you can get people with computing expertise to assist. --Slowking Man (talk) 20:39, 31 December 2024 (UTC)
Locked out of account
I got locked out of my DooplissTTYD account because I forgot the complex password and didn’t have an email address linked to it. Is there any way that account can be renamed to something else and I change this one to DooplissTTYD? TTYDDoopliss (talk) 21:50, 28 December 2024 (UTC)
- You would have to request a renaming from the account you want renamed. It can't be requested by in essence a third party(as we have no way to know who is on the other end of the computer). The best you can do is post on your current and previous user pages that you lost access to your old account and have a new one. 331dot (talk) 00:06, 29 December 2024 (UTC)
- This is why the first thing you want to ensure you do right now is verify an e-mail address for your account, so you can restore it: follow this link if you haven't. Step 2: I highly recommend using a password manager to save passwords—as well as generate secure passwords. I suggest trying out Bitwarden.
- Now in this particular case, your prior account has only a few edits; this means if you wait a bit, say 6 months, you can request to "usurp" it which will accomplish what you want. (Also, a very good game.) --Slowking Man (talk) 18:06, 31 December 2024 (UTC)
- I mean that’s what I did. I generated a password using a feature on my phone, and it kept saying the one I had on my phone was incorrect.
- and yes I have an email linked to this account now. 6 months is a very long time though… TTYDDoopliss (talk) 19:13, 31 December 2024 (UTC)
donating - would like to donate
Hail... Would like to donate 50 quid to the cause but stop at the name and address part. Don't really see the need for full name and address. Just old and not particularly wise. Any suggestions? 81.96.25.61 (talk) 12:28, 29 December 2024 (UTC)
- Hi IP editor, the Wikimedia Foundation deals with all donation issues and questions - editors here at the Teahouse don't have any input. Please direct your query to the email address at the bottom of donate.wikimedia.org/wiki/Ways_to_Give qcne (talk) 12:49, 29 December 2024 (UTC)
- Just to elaborate slightly: all editors here give their time entirely voluntarily and gain absolutely no financial benefit from any contributions made to keep the broader Wikipedia projects going. So we have little knowledge of how the donation systems work - despite being grateful for everyone's contributions. The advice above is sound. Nick Moyes (talk) 18:21, 29 December 2024 (UTC)
- Also I'm not certain (not WMF personnel), but I am pretty confident the reason for requiring that info is fraud protection and banking and money laundering laws; it's not something the Foundation has any control over. WMF is a registered 501(c)(3) organization in the US, meaning it has to comply with a bunch of laws and regulations. (For furrin types, this is near-synonymous with "non-profit/charity" in the US, being simply the section of the Internal Revenue Code setting out the criteria said organizations have to meet to get tax exemption.) Note, a way that one can make semi-anonymous donations, is to pass them through what's called a "donor-advised fund" in the US, but this is a bit more involved and requires going through a bank or brokerage that will arrange your transfer. Alternatively, I suppose if one is accepting of the fact there are no guarantees in life one could always send cash to the WMF's front door with no return address and leave its final disposition up to them. --Slowking Man (talk) 18:38, 31 December 2024 (UTC)
I don't find the ressource to add books wrote by someone
Hello ! I'd like to add a book on the article about "David Murphy (CIA)".
This is a book wrote by him not mentionned in the article.
I don't find the ressource explaining how to add the bibliography of someone. Anatole-berthe (talk) 16:17, 29 December 2024 (UTC)
- @Anatole-berthe The standard template to use for book citations is {{cite book}}. It is usual, but not essential, to use its |URL= parameter to link to Google Books for the convenience of our readers. In this case that would be this link, from which you can also find the ISBN and full list of authors. Mike Turnbull (talk) 16:27, 29 December 2024 (UTC)
- Hello, @Anatole-berthe. Why do you want to add that book to that article? Has the book been discussed by independent sources? If not, why is it significant ednough to feature in a Wikipedia article?
- More to the point, that article is woefully short of sources, and does not establish that Murphy meets the criteria for notability. ColinFine (talk) 17:42, 29 December 2024 (UTC)
- The book was wrote by himself as I said.
- The ISBN-10 is "0300107803" and the ISBN-13 is "978-0300107807" for the first edition.
- The title is "What Stalin Knew: The Enigma of Barbarossa" , it was published by "Yale University Press" in 2005.
- The ISBN-10 is "030011981X" and the ISBN-13 is "978-0300119817" for the second edition.
- The second edition was published in 2006 by the same publisher.
- In this edition , it's mentionned the review of the first edition by "Niall Ferguson" for "New York Time Book Review".
- The first edition was also reviewed by "Andrew Nagorski" for "The Weekly Standard".
- There are also a review by "Henry Kissinger" but I consider it doesn't count to add this book on Wikipedia.
- This person have to be considered as a reviewer among others even if he's notable accorded to criterias.
- He didn't wrote the review for a magazine or a journal and therefore the fact "Kissinger" wrote a review should not be taken into account. This is what I think.
- The same for the review by "Donald Kagan" for "Yale University" for a particular reason. This person worked for "Yale" and the book was published by "Yale University Press". Therefore , I consider it's like a review by the publisher itself.
- Conclusion : I think this book met the criteria n°1 for "Wikipedia:Notability_(books)" to create an article on this book.
- I don't want to create an article on the book itself.
- The "Threshold standards" is met because "Library of Congress" catalogued this book.
- LCCN is "2004065916".
- If it is considered as notable to create an article , I consider it is notable enough to mention this book on the article "David Murphy (CIA)". Anatole-berthe (talk) 12:03, 1 January 2025 (UTC)
- The book was wrote by himself as I said.
can you give me a lnk of the Roblox page
i need Roblox link to sign in 24.192.134.19 (talk) 01:57, 30 December 2024 (UTC)
- Welcome to the Teahouse. Did you have a question about editing or using Wikipedia? —Tenryuu 🐲 ( 💬 • 📝 ) 02:07, 30 December 2024 (UTC)
- @Tenryuu I think hes trolling. SimpleSubCubicGraph (talk) 02:50, 30 December 2024 (UTC)
- That's my standard reply to anyone who doesn't ask questions about Wikipedia here. —Tenryuu 🐲 ( 💬 • 📝 ) 02:58, 30 December 2024 (UTC)
- @Tenryuu I think hes trolling. SimpleSubCubicGraph (talk) 02:50, 30 December 2024 (UTC)
- Does this link help at all? If you have general computing questions that aren't Wikipedia-specific, go to the Computing reference desk. --Slowking Man (talk) 18:46, 31 December 2024 (UTC)
Good Sources for Articles
G-Day
I wanted to ask if there is any reliable source for Articles. I cannot afford my personal favourite, The "Britannica" Encyclopedia, since they seem to be rare and expensive. Thank you. (I know this isn't about a specific Article but I need to know this) PizzaFrank (talk) 13:51, 30 December 2024 (UTC)
- @PizzaFrank there are all sorts of reliable sources, not just Encyclopædia Britannica, and most don't require a purchase of any kind. See Wikipedia:Reliable sources for a detailed overview of RSes here on Wikipedia. '''[[User:CanonNi]]''' (talk • contribs) 13:55, 30 December 2024 (UTC)
- Thank you, I will check that out. PizzaFrank (talk) 14:03, 30 December 2024 (UTC)
- PizzaFrank Note that you don't have to personally own the source- it just needs to be publicly accessible, like online, or in a library. 331dot (talk) 14:01, 30 December 2024 (UTC)
- Understood Thank you for the help. PizzaFrank (talk) 14:04, 30 December 2024 (UTC)
- PizzaFrank I have used https://www.britannica.com a few times, and there is no fee to use the online version of Encyclopedia Britannica. Karenthewriter (talk) 18:27, 30 December 2024 (UTC)
- Understood Thank you for the help. PizzaFrank (talk) 14:04, 30 December 2024 (UTC)
- And besides public libraries which are great, Wikipedia editors with some experience can access The Wikipedia Library which grants free access to tons of "paywalled" "digital content"! Pretty neat! Also see there: even if your account is not "seasoned" enough yet, you can request experienced editors assist you with researching sources through it. --Slowking Man (talk) 17:45, 1 January 2025 (UTC)
Photograph attribution
I want to use a portrait of my grandfather taken in 1915 by a company that no longer exists. I can show an attribution in the caption, but there is nobody I can seek permission from. How do I proceed, please? Gangnam Woodford (talk) 16:52, 30 December 2024 (UTC)
- Hello. It depends on the laws of your country, but a photo taken in 1915 is likely in the public domain. According to https://commons.wikimedia.org/wiki/Commons:Copyright_rules_by_territory/United_States in the US anything published before 1929 is public domain(other than sound recordings), so it would just depend on your own country. 331dot (talk) 16:56, 30 December 2024 (UTC)
- Yes BUT (big but!), "publication" in the context of US copyright law has a specifical technical, legal definition. It doesn't mean "creation" or even "giving a copy to a particular person". Hirtle chart. A photograph of a private individual is almost certainly an "unpublished work" under the US copyright meaning of that term, which matters significantly for the length of its copyright term especially if an "older" work. Copyright is a subject that gets quite technical, so if not familiar and experienced with copyright issues, it is best to hesitate to advise others before researching things in detail and double-checking, and if in doubt, direct others to forums like Commons where experts can be found. --Slowking Man (talk) 20:02, 31 December 2024 (UTC)
- @Gangnam Woodford Is this an American picture? If so, you can upload it as public domain, see [1]. Gråbergs Gråa Sång (talk) 16:57, 30 December 2024 (UTC)
- Sorry, should have said: UK. Gangnam Woodford (talk) 16:58, 30 December 2024 (UTC)
- @Gangnam Woodford Still good, see this example: File:Archibald Joyce.jpg. Gråbergs Gråa Sång (talk) 17:02, 30 December 2024 (UTC)
- Thank you! Gangnam Woodford (talk) 17:06, 30 December 2024 (UTC)
- Please also see MOS:CREDITS ".... do not credit the image author or copyright holder in the article. .... as long as the appropriate credit is on the image description page." - Arjayay (talk) 17:16, 30 December 2024 (UTC)
- Thank you! Gangnam Woodford (talk) 17:06, 30 December 2024 (UTC)
- @Gangnam Woodford Still good, see this example: File:Archibald Joyce.jpg. Gråbergs Gråa Sång (talk) 17:02, 30 December 2024 (UTC)
- Sorry, should have said: UK. Gangnam Woodford (talk) 16:58, 30 December 2024 (UTC)
- To editor Gangnam Woodford: "Congratulations", you've just tripped onto one of the fun little copyright law landmines that are out there (alternately: "job security for lawyers"). Everything on Wikimedia Commons must be public domain in both the US, and origin country. This means you have to care about US copyright law and all its nuances: see Hirtle chart. Unless the picture was like put on display in an art gallery or something, it is an "unpublished" work and therefore still under copyright in the US until 2035! (120 yrs from creation date) Really! Welcome to copyright law!
- Here are the practical implications for you in this case: Is this image going to be used in an article? If so, upload it here on enwp (not Commons) by following this link, and tag it {{PD-UK-unknown}}, as well as adding a fair use rationale since it's still under US copyright.
- Add it to the article(s), nothing more needed—some bot should automatically transport it over to Commons when it finally lapses into PD-US. However, if you just, say, want to use the image on your user page, I'm afraid you aren't going to be able to for 10 more years. So stick a pin in the calendar for that. --Slowking Man (talk) 19:54, 31 December 2024 (UTC)
An Article About CK
Extended content
|
---|
I was asked by christen kuikoua representation to write an article about him and it was declined saying I didn't correctly reference it. Please if anyone wishes to jump in and help me with Will appreciate it Draft:Christen Kuikoua Silvernet123 (talk) 18:27, 30 December 2024 (UTC)
|
- Note that the account Christenkofficial has (correctly or not, I'm just reporting) now been blocked as a sockpuppet of Silvernet123, per Wikipedia:Sockpuppet investigations/Silvernet123. {The poster formerly known as 87.81.230.195} 94.1.223.204 (talk) 03:35, 31 December 2024 (UTC)
How to add sources
I need a simplified tutorial in how to add the number reference and the cite the source. M. Chris Tucker (talk) 19:02, 30 December 2024 (UTC)
- Hello M. Chris Tucker (talk · contribs), might I suggest Referencing For Beginners. -- D'n'B-📞 -- 19:06, 30 December 2024 (UTC)
- Hi @M. Chris Tucker and welcome to Wikipedia! The simplest way to add references is, directly after the sentence or paragraph that your source supports, add <ref>, followed by the text of your reference, followed by </ref>. The software will sort out the numbering for you. Others here should be able to give you more detailed advice, and the guide that D'n'B linked above looks like a great place to learn more. Thanks for your additions to Edward Dickson (Canadian politician)! Best, Wham2001 (talk) 19:07, 30 December 2024 (UTC)
- 2607:FEA8:7D00:95A0:FDD3:6EB9:68D5:D6D7 (talk) 00:38, 31 December 2024 (UTC)
- Thanks very much! I will follow up on your suggestion and wish you a Happy New Year. M. Chris Tucker (talk) 00:42, 31 December 2024 (UTC)
- 2607:FEA8:7D00:95A0:FDD3:6EB9:68D5:D6D7 (talk) 00:38, 31 December 2024 (UTC)
How do I add a "Main Article" to Wikipedia
I updated our "Cornhusker Council" section under Boy Scouts of America, Nebraska Scouting. and would like to create a main article for our council. how do I do that?
Cornhusker324 (talk) 21:15, 30 December 2024 (UTC)
- Cornhusker324 Hello and welcome to the Teahouse. First, please see conflict of interest, as that needs to be formally disclosed. For a standalone article about your specific council, you would need to show with significant coverage in independent reliable sources that your council meets the special Wikipedia definition of a notable organization. That coverage can't just be the reporting of its routine activities, but in depth coverage as to what makes your council important/significant/influential. Be aware that writing a new article is the most difficult task to attempt on Wikipedia, and it's even harder with a conflict of interest. Also be aware that an article is not necessarily desirable. 331dot (talk) 21:23, 30 December 2024 (UTC)
- Hello @Cornhusker324. I have just reverted your edits in accordance with our conflict of interest policy. Please request specific edits on the article's respective talk page. Thanks. Tarlby (t) (c) 21:25, 30 December 2024 (UTC)
- And I'm not sure why the council itself would merit a standalone article from the article about scouting in Nebraska. 331dot (talk) 21:31, 30 December 2024 (UTC)
- Many other Councils already have a standalone article. I would think this would be supported for consistency sake. Cornhusker324 (talk) 21:35, 30 December 2024 (UTC)
- @Cornhusker324, please see WP:42. Some council have articles. Others don't, and maybe they shouldn't. It all depends on the reliable sources that we need to use so that an article can be made. An article on Cornhusker Council won't be made if there are no such sources. Tarlby (t) (c) 21:40, 30 December 2024 (UTC)
- Many other Councils already have a standalone article. I would think this would be supported for consistency sake. Cornhusker324 (talk) 21:35, 30 December 2024 (UTC)
- I have changed my user name in hopes of complying with your COI policy. The stand-alone article seems too difficult at this point. Perhaps in the future.
- As such, I would simply like to update our section, "Cornhusker Council," in "Scouting in Nebraska." and will attempt that in the 'talk page,' as you suggested. Cornhusker324 (talk) 21:42, 30 December 2024 (UTC)
- And I'm not sure why the council itself would merit a standalone article from the article about scouting in Nebraska. 331dot (talk) 21:31, 30 December 2024 (UTC)
- Hello @Cornhusker324. I have just reverted your edits in accordance with our conflict of interest policy. Please request specific edits on the article's respective talk page. Thanks. Tarlby (t) (c) 21:25, 30 December 2024 (UTC)
Your name change does not show up yet, and regardless of a name change, you still have a COI. I see that you are proposing changes on the Talk page of the article, but much of what you wrote is not referenced. See Help:Referencing for beginners to understand how to insert inline refs. David notMD (talk) 05:30, 31 December 2024 (UTC)
- I provided four references which is more than what is currently listed. Those currently used are not authoritative and factually incorrect. 209.92.187.50 (talk) 15:34, 31 December 2024 (UTC)
- How exactly did you change your username? It should be done via either Special:GlobalRenameRequest or WP:CHUS. You should continue to use your original username until it is changed, at which time you can begin using the new name. 331dot (talk) 15:48, 31 December 2024 (UTC)
- Never mind, I see it is changed now. 331dot (talk) 15:49, 31 December 2024 (UTC)
- That is not how references are created. Also, you now appear to be editing as an IP address, i.e., not logged into an account versus a name-change. David notMD (talk) 16:54, 31 December 2024 (UTC)
- I've used two different means to source the information I provided, both of which were recommended by this site and/or a moderator. CCJLJ (talk) 17:04, 31 December 2024 (UTC)
- Again, see Help:Referencing for beginners to understand how to insert inline refs. What you posted on the Talk page of the article (below) are not properly formatted references. David notMD (talk) 21:22, 31 December 2024 (UTC)
- I've used two different means to source the information I provided, both of which were recommended by this site and/or a moderator. CCJLJ (talk) 17:04, 31 December 2024 (UTC)
- That is not how references are created. Also, you now appear to be editing as an IP address, i.e., not logged into an account versus a name-change. David notMD (talk) 16:54, 31 December 2024 (UTC)
References supporting change:
- Salistean, John, "A History of the Cornhusker Council 1940-1975," Houchen Bindery LTD of Utica, Nebraska, 1st ed., 2011.
- Golden Sun Lodge Website, www.goldensunlodge.org
- Cornhusker Council Website, www.cornhuskercouncil.org
- Brown, Elinor L., "History of Lancaster County, Then and Now," ASIN B0006CJTC2, Jan 1971.
Notability and Independence.
My grandmother, Ethel Margaret Streit Harrison, was the first woman elected as Clerk of the Montana Supreme Court, one of the founders of the Montana Association of Female Executives and one of the original board members of the Holter Museum of Art. There are printed sources that talk about her achievements but according to wikipedia the only thing notable about her was that she was married to John C. Harrison (judge). I understand that, as a relative, there is a potential conflict here, but I think it is important her contributions are documented on something other than microfiche. Any thoughts? / Thank you :) Mehap dwhx (talk) 22:40, 30 December 2024 (UTC)
- Mehap dwhx, none of those three roles that you describe automatically confers Wikipedia-defined "notability" on a person. And the sum of the three doesn't either. But it's possible that she's "notable" all the same. Regardless of your conflict of interest, you're free to create Draft:Ethel Margaret Streit Harrison. If this demonstrates her notability and follows Wikipedia's other policies, it will be promoted to an article (possibly with a slightly different title), whereupon you shouldn't continue to edit it but would be free to make suggestions and requests on its talk page. -- Hoary (talk) 23:12, 30 December 2024 (UTC)
- To expand an aspect of Hoary's excellent advice above: in Wikipedia, 'Notability' boils down to "is there enough substantial material, published independently of the subject, in multiple Reliable sources, to form the basis of an article about the subject. It doesn't necessarily require that the subject is 'famous', or in one of many possible senses 'important'.
- Also, the fact that your grandmother's achievements are not yet in Wikipedia doesn't mean Wikipedia asserts they aren't, or she as a whole isn't, notable; merely that no volunteer editor has yet gathered the necessary sources and added the information (with citations), whether in her husband's article or in a draft for her own. I encourage you to try. Good luck!
- While we're here, I notice that John C. Harrison's article is on shaky ground, because it's entirely cited to a single (though reliable) source: we usually prefer a minimum of three, so if you could flesh out that article with further referenced material, it would be a good thing. {The poster formerly known as 87.81.230.195} 94.1.223.204 (talk) 03:56, 31 December 2024 (UTC)
- Add Ethel Margaret Streit Harrison to: John_C._Harrison_(judge)#Personal_life
- 69.181.17.113 (talk) 04:45, 31 December 2024 (UTC)
- can you find at least 4 WP:RS sources ? 69.181.17.113 (talk) 04:47, 31 December 2024 (UTC)
- She's already mentioned in that section as Ethel Harrison, cited to the article's (only) reference. Even to add her middle names would require a further citation to a published reliable source that mentions them (the existing one does not: I've downloaded and searched it). {The poster formerly known as 87.81.230.195} 94.1.223.204 (talk) 06:56, 31 December 2024 (UTC)
- Being married to JCH can be mentioned in her Personal life section, but does not contribute to establishing her notability. David notMD (talk) 05:34, 31 December 2024 (UTC)
Restoring previously unmerged articles
I would like to create a new article but it once existed back in 2009 and got merged without consensus. Is it okay if I restore the article but with more and better sources from my sandbox later on? Because one user tried to restore a merged article and it got immediately undone because a consensus existed to merge it. I'm not saying that I'll do it right now as it's still unfinished but I need to ask because if I can't then I'll just cancel. Underdwarf58 (talk) 00:29, 31 December 2024 (UTC)
- There's very little about the original subject 'Cyprus–Jordan relations' in Foreign relations of Cyprus that it was merged into, so I think that if you think you have enough Reliably sourced material (bearing in mind there's also been another 15 years of history) you should go ahead and create a new Draft on the subject, via the normal Wikipedia:Articles for creation process, which can be submitted for assessment. Good luck! {The poster formerly known as 87.81.230.195} 94.1.223.204 (talk) 04:11, 31 December 2024 (UTC)
template
Hello, I'm need of some assistance with creating a Wikipedia page about a music artist. Can anyone advise which template to use? RATHOMP (talk) 02:45, 31 December 2024 (UTC)
- look at other Wikipedia pages about music artists ... 69.181.17.113 (talk) 04:40, 31 December 2024 (UTC)
- Hello, IP user, and welcome to the Teahouse. From your question, I suspect that you are focussing on the layout of your proposed article (I'm guessing that that's what you mean by a "template" - we use the word a bit differently here). But while the layout of an article is important, it is MUCH less important than the quality of the sources used. Until you have found adequate sources to establish that the artist meets Wikipedia's criteria for [[WP:notability|}], it's pointless spending any time thinking about the content or the layout. To use a house-building analogy, you may have an idea for what you want your house to look like, and even a plan; but until you've surveyed the site to make sure it's fit to build on, and checked that your plans meet local building regulations, it would be a waste of effort to start building.
- More generally, My earnest advice to new editors is to not even think about trying to create an article until you have spent several weeks - at least - learning about how Wikipedia works by making improvements to existing articles. Once you have understood core policies such as verifiability, neutral point of view, reliable, independent sources, and notability, and experienced how we handle disagreements with other editors (the Bold, Revert, Discuss cycle), then you might be ready to read your first article carefully, and try creating a draft. ColinFine (talk) 10:36, 31 December 2024 (UTC)
- Is there a specific place you're currently working on the article at? I can't seem to find it on your userpage as a subpage. For advice, I'd recommend looking at other music artist articles (specifically various quality articles from The Beatles [FA] to Sepultura [C-class]) for general outlines on how to write it. For infoboxes, use {{infobox musical artist}} and fill out the template using it's template page at Template:Infobox musical artist. For general advice on writing, see Wikipedia's content policies and guidelines, I'd recommend WP:Nutshell as a starting point.
- Thanks, Sparkle and Fade talkedits 08:23, 31 December 2024 (UTC)
how can I rigth a article in wikipedia
how can I rigth Daniel Muanga (talk) 03:49, 31 December 2024 (UTC)
- Wikipedia:Article wizard
- 69.181.17.113 (talk) 04:39, 31 December 2024 (UTC)
- New editors are strongly advised to first gain skills by doing time improving existing articles. References required. David notMD (talk) 05:39, 31 December 2024 (UTC)
Find a Grave = WP:RS ?
Is Find a Grave = WP:RS ?
69.181.17.113 (talk) 04:37, 31 December 2024 (UTC)
- Welcome to the Teahouse. Please consult Wikipedia:FINDAGRAVE. —Tenryuu 🐲 ( 💬 • 📝 ) 05:03, 31 December 2024 (UTC)
- The answer is "no" because Find a Grave consists of user generated content, and is specifically mentioned in the WP:USERGENERATED section of the reliable sources guideline as a source that should not be used as a reference on Wikipedia. That does not mean that Find a Grave is of no value to Wikipedia editors. You may be able to find nuggets in those listings that will inform your searches about various people. Some but not all Find a Grave listings include references to reliable sources, and those sources may be useful as Wikipedia references. One thing that Find a Grave can teach editors is that many people down through the years share the same name, and we need to be very careful to avoid including biographical details about one Andrew Wilson in an article about another Andrew Wilson. That's just one of countless examples. I have been working on Andrew Stephen Wilson today, so that's why I chose that example.. Cullen328 (talk) 06:22, 31 December 2024 (UTC)
Hello -- I would like to start helping, but I'm having trouble finding what to do!
I see lots of support for ultra basics, but I know what the concept is, and how to edit, and how to make italics and hyperlinks, and that I should use a neutral voice, etc. I am trying to find some guidance on WHAT to contribute. I found the Typo Team (or at least, this typo team), but I haven't found guidance on interacting with it. (Do I delete entries if I resolve them? Yesterday I found many entries to check, but today none of the articles seem to HAVE the potential typo that was listed, or even a fixed version). I have found this Growth page, but can't get the features working. For example, it says to enable the Help panel in the Editing tab, but I don't see such a thing in the Editing tab. I also can't find "Display newcomer homepage" in my user preferences. Similar with Suggested Edits -- how can I "use Special:NewcomerTasksInfo
"? Etc., etc. I must be missing some key piece of advice -- where can I figure out how to get things rolling? SKM (talk) 05:08, 31 December 2024 (UTC)
- Hello @Skmccormick. As a volunteer project, you're free to do whatever you wanna do best. Want to copyedit a bunch of articles and bring our grammar up to shape? Join the Guild of Copy Editors and go wild. Wanna fight vandalism? Go patrol Special:RecentChanges and stop those dang vandals! Wanna go help out that typo team? Go right ahead. It's your choice. Tarlby (t) (c) 05:26, 31 December 2024 (UTC)
- @Skmccormick
- You should find display user homepage at the bottom of Special:Preferences, "User profile" tab.
- If I wanted to interact with the typo team, my first place to try would be Wikipedia talk:Typo Team.
- Have you found WP:TASKS and WP:REQUEST? Gråbergs Gråa Sång (talk) 07:28, 31 December 2024 (UTC)
- Thank you! These links are helpful. I found "display user homepage" under User Profile. So maybe a silly question, but: how do I find this user homepage? I don't really go to Wikipedia generically, I usually jump straight to an article. Tasks and Requests seem like what I'm looking for. SKM (talk) 01:47, 1 January 2025 (UTC)
- The link should be right at the top of every page when you're logged in, in the same place/menu as the link to your userpage. -- asilvering (talk) 02:05, 1 January 2025 (UTC)
- @Skmccormick
- @Skmccormick, do you have any particular topics that you'd like to edit about? One way to find a lot of articles that need help is to go through our various maintenance backlogs (Gråbergs Gråa Sång has already linked you to WP:TASKS). Some people are content to plug away at a particular backlog chronologically, but if you prefer to edit on things you're generally interested in, it's helpful to filter these by wikiproject. Alternatively, do you have any particular skills or outside knowledge that might be helpful here? There's always demand for multilingual editors, the copyright folks are always backlogged, etc. -- asilvering (talk) 10:16, 31 December 2024 (UTC)
- Thank you. I would prefer to get into the groove with smaller contributions before I start throwing any weight around. Sadly, I cannot offer multilingual help; I am American. I'm mostly having trouble navigating the various pages and internal tools like TASKS or RecentChanges (anything labeled "Special:" is still new to me). SKM (talk) 01:50, 1 January 2025 (UTC)
- I'm a fan of Category:Wikipedia introduction cleanup as a newbie task - not small, exactly, but hey, no time like the present to learn to WP:BEBOLD. You don't (or shouldn't) need to do any research to fix these articles - most of them are here because they are tagged with "lead too short". Find one of those, read the article, then rewrite the lead so it summarizes it accurately. Then remove the tag. All this requires is good English literacy, and since the lead is what most people read and what is used in the google knowledge box etc, it's a high-impact change that requires very little wiki-knowledge. -- asilvering (talk) 02:16, 1 January 2025 (UTC)
- Thank you. I would prefer to get into the groove with smaller contributions before I start throwing any weight around. Sadly, I cannot offer multilingual help; I am American. I'm mostly having trouble navigating the various pages and internal tools like TASKS or RecentChanges (anything labeled "Special:" is still new to me). SKM (talk) 01:50, 1 January 2025 (UTC)
Different images for Light/Dark mode
Is there any way to tell Wikipedia to display different images / media for Light vs Dark mode users? Due to transparency, some SVG and PNG images have bad contrast when viewed in Dark mode. CrushedAsian255 (talk) 06:04, 31 December 2024 (UTC)
- @CrushedAsian255 This question is likely to have a better audience at WP:VPT. That board has a more technically oriented team than here. 🇺🇦 FiddleTimtrent FaddleTalk to me 🇺🇦 09:26, 31 December 2024 (UTC)
Wider vs specific consensus
Can a case-specific consensus triumph a wider topic consensus already established? Not that consensus over wider topic is changed but maybe because that case is viewed from a different perspective. ExclusiveEditor 🔔 Ping Me! 07:33, 31 December 2024 (UTC)
- @ExclusiveEditor I doubt it. Were that to be the case we would not have consistency of operation. This question is likely to have a better audience at WP:VPP. That board has a more policy oriented team than here 🇺🇦 FiddleTimtrent FaddleTalk to me 🇺🇦 09:28, 31 December 2024 (UTC)
- Hello, @ExclusiveEditor, and welcome to the Teahouse. While I agree with FiddleFaddle's advice, I would also remark that general questions like yours are very frustrating for people who attempt to answer questions here. If you explain the specific issue you want guidance on, you are much more likely to get a useful answer (and also be less likely to be suspected of wikilawyering).
- I am aware of the possible irony in my answer, given your question. ColinFine (talk) 10:43, 31 December 2024 (UTC)
- Thanks for replying! Being a half-host on Teahouse myself I get you. I was inspired to ask this question by the discussion happening here. The latter part of the discussion specifically focuses on if 'Nobel prize' should be added or not in Jimmy Carter's death blurb on Main page's ITN section. It was initially proposed by the nominator and many supported it (albeit not mentioning specifically the Nobel prize) and it got posted without the mention of the prize. However later there was some more scrambling, this time with more regard to the Nobel prize and so it currently updated to include the prize in the blurb. The opposers are generally arguing that it is editorializing and other things. I may not be very good in summarizing discussions, so I left it in the question. --ExclusiveEditor 🔔 Ping Me! 10:58, 31 December 2024 (UTC)
How to upload a >100 MB file to Wikipedia?
I've uploaded plenty of files to Commons before, but I've only uploaded a few (non-free) files to WP. I would like to upload a short film that will become public domain in the US at the start of 2025 (won't be PD in its country of origin for a few years, so no uploading it to commons), but the file is over 100 MB, the maximum file size listed on the upload page. I don't want to compress it any more than it already is, so how to I get around this? I've seen several films large than 100 MB on WP already (1, 2), so it must be possible.
Any help with this is greatly appreciated. Thanks. — Toast for Teddy (talk) 07:48, 31 December 2024 (UTC)
- @Toast for Teddy This question is likely to have a better audience at WP:VPT. That board has a more technically oriented team than here 🇺🇦 FiddleTimtrent FaddleTalk to me 🇺🇦 09:26, 31 December 2024 (UTC)
- @Toast for Teddy That first file you linked was uploaded with the aid of a userscript discussed at Commons:User talk:Rillke/bigChunkedUpload.js. I don't pretend to understand the details, but you may ;-) Mike Turnbull (talk) 14:27, 31 December 2024 (UTC)
- Even if you're uploading "locally", Commons:Upload tools is likely helpful. All the projects use the same MediaWiki software, all that needs adjustment is the destination for the upload. --Slowking Man (talk) 20:54, 31 December 2024 (UTC)
Editing a sandbox, references number is doubling
Hi there! I was editing my sandbox and for some reasons all the different references I am adding are doubling the number of them and the previous ones are not disappeared. I was following your suggestions to add COI edit to my text, eliminating the internal sources and the bold words. Can you help me or it is just a matter of viewing and once I publish the sand box they will all disappear? Andrea Biographer (talk) 09:10, 31 December 2024 (UTC)
- Hi @Andrea Biographer: it seems that with this edit you've duplicated the contents by pasting an earlier edit into the page, thus embedding a copy of the entire page within the page (if that makes sense). You should undo your most recent edits up to and including that one. Or if you'd like me to do it, let me know. -- DoubleGrazing (talk) 09:16, 31 December 2024 (UTC)
- Hi @DoubleGrazing, thanks for your prompt answer! I think you made sense, could you check and fix this for me? I don't want to commit any further mistakes in the editing process or in the COI different templates, thanks in advance! Andrea Andrea Biographer (talk) 09:31, 31 December 2024 (UTC)
- User:Theroadislong has already sorted this out. Also, your draft is now located at Draft:Gridspertise. -- DoubleGrazing (talk) 10:30, 31 December 2024 (UTC)
- Hi @DoubleGrazing, thanks for your prompt answer! I think you made sense, could you check and fix this for me? I don't want to commit any further mistakes in the editing process or in the COI different templates, thanks in advance! Andrea Andrea Biographer (talk) 09:31, 31 December 2024 (UTC)
how to give feedback
i had a question on why an article was written and sent it to info@ wikipedia. they told me that I needed to engage in the "talk" feature and ask the editors, since wikipedia is only a platform. I did that and not only was the answer not given I was ridiculed, because I am not actually sure why. I am not interested in editing wikipedia, I am only interested in engaging with the editors, to understand inconsistencies. How exactly would I do that, if I should not be using the talk feature. thanks Mommer264 (talk) 12:34, 31 December 2024 (UTC)
- Hello. Discussion about an article usually takes place on its talk page. Discussion with an editor directly can occur on their user talk page.
- Note that your only other edit was about the Israeli-Arab/Palestinian conflict, which is a topic area with special rules that I will notify you of on your user talk page. One of those is that you must be an experienced user in order to make edits to any type of page about it. Your account must be 30 days old with 500 edits. 331dot (talk) 12:53, 31 December 2024 (UTC)
- Also answering your query: :You asked a question that is now at Talk:State of Palestine/Archive 21. Talk pages of articles are not for 'open' questions. Instead, the proper method is to propose a specific change of text, as in replace A with B, and include a reference to support your proposal. Given that State of Palestine is a very controversial article, editors who participate there - at both article and talk - can be short on tact. David notMD (talk) 12:54, 31 December 2024 (UTC)
I am not interested in editing wikipedia
If you're not interested in contributing to the project, then you probably ought to go elsewhere. The purpose of Wikipedia is to produce an encyclopedia; notably, talk pages are for constructively discussing the project and its content, not general Internet forums for discussion. There are many many discussion forums elsewhere on the Internet, and Wikipedia can even help direct you to some of them. If you do wish to contribute to the encyclopedia, take a look at WP:Welcome. Thank you and I hope you have a good day. --Slowking Man (talk) 21:03, 31 December 2024 (UTC)
VisualEditor
I've just changed my Preferences > Editing
Turned on "Enable the visual editor" (which, I think, it's on by default in Wikipedia in Italian language)
But i can not find out how to edit with VisualEditor.
I've looked at Help:VisualEditor and it says (Help:VisualEditor#Opening VisualEditor to click on the "Edit" tab (in the picture I can see a "drop down" menu to choose the editor).
But I've not this choice. My tab is named "Edit source". Centrodiurnomilano (talk) 14:47, 31 December 2024 (UTC)
- I've found out, reading Help:VisualEditor#First step: enabling VE (which seems a bit outdated, by the way).
- After I've changed my Preferences > Editing > "Enable the visual editor" = ON
- and the press "save" button at the bottom.
- Only then, a "drop down menu" show up in the same section: "Editing mode", which is setted by default "Remember the last editor". I've changed it to "Show both editor tabs".
- It's not easy to find out. Centrodiurnomilano (talk) 15:04, 31 December 2024 (UTC)
- By the way, I've looked at the settings on Wikipedia in Italian language and "Enable the visual editor" it's enough there (there is no "Editing mode" option at all) Centrodiurnomilano (talk) 15:08, 31 December 2024 (UTC)
- Italian Wikipedia and English Wikipedia run on the same system, MediaWiki, but are two different Wikipedias. So if this is a question about itwiki, then you should go to a help board on itwiki. If this a question about enwiki, you should be able to switch between the source editor and visual editor by pressing the pencil button next to the preview button in the source editor. Cowboygilbert - (talk) ♥ 16:07, 31 December 2024 (UTC)
- But, @Centrodiurnomilano, do note that you can only use the VisualEditor is some namespaces but not all. You can in mainspace, userspace, draftspace but not in wikispace which is the space with the Wikipedia: prefix. Cowboygilbert - (talk) ♥ 16:09, 31 December 2024 (UTC)
- Thank you, @Cowboygilbert . I've tried in main space and the in user space. Centrodiurnomilano (talk) 16:29, 31 December 2024 (UTC)
- Were you able to fix it using the instructions that I put,
you should be able to switch between the source editor and visual editor by pressing the pencil button next to the preview button in the source editor.
? Cowboygilbert - (talk) ♥ 16:30, 31 December 2024 (UTC)
- Were you able to fix it using the instructions that I put,
- Thank you, @Cowboygilbert . I've tried in main space and the in user space. Centrodiurnomilano (talk) 16:29, 31 December 2024 (UTC)
- But, @Centrodiurnomilano, do note that you can only use the VisualEditor is some namespaces but not all. You can in mainspace, userspace, draftspace but not in wikispace which is the space with the Wikipedia: prefix. Cowboygilbert - (talk) ♥ 16:09, 31 December 2024 (UTC)
"No Such Number tone" / "crybaby tone" (type of intercept error)
Hello everyone. This is one of my first times writing a brand-new article, and I'm not sure if this is a notable enough topic for its own page or a subsection in the intercept message.
This technically isn't a message, but before the intercept message was used, a different type of tone could also be returned instead of connecting to an operator. This is known under many different names, such as the "no such number tone" or even the "crybaby tone." This tone would be returned if a caller attempts to dial a number that can't possibly exist according to the numbering plan. The call would not be allowed to go through because some of these numbers could be reserved for private use. The tone itself is continuous, sweeping from 200-400Hz back to 200Hz again over the course of one second.
The issue is that it's almost exclusively used in North America, if not, completely exclusively. It was introduced by the Bell System in 1941, but I've also heard of it in use by the 3CX Asterisk system. However, my intent is to preserve information about this tone, as next to no information seems to exist about it. An excerpt from the Bell Labs Record describes it here. It also seems to be exclusive to crossbar systems.
[2] Bell Labs record
[3] Article about this record
[4] Sample of this sound
What do you think of this? ZetaformGames (talk) 19:01, 31 December 2024 (UTC)
- Hi @ZetaformGames, welcome to the teahouse and welcome to Wikipedia! I think that this could be a good idea as a subsection in the article for Intercept message or some other related article. I don't think that there is enough for a full article with the sources you've posted but could definitely be an interesting paragraph! Justiyaya 07:12, 1 January 2025 (UTC)
- Alright, thank you! And thanks for the welcome. I made this account a while ago, but haven't felt confident enough in my editing skills until now to contribute. ZetaformGames (talk) 17:46, 1 January 2025 (UTC)
Maryam Mirzakhani
Can someone fix the error in the Maryam Mirzakhani article?
Mirzakhani solved this counting problem by relating it to the problem of computing volumes in moduli space—a space whose points correspond to different complex structures on a surface genus Failed to parse (SVG (MathML can be enabled via browser plugin): Invalid response ("Math extension cannot connect to Restbase.") from server "http://localhost:6011/en.wikipedia.org/v1/":): {\displaystyle g} . In her thesis, Mirzakhani found a volume formula for the moduli space of bordered Riemann surfaces of genus g {\displaystyle g} with n {\displaystyle n} geodesic boundary components. From this formula followed the counting for simple closed geodesics mentioned above, as well as a number of other results. This led her to obtain a new proof for the formula discovered by Edward Witten and Maxim Kontsevich on the intersection numbers of tautological classes on moduli space.
Thanks. 76.14.122.5 (talk) 19:28, 31 December 2024 (UTC)
Hi IP 76.14.122.5. Generally, the best place to discuss something like this is on the talk page for the article in question, which in this case would be Talk:Maryam Mirzakhani; however, if you truly believe there's an error in the article, you can be WP:BOLD and fix it yourself if you think you can. Please understand though that "fix it" in this context means to correct the article in accordance with relevant Wikipedia policies and guidelines, and the policies/guidelines most likely applicable in this case are going to be Wikipedia:Verifiability and Wikipedia:No original research. Ideally, you're going to need to find WP:SECONDARY reliable sources (as defined by Wikipedia) to cite in support of the change you want to make; even if you know such changes to be true, you're still going to need to cite reliable sources in support to allow others to verify the changes. If you just make a change without providing any citations to a reliable source in support, there's a good chance the change will be undone by another user. Given that this seems to be related to mathematics, you might argue that "proving" the information to be incorrect based on Mirakhani's academic thesis is more than sufficient in and of itself, but Wikipedia generally requires something more and a thesis is going to be, for the most part, considered a WP:PRIMARY source and could have other issues as explained in WP:SCHOLARSHIP. You could also try asking about this at Wikipedia:WikiProject Mathematics since that's where you're going to have a better chance of finding someone sufficiently versed in mathematics who might be able to help sort this out. -- Marchjuly (talk) 19:52, 31 December 2024 (UTC);post edited. -- 22:29, 31 December 2024 (UTC)- Please disregard my reply IP 76.14.122.5. I misunderstood what you were asking about. ColinFine's suggestion below seems to be the best course of action here. -- Marchjuly (talk) 22:29, 31 December 2024 (UTC)
- Hello, IP user. What you are reporting is a problem in the software, the network, or the user interface, and not in the content of the article. Generally, technical problems of this sort are better handled at WP:VPT than here. However, I'm not seeing that problem, either on the browser on my laptop, or on the Android app. Is it repeatable, or might it have been a temporary glitch?
- Actually, now I look at it, the URL above appears to be a local proxy, so it may be that whoever manages your local network has not configured the proxy in a way that Wikipedia requires. Again, WP:VPT is a better place to ask about this.
- @Marchjuly. The problem that the IP is reporting is obviously a technical one, so your answer is entirely off the point. ColinFine (talk) 20:21, 31 December 2024 (UTC)
- Thanks for catching my mistake and pointing the OP in the right direction ColinFine. I've stricken my reply so as to not confuse the OP or anyone else. -- Marchjuly (talk) 22:29, 31 December 2024 (UTC)
Stuck on Puerto Rico outages notability.
Hi, new to Wikipedia here and probably gonna create a draft on the Puerto Rico power outage crisis but wanted to ask: I’m stuck in the notability of this topic, so, is the Puerto Rico power outage crisis notable enough for Wikipedia? By power outage crisis I mean the beginning of the Puerto Rican outages from Hurricane Maria to now since it has lasted multiple years with sustained media coverage when an outage does occur. Cheers! 66.50.50.222 (talk) 20:29, 31 December 2024 (UTC)
- Rather than a new article, you could expand on the outages already documented in Puerto Rico Electric Power Authority and in LUMA Energy. Schazjmd (talk) 20:44, 31 December 2024 (UTC)
- A search within Wikipedia on "Puerto Rico power outage" yields a list of several articles, including Puerto Rico Electric Power Authority and in LUMA Energy David notMD (talk) 21:15, 31 December 2024 (UTC)
- Noted. Will expand those articles then instead. Thanks! 66.50.50.222 (talk) 21:40, 31 December 2024 (UTC)
- My first impression would be that an article is likely warranted. PR is an island of ~3 million people, in the same neighborhood as Los Angeles. If Los Angeles were having sustained multi-year-long Issues with electrical service, there would be an article about it. Non-English language sources are perfectly acceptable for citing in articles, as long as a little care is observed.
- Note, to the anonymous editor: if you create an account you get your own shiny neato userspace to use mostly at your leisure, where for instance you can work on draft articles with no hurry. I have one underway in mine in fact. And thank you again for being interested in contributing to Wikipedia!
- (Regarding LA: some may have had come to mind the California energy crisis, but, neat fact, LA actually escaped impact from that because of having its own municipal utility with its own generation capacity!) --Slowking Man (talk) 01:38, 1 January 2025 (UTC)
- Note to User:Slowking Man: The userspace is the wrong place to draft an article. Either use your Sandbox or else follow instructions at WP:YFA to create a draft. David notMD (talk) 03:33, 1 January 2025 (UTC)
- Is "userspace" not the colloquial term for "your user page and any and all subpages of it"? The "official" user sandbox link is Special:MyPage/sandbox, which takes you to the /sandbox subpage of your user page. --Slowking Man (talk) 05:29, 1 January 2025 (UTC)
- Hello, the IP editor (66.xx) here, just created my account, and I mainly refrained from creating an article due to being worried about a potential conflict-of-interest (I live in Puerto Rico myself). I might work on a draft later today and collect sources (as @BusterD suggested), thank you all btw for helping me clear up this question I’ve had for some time now! Atheions (talk) 04:11, 1 January 2025 (UTC)
- Note to User:Slowking Man: The userspace is the wrong place to draft an article. Either use your Sandbox or else follow instructions at WP:YFA to create a draft. David notMD (talk) 03:33, 1 January 2025 (UTC)
- On the other hand, User:Slowking Man is correct that an article might be created on this newsworthy subject and that one shouldn't necessarily rely strictly on English-language sources. Looking at the existing material, it certainly seems a sequence of outages could be established from some of the reliable sources already applied to pagespace. I agree with User:David notMD that WP:YFA is a place to consider how to start a new page. I would start collecting sources, online and in print. BusterD (talk) 03:58, 1 January 2025 (UTC)
- Apologies if jumping the gun or creating a draft too early, but I have created one. Won’t be able to do much progress today but will def collect sources to use. Atheions (talk) 07:32, 1 January 2025 (UTC)
Am I allowed to post something about a game I am making on Wikipedia?
Am i allowed to post somthing about a game i am making? On Wikipedia Aaronfart14 (talk) 22:09, 31 December 2024 (UTC)
- Allowed? Yes. Good idea? No. Likely to remain on Wikipedia? No. Writing an article is difficult, particularly for new editors. And since this is about your game, you have a conflict of interest in writing about it. See H:YFA and WP:COI. Simply posting information about your game rather than writing an article would be promotion. See WP:PROMOTION Meters (talk) 22:20, 31 December 2024 (UTC)
- Aaronfart14, the relevant content guideline is WP:Wikipedia is not for things made up one day. Cullen328 (talk) 02:36, 1 January 2025 (UTC)
- If your game goes public, and if it becomes so popular that people are publishing about it, then there is a chance that someone other than you will create an article about it. David notMD (talk) 03:37, 1 January 2025 (UTC)
- Aaronfart14, the relevant content guideline is WP:Wikipedia is not for things made up one day. Cullen328 (talk) 02:36, 1 January 2025 (UTC)
Writing about Childhood and Early Life
I am writing an article about Andrea Sheridan Ordin, who is already included in two existing wikipedia articles entitled, "List of first women lawyers and judges in California" and "United States District Court for the Central District of California." I am interviewing her personally and have reputable sources about her career notability, but I'm not sure how to write her "early life" section, since there are not many sources describing her childhood aside from her firsthand account. How to I write about her early life without secondary sources? Aharten97 (talk) 22:17, 31 December 2024 (UTC)
- @Aharten97: It's not possible without reliable secondary published sources. For the policies, see WP:V. Personal notes from an interview are neither published nor reliable, and self-published statements are rarely reliable. If it's noteworthy it will have been published somewhere. If not then it isn't. -- zzuuzz (talk) 22:45, 31 December 2024 (UTC)
- https://ethics.lacity.gov/news/murray-and-ordin-re-elected-as-ethics-commission-leaders/ provides some info for an Early life and education section. David notMD (talk) 04:24, 1 January 2025 (UTC)
What to write on your talk page?
Above question HELSINKI!233 (talk) 22:32, 31 December 2024 (UTC)
- Hi and welcome, @HELSINKI!233. Your talk page is where other editors can leave messages for you or begin conversations with you. You can learn more at the guidelines for user pages. Schazjmd (talk) 22:37, 31 December 2024 (UTC)
- It is not for chat, or your thoughts/opinions about stuff. Also, with a few exceptions, you are allowed to delete content from your Talk page, although some people prefer to archive older content instead. David notMD (talk) 04:19, 1 January 2025 (UTC)
Citations about movies
I'm trying to improve The Lincoln Lawyer (film)
1) Is IMDB considered a good citation for the cast list, producer name, etc?
2) How do I add a citation to an existing infobox? (Visual or Source editor)
Many thanks
Ben (talk) 22:36, 31 December 2024 (UTC)
- Hi and welcome, @Littenberg. Imdb is not considered a reliable source because much of its information is user-generated. You can learn how to add citations at Easier Referencing for Beginners. Schazjmd (talk) 22:39, 31 December 2024 (UTC)
- If you click on Edit at the top menu it allows you to edit the entire article, including the Infobox. David notMD (talk) 04:15, 1 January 2025 (UTC)
How do I create a Wikipedia article
How do I create an article on Wikipedia? Красный Октябрь (talk) 23:12, 31 December 2024 (UTC)
- Здравствуйте, @Красный Октябрь, and welcome to the Teahouse. My earnest advice to new editors is to not even think about trying to create an article until you have spent several weeks - at least - learning about how Wikipedia works by making improvements to existing articles. Once you have understood core policies such as verifiability, neutral point of view, reliable, independent sources, and notability, and experienced how we handle disagreements with other editors (the Bold, Revert, Discuss cycle), then you might be ready to read your first article carefully, and try creating a draft. ColinFine (talk) 23:33, 31 December 2024 (UTC)
- Can you help me learn how to make one? I want to make an article on the Kazan bombing that happened about a 1.5 week ago. Красный Октябрь (talk) 23:50, 31 December 2024 (UTC)
- Красный Октябрь, please be aware that the Russo-Ukrainian War is a designated contentious topic. You cannot write new content about that war until your account is Extended confirmed, which means the account is over a month old (it is) and has made over 500 constractive edits (you have a long way to go). Cullen328 (talk) 02:56, 1 January 2025 (UTC)
- Then can someone else make an article about it? Красный Октябрь (talk) 04:08, 1 January 2025 (UTC)
- Teahouse Hosts are here to advise, but not to be authors or co-authors. David notMD (talk) 04:13, 1 January 2025 (UTC)
- Who do I ask about it? Красный Октябрь (talk) 08:59, 1 January 2025 (UTC)
- Красный Октябрь, if you persist in discussing a designated contentious topic such as the Russo-Ukrainian War before you are extended confirmed, you may be blocked. So, please edit other topic areas until then. Cullen328 (talk) 09:16, 1 January 2025 (UTC)
- You could ask on Talk:Russo-Ukrainian War. Normally such talk pages are protected, but that one doesn't seem to be at this time. What might happen is that someone adds a paragraph about the incident to the existing article rather than create a new article.
- I'll add that editors who aren't extended-confirmed generally aren't permitted to use the talk pages of such articles either, and I have always disagreed with this, mainly because constructive edit requests end up in WP:RFED, making extra work for administrators. ~Anachronist (talk) 16:34, 1 January 2025 (UTC)
- Who do I ask about it? Красный Октябрь (talk) 08:59, 1 January 2025 (UTC)
- Teahouse Hosts are here to advise, but not to be authors or co-authors. David notMD (talk) 04:13, 1 January 2025 (UTC)
- Then can someone else make an article about it? Красный Октябрь (talk) 04:08, 1 January 2025 (UTC)
- Красный Октябрь, please be aware that the Russo-Ukrainian War is a designated contentious topic. You cannot write new content about that war until your account is Extended confirmed, which means the account is over a month old (it is) and has made over 500 constractive edits (you have a long way to go). Cullen328 (talk) 02:56, 1 January 2025 (UTC)
- Can you help me learn how to make one? I want to make an article on the Kazan bombing that happened about a 1.5 week ago. Красный Октябрь (talk) 23:50, 31 December 2024 (UTC)
Why are padlocks not automatically added when an article is protected?
Sorry if this is the wrong place. Heyaaaaalol (talk) 02:38, 1 January 2025 (UTC)
- @Heyaaaaalol Because an administrator needs to add a protection notice (the padlocks) to a page in order to show the padlock on pages. Sometimes the padlocks do not get placed on a page, especially if it's a user page, unless if the protection notice is placed by an administrator. NicePrettyFlower (talk) 03:35, 1 January 2025 (UTC)
- Heyaaaaalol, I have done some page protection myself. Usually when I apply protection, I wish to see the "padlock" icon applied. Sometimes when I use the protection script, I forget to click the toggle which leaves the padlock icon. In that case, there's a bot which usually fixes that mistake automatically. If you see a protected page without an icon, you might tell someone. Do our answers help? BusterD (talk) 03:44, 1 January 2025 (UTC)
- Hello, @Heyaaaaalol. To actually answer your question: because nobody has implemented that function in the software. I don't know whether there is a technical reason for that, or whether it's just that nobody's got round to it. Questions about the software and user interface are better asked at the Village pump: either WP:VPT or WP:VPR ColinFine (talk) 13:34, 1 January 2025 (UTC)
- @Heyaaaaalol: Protecting a page is a log action in the MediaWiki software which powers Wikipedia and thousands of other wikis. It does not make an edit to the page. Logs are separate from edits. Displaying a padlock with a link is a Wikipedia practice. We do it by editing the page and adding special code which places a padlock in the corner instead of the normal text area. Other wikis may use no or other symbols for protected pages, place them in other places, and make no or other links on them. A MediaWiki feature to automatically display a symbol on protected pages was recently added at phab:T12347 but it's disabled by default. I haven't examined how flexible it is but I guess it would be non-trivial for us to convert to using it when we already have a well-functioning system. There is so far only a single Wikimedia wiki which has set
wgEnableProtectionIndicators
to true in https://noc.wikimedia.org/conf/highlight.php?file=InitialiseSettings.php. PrimeHunter (talk) 15:08, 1 January 2025 (UTC)
Multiple non-free album covers on one page
For the album Breakfast with Girls, there's an associated EP located at the section Brunch. The EP has an infobox but is currently without cover artwork, so I'm wondering if it's appropriate for the article to have another non-free image when one is already in use for the album's artwork. If not within free use, Brunch's artwork (seen here) is mostly text on a black background, so would I be allowed to crop out the non-text part and use that as a public domain text logo image in an infobox? Koopastar (talk) 04:50, 1 January 2025 (UTC)
- Hi Koopastar, I do think that another cover would be within fair use. There's also been instances of this in the past like In Rainbows#In Rainbows Disk 2. Happy editing! Justiyaya 06:59, 1 January 2025 (UTC)
afd
I really like a certain article, but it's afd, if it is deleted, is there a way to still view it 🐢 (talk) 05:24, 1 January 2025 (UTC)
- History checker SimpleSubCubicGraph (talk) 05:26, 1 January 2025 (UTC)
- If the article is still "public" currently, you can save a copy for yourself: Download as PDF (maybe see also Help:Export). Otherwise go to: CAT:RESTORE. --Slowking Man (talk) 06:14, 1 January 2025 (UTC)
- This appears to be about X11 color names, which has been in existance since 2003 and has been edited more than 700 times since then. While it is unlikely that it will be deleted, as noted, you can save a copy to your computer. David notMD (talk) 13:55, 1 January 2025 (UTC)
Hypothetically, could it be possible to write a netural autobiography?
Hypothetically, would it be possible for someone on Wikipedia to write a neutral autobiography of themselves? Showing the good and bad and making no favor to either side, providing proof and checking all of the boxes. How would that go and would it be accepted? What if say a Wikipedia administrator that is not really well known becomes for example the president of the US? Can they have oversight over their own article? Are they removed from their position? Can they no longer edit anything involving US politics due to their inherent bias? And finally has there been any real examples of this over the past 20+ years? SimpleSubCubicGraph (talk) 05:25, 1 January 2025 (UTC)
- Hello, SimpleSubCubicGraph. Hypothetically, what you describe is possible but the president of the US example is implausible because credible candidates for that office would already be the subject of a Wikipedia article. More plausible would be a longtime editor elected to a state or provincial legislature. I see no reason why that person could not submit an autobiography through Articles for Creation with full disclosure, have the article accepted and continue as an editor or even as an administrator. They should certainly recuse from the current legislative affairs of their state or province but otherwise I would not see a broader problem. As for whether anything like that has ever happened, I do not know. Maybe another editor does. Cullen328 (talk) 06:49, 1 January 2025 (UTC)
- @SimpleSubCubicGraph welcome to the Teahouse! Interesting questions. I would say that while it is hypothetically possible, guidelines still strongly discourage creations of autobiographies and I haven't seen an instance of it being successful outside of early Wikipedia. This would have to go through articles for creation and the editor should declare their conflict of interest. Subjects do not have oversight of their own article.
- I would argue that if someone is elected president of the US they would have a financial relationship with the federal government and should stay away from editing those topics. I think they would have potentially less of a conflict of interest with state governments and historical united states politics. I don't think one would be removed as an administrator because they are elected to a public office here. I don't know of any Wikipedian that has been elected. CongressEdits is probably the closest to an example of this. Justiyaya 06:46, 1 January 2025 (UTC)
- Succeeding at autobiography is more likely to occur in an area such as WP:NACADEMIC. A senior professor at a university would have as models articles about other professors at their university. David notMD (talk) 14:02, 1 January 2025 (UTC)
- The only time I recall a good autobiography being written was when the CEO of a small business wrote one, submitted it for review, and it was accepted after some minor revisions. So it is possible.
- By now I've gained enough experience on Wikipedia that I could probably write a neutral biography about myself, but because I am not notable, there's no point. ~Anachronist (talk) 16:26, 1 January 2025 (UTC)
- a) It would probably go over reasonably well, presuming they followed the Correct way to do that kind of thing: don't create your own bio article (meaning here the literal "creating a new page in Main namespace") and don't edit it directly but put up stuff on the Talk page for others to evaluate, revise, and put into the article if they decide.
- b) No, no one "owns" articles and gets special "powers" over them. Since all Wikipedia content is "free as in freedom" you and anyone else can copy it put it up elsewhere and do whatever with the copies, as long as you credit the original creators.
- c) Why would they get adminship removed if they haven't misused it?
- d) They probably ought to stay away from US politics content yes, being rather WP:INVOLVED. Also realistically the POTUS is not going to have ample free time to devote to Wikipedia contributing, or to be inclined to devote what little precious free time they get to, doing more work.
- e) Have a look at WP:List of Wikipedians with articles. --Slowking Man (talk) 17:26, 1 January 2025 (UTC)
Wiki page written by subject or friend of subject?
Hello, I saw a page on Wikipedia that looks like it was, based on the way it's presented, mostly written by the person who is the subject of the page, or a close friend. What is the standard flag or way to raise this on the page's talk page? Thank you for your help. FireBatV (talk) 08:05, 1 January 2025 (UTC)
- Hello @FireBatV! Welcome to the Teahouse. The standard way is to tag the article with the Template:COI on the main article page. Alternatively, you can use Twinkle to tag it as well. TNM101 (chat) 12:17, 1 January 2025 (UTC)
- Thank you for the help! I've added the
{{Template:COI}}
to the page in question (Rhett Ayers Butler) and as per the instructions on the template page I am have also added{{Connected contributor}}
to the talk page, so editors can take it from there. Thank you again for the help and the friendly welcome, my issue has been resolved. FireBatV (talk) 21:13, 1 January 2025 (UTC)- No problem! Feel free to reach out if you need any more help TNM101 (chat) 06:46, 2 January 2025 (UTC)
- Thank you for the help! I've added the
Editing the Wikipedia page of "The crown"
i want to edit the Wikipedia page of "the crown" on netflix why i wont add the seventh season of "the crown" if you cant tell the staff at netflix please add aditional information like the royal wedding of prince william and kate Middleton and the queen involvement in the 2012 summer olympics in london england alongside James bond!!! 89.128.137.159 (talk) 14:08, 1 January 2025 (UTC)
- Hello IP Editor: there was no 7th season of The Crown, so your additions were inappropriate and were removed. Please don't add fake information to Wikipedia. qcne (talk) 14:18, 1 January 2025 (UTC)
- False information is vandalism, which if continued can lead to your IP address being blocked and any account your subsequently register being blocked. David notMD (talk) 15:20, 1 January 2025 (UTC)
The show button doesn't work on phone
Hi. Some articles have this dialouge box on top of them. But when we touch the show button on phone (Chrome for Android for me), nothing happens. I don't know about desktop. Aminabzz (talk) 15:38, 1 January 2025 (UTC)
- What is an example of such an article? What happens when you use the Wikipedia app? Does it work then? ~Anachronist (talk) 16:21, 1 January 2025 (UTC)
- Hello, @Aminabzz, and welcome to the Teahouse.
- I agree. If I look at Amathus, I get that box (from template {{expand language}}) at the top, but on a browser on my Android phone, when I pick "Show" it replaces the word "Show" with "Hide", but doesn't expand the box, so I can't see additional information.
- However, the box has a "Learn more" button, and if I pick that it shows me a little more, but not the full information that I see on my browser on a computer.
- This looks like a bug in the user interface: WP:VPT is a better place to ask/report such things than here, and I suggest you post there. Thank you for pointing it out. ColinFine (talk) 18:04, 1 January 2025 (UTC)
How to find the full form of abbreviations on phone?
Hi. On Wikipedia, some abbreviation words have dotline underlines. When we hover the mouse cursor on them on PC we can see the full form. For example, TBA reveals to be "to be announced".
But on mobile phones (Chrome for Android for me) there is no mouse so that we hover the cursor on them! So how can we find the full form of abbreviations on these dotline-underlined words in cellphones? Holding the word doesn't work.
Look at this for seeing the dotlines. Aminabzz (talk) 15:40, 1 January 2025 (UTC)
- That's one of the limitations of using a smartphone as a browser. There is no notion of "hover". I have seen hovering implemented in some Samsung phones, in which holding your finger near to the screen without touching it is sensed as a hover, but this worked only in certain apps and wasn't a universal experience across all apps on the phone. Unless someone has a better answer, I'd say that features reliant on hovering are generally not accessible on smartphones. ~Anachronist (talk) 16:16, 1 January 2025 (UTC)
- Samsung devices with S Pen support actually support "hover" by well, hovering the S Pen close to the display. The display has an "active digitizer" that can sense the Pen with NFC. Shows a little cursor and pops up stuff and you can hit the Pen's button to do things, very neat actually. ...Buuuut still doesn't work for the abbr stuff in browsers because it's handled differently in the system software (as it's not "really a mouse", it's handled by different code) and so doesn't "pass through" a mouse hover event down to the browser software. Shucks. --Slowking Man (talk) 17:07, 1 January 2025 (UTC)
- Oh yeah this is a known problem for years. Abbr and friends use the standard HTML tags for such things, and browsers don't gaf apparently about making them "work" when there's no pointing device (mouse), and the attitude appears to be ¯\_(ツ)_/¯. WP could implement some sort of JavaScript "workaround" that pops up a thing, but, that requires deploying something "globally" site-wide and that is a Big Deal so it needs Official approval: ask around at WP:VPT whether there's any effort in this direction. (FYI templates can't have JavaScript, it needs to be JS to "dynamically" add new page elements and display them)
- In the meantime the "workaround" is to hit edit and look at the wikitext. Or you could also view the page HTML source and use the browser's "find" to go to the abbreviation which will show its definition. Also also there might be some userscript someone has made to make it pop up, which you can "install" to use while logged-in. (You could also always plug in a mouse/trackpad/etc or connect a wireless one ) --Slowking Man (talk) 17:07, 1 January 2025 (UTC)
- @Aminabzz, Anachronist, and Slowking Man: I couldn't find any such script, so I made it. JJPMaster (she/they) 05:08, 2 January 2025 (UTC)
GA Spotcheck
Apparently I might've misunderstood what is needed for a source spotcheck in a good article review. Could someone please explain what is needed? History6042😊 (Contact me) 15:53, 1 January 2025 (UTC)
- Appears that at Wikipedia talk:Good article nominations, date 1/1/25, there has been a question raised about GA nomination reviews conducted by History60432. David notMD (talk) 17:07, 1 January 2025 (UTC)
- Yes, that is why I am asking. History6042😊 (Contact me) 17:09, 1 January 2025 (UTC)
- @History6042, note 2 on the Wikipedia:Good article criteria page explains source checking. If you're concerned that your interpretation isn't in line with the community's, you might find it helpful to discuss with other editors involved with the good article process at Wikipedia talk:Good article nominations. Schazjmd (talk) 17:33, 1 January 2025 (UTC)
- So it means Verifiable with no original research:
- it contains a list of all references (sources of information), presented in accordance with the layout style guideline;
- reliable sources are cited inline. All content that could reasonably be challenged, except for plot summaries and that which summarizes cited content elsewhere in the article, must be cited no later than the end of the paragraph (or line if the content is not in prose);
- it contains no original research; and
- it contains no copyright violations or plagiarism? History6042😊 (Contact me) 19:25, 1 January 2025 (UTC)
- The note says
"Ideally, a reviewer will have access to all of the source material, and sufficient expertise to verify that the article reflects the content of the sources; this ideal is not often attained. At a minimum, check that the sources used are reliable (for example, blogs are not usually reliable sources) and that those you can access support the content of the article (for example, inline citations lead to sources that agree with what the article says) and are not plagiarized (for example, close paraphrasing of source material should only be used where appropriate, with in-text attribution if necessary)."
Schazjmd (talk) 20:40, 1 January 2025 (UTC)- Okay I think i did that but do I just need to like write it down? History6042😊 (Contact me) 21:06, 1 January 2025 (UTC)
- Saying in your review what you checked is helpful. Perhaps read through this discussion on spot checks. You might also find it useful to read through some GA reviews by experienced reviewers, see what they're doing in their written reviews that you can learn from. Schazjmd (talk) 21:47, 1 January 2025 (UTC)
- Thank you. History6042😊 (Contact me) 00:39, 2 January 2025 (UTC)
- Saying in your review what you checked is helpful. Perhaps read through this discussion on spot checks. You might also find it useful to read through some GA reviews by experienced reviewers, see what they're doing in their written reviews that you can learn from. Schazjmd (talk) 21:47, 1 January 2025 (UTC)
- Okay I think i did that but do I just need to like write it down? History6042😊 (Contact me) 21:06, 1 January 2025 (UTC)
- The note says
Creating an article heavily lacking sources :/
I've been working on trying create an article (Draft:Millennium Force's effects) but I've run into some issues; including after initial submission. The only sources I can find are mainly from various YouTube videos (not tied to the subject) and very few separate links; so it's no wonder why it wasn't accepted.
So I know what I've written down is true, but I don't have the secondary sources to prove it and that makes my info Original Research. What can I do? I was told I could try another Wiki (ex: Amusement Park Wiki) but is there anything I can do to keep it on Wikipedia? Thanks! Therguy10 (talk) 16:49, 1 January 2025 (UTC)
- @Therguy10 Welcome to theTeahouse. I'm afraid not. Reliable sources are fundamental to Wikipedia articles. See WP:42. Shantavira|feed me 18:01, 1 January 2025 (UTC)
- Hello, @Therguy10. I agree with Shantavira. Please see also No amount of editing can overcome a lack of notability ColinFine (talk) 18:07, 1 January 2025 (UTC)
- @Shantavira @ColinFine Thank you both! I do believe that the article has potential in the future, but I also understand it isn't notable enough as of now. With the article being a history of a subject, there is older information that I fear may not be written about. My hope is that the new changes made this year were important enough to spark some kind of interest for a source; I'll have to wait and see. Therguy10 (talk) 18:13, 1 January 2025 (UTC)
- well i guess technically the changes were made last year...lol Therguy10 (talk) 18:14, 1 January 2025 (UTC)
- If you can find reliable sources, why not improve the Ride experience section of Millennium Force? Schazjmd (talk) 18:27, 1 January 2025 (UTC)
- That was a strong consideration of mine. I certainly won't be able to fit everything inside of the main article but it might not hurt to add a little more than what I already have there. Once again my biggest issue would be collecting sources; this seems more doable. Thanks! Therguy10 (talk) 18:34, 1 January 2025 (UTC)
- If you can find reliable sources, why not improve the Ride experience section of Millennium Force? Schazjmd (talk) 18:27, 1 January 2025 (UTC)
- well i guess technically the changes were made last year...lol Therguy10 (talk) 18:14, 1 January 2025 (UTC)
How do i delete a Wikepedia page?
a school has a wikepedia page, (carlisle public schools) i have moved the page to inside the town page (carlisle masssachusetts) so how do i delete the old school page? it is useless (i moveed the page because it contains about only 1 paragraph, so why not make it a section) so yeah, if someone knows how to delete a page, please delete Carlisle Public Schools
how to find page - go to the education part of carlisle, massachusetts, press carlisle public schools, and you will be there. Theawezomefriend12 (talk) 19:39, 1 January 2025 (UTC)
- Carlisle Public Schools is an article. If you believe it should be deleted, the process is to nominate it for deletion via the WP:AFD process. What you added to Carlisle, Massachusetts was rightfully deleted because you did not include any references. See Maynard, Massachusetts for an example of a referenced education section. I was going to point you to Acton or Sudbury, but those have unreferenced content. There is also Concord-Carlisle High School, but that has its own problems. David notMD (talk) 20:02, 1 January 2025 (UTC)
- Okay, Thanks! Theawezomefriend12 (talk) 20:30, 1 January 2025 (UTC)
- @Theawezomefriend12: I'm going to offer some slightly different advice. For a small article in this situation: First copy the relevant text to the target article (the article about the town). If there are no references then be sure to add one or some. See WP:CWW for the correct process, if you are moving any text. Once that has been successfully done, turn the former article (the school district) into a redirect to the town as you did previously. There is no need to actually delete anything: redirects are useful for people trying to find things, such as information about Carlisle Public Schools, and can be safely left behind. -- zzuuzz (talk) 21:12, 1 January 2025 (UTC)
- Thanks for the advice, i will remove the deletion (i dont know if thats possible i am new) and make the page redierect to Carlisle, Massachusetts. Theawezomefriend12 (talk) 21:30, 1 January 2025 (UTC)
- @Theawezomefriend12: I've helped a bit with the deletion part. The best way to learn is to just do things, so I'll leave you to continue the process (and will probably pop back in a bit). I'll just mention that it appears that the only reason your previous attempt was reverted was because you didn't include any references for the content you were adding/moving to the town article. Sufficient references probably exist in the district article so they can just be copied. If not, I'm sure they won't be difficult to find. -- zzuuzz (talk) 21:38, 1 January 2025 (UTC)
- oh ok, i am propably going to be moving the main description and the new gallery part i added, i probably will not be added refrences and external links. Theawezomefriend12 (talk) 21:45, 1 January 2025 (UTC)
- @Theawezomefriend12: I've helped a bit with the deletion part. The best way to learn is to just do things, so I'll leave you to continue the process (and will probably pop back in a bit). I'll just mention that it appears that the only reason your previous attempt was reverted was because you didn't include any references for the content you were adding/moving to the town article. Sufficient references probably exist in the district article so they can just be copied. If not, I'm sure they won't be difficult to find. -- zzuuzz (talk) 21:38, 1 January 2025 (UTC)
- Thanks for the advice, i will remove the deletion (i dont know if thats possible i am new) and make the page redierect to Carlisle, Massachusetts. Theawezomefriend12 (talk) 21:30, 1 January 2025 (UTC)
- @Theawezomefriend12: I'm going to offer some slightly different advice. For a small article in this situation: First copy the relevant text to the target article (the article about the town). If there are no references then be sure to add one or some. See WP:CWW for the correct process, if you are moving any text. Once that has been successfully done, turn the former article (the school district) into a redirect to the town as you did previously. There is no need to actually delete anything: redirects are useful for people trying to find things, such as information about Carlisle Public Schools, and can be safely left behind. -- zzuuzz (talk) 21:12, 1 January 2025 (UTC)
- Okay, Thanks! Theawezomefriend12 (talk) 20:30, 1 January 2025 (UTC)
Hi! PARIS
Good morning, good afternoon or good evening. Happy new year 2025. Please how to move to the main space the Draft:Hi! PARIS. This research center in AI is now one of the most important in France : https://www.lesechos.fr/tech-medias/intelligence-artificielle/une-dotation-de-70-millions-pour-la-formation-de-lelite-de-lia-en-france-2096150 and https://www.lemondeinformatique.fr/actualites/lire-hec-et-l-institut-polytechnique-remportent-l-appel-a-projets-cluster-ia-93818.html. Many thanks in advance for your help. Have a great day. 2A01:CB00:B48:9900:1C67:231F:4C2F:9933 (talk) 21:57, 1 January 2025 (UTC)
- On the AoPS Wiki go to special:move. I don't know bout wikipedia tho 73.31.42.97 (talk) 22:22, 1 January 2025 (UTC)
- Sorry its Special:MovePage 73.31.42.97 (talk) 22:23, 1 January 2025 (UTC)
- Bonsoir, IP user.
- 73. is right that the technique for moving pages is to use the Move function (though most people do it from their user-interface, rather than going to Special:MovePage.
- However, not logged in users do not have access to that function.
- I was going to advise you to submit it for review. However, I see that the draft was created in mainspace user BobVillars (since blocked for sockpuppetry) was then moved to Draft space by @Rosguill, was submitted for review by an ipV6 user in the same range as you, then moved to main space again by McSyl, also blocked as a sockpuppet of BobVillars, and moved back to draft space again by @Janhrach.
- Neither you nor anybody else has since made any substantial edits to the draft.
- I am finding it very difficult to assume good faith. I will ask you directly: are you BobVillars/McSyl? If so, you are evading your block, which is not permitted. ColinFine (talk) 22:59, 1 January 2025 (UTC)
- Hi Colin. Thanks for your answer. I don’t know what you are talking about. I am using 5G access so my Ip range is used by million of users. But if it is too complicated no worries, please cancel my request. This AI cluster is one of the most important in France and one of the biggest in Europe. So soon or later it will be back to mainspace. Have a great day. 2A01:CB06:B064:82B5:2C9D:5034:3AC5:1BD4 (talk) 06:21, 2 January 2025 (UTC)
AoPS page
Why isn't there a page for Art of Problem Solving? 73.31.42.97 (talk) 22:15, 1 January 2025 (UTC)
- Hi IP 73.31.42.97. There could be a couple reasons why: (1) nobody thought enough about the book/series to try and create one; or (2) somebody did try to create one, but the subject wasn't deemed to meet Wikipedia:Notability (books) and was either deleted altogether or added to Richard Rusczyk. Art of Problem Solving current WP:REDIRECTs to the "Rusczyk" article and its page history shows that there once was a stand-alone article about the book/series, but it was "redirected" per Wikipedia:Articles for deletion/Art of Problem Solving (2nd nomination). If you feel something has changed since then, you might want to first explain why to the administrator who closed the Articles for Deletion discussion. That administrator's name is Liz. Perhaps by asking at User talk:Liz, Liz can tell you what is needed for the article to be recreated or restored. -- Marchjuly (talk) 22:30, 1 January 2025 (UTC)
Where is Wikipedia year in review for iOS?
I am looking for Wikipedia year in review in iOS where is it? 172.59.25.192 (talk) 22:18, 1 January 2025 (UTC)
- Welcome to the Teahouse, IP user. This isn't something I'm familiar with, but a Google search yields this link which I hope gives you what you seek: https://www.mediawiki.org/wiki/New_Engagement_Experiments/PES_1.3.1:_Wikipedia_insights. Regards, Nick Moyes (talk) 01:56, 2 January 2025 (UTC)
How to find vandalism on Wikipedia?
The closest thing I have found to vandalism is on the page for the number 3, where someone changed references to “3” with “2.” I’m wondering how to find vandalism, so I can revert it. Most of my edits are fixing grammar. Thank you. Heyaaaaalol (talk) 00:18, 2 January 2025 (UTC)
- @Heyaaaaalol Welcome to the Teahouse. If you'd care to read WP:VANDALISM, and especially the section linked from this shortcut: (WP:SPOTVAN), you should have all the answers you need. Monitoring 'Recent Changes' is the best way to spot ongoing vandalism, and you can choose to select only certain types of edits which highlight the most likely problematic edits for you to assess and respond to, and ignore all the honest ones. Thank you for your interest in helping out in this field. Regards, Nick Moyes (talk) 01:50, 2 January 2025 (UTC)
"Series of small edits"
I find it difficult to follow and review edits for valid constructive contributions when an editor makes a "series of small edits" (example: 1, 2, 3) where several of the edits could, IMHO, easily and conceivably be made in a single edit with a single edit summary encompassing all of what is edited. Is there policy or guidelines about this practice that could help me in a talk page discussion I am currently engaged in? Iljhgtn (talk) 00:23, 2 January 2025 (UTC)
- Well, there's WP:Cautious, which touches on not making too large edits. There is probably better material than this, but I can't find it.
- The point has been discussed on the Help/Teahouse desks before, and various editors have said that more smaller edits are better than one all-encompassing one, because if only one or two of several small edits are objected to, they can be reverted individually, but if they're part of a larger edit the whole thing has to be reverted and then it becomes difficult to sort out which details are accepted and which disputed.
- I myself, when reading an article for its content, sometimes make small edits as I go along on noticing typos, misspellings, incorrect grammar, etc. If I were to be deliberately setting out to copyedit the article (I used to be a professional editor) I would likely make them in larger batches, perhaps one per Section or Subsection (basically, as often as [edit] is present in the text), because that makes for a smaller and easier chunk (with all its confusing reference codings) to navigate in the edit box.
- In general, I don't think an editor making good-faith edits need think primarily of the convenience of some hypothetical would-be reverter, and I wouldn't criticise anyone for making successive small edits. Were the edits to smell of some deliberate obfustication as a cover for vandalism, or edit-count inflation, it would be another matter. {The poster formerly known as 87.81.230.195} 94.6.84.253 (talk) 02:11, 2 January 2025 (UTC)
Political Party Relations
Just like how we have , should we have that for political parties kind of like below but for whole countries? These would be based on the infobox political party preceded by/succeeded by/merger of/merged into. If so would these be their own article or would they be in the many "List of political parties in ___"? I am curious and want to know if these are wanted/needed.
SPD | |||||||||||||||||||||||||||
Spartacus League | |||||||||||||||||||||||||||
KPD | |||||||||||||||||||||||||||
SED | |||||||||||||||||||||||||||
PDS | WASG | ||||||||||||||||||||||||||
Die Linke | |||||||||||||||||||||||||||
BSW | |||||||||||||||||||||||||||
History6042😊 (Contact me) 00:39, 2 January 2025 (UTC)
- Hi History6042, that's an interesting idea, you might want to pitch this at Wikipedia talk:WikiProject Politics. Justiyaya 03:23, 2 January 2025 (UTC)
- Okay, I'll do that, thanks. History6042😊 (Contact me) 03:27, 2 January 2025 (UTC)
Freedom of Panorama Inquiries
This is regarding the article Liliget Feast House. The site where the restaurant once stood was torn down 2008, and I wanted to inquire regarding the best way to navigate Fair Use and Freedom of Panorama rules on Wikipedia in order to get photographs of the site on the page. This is probably best asked on Commons, but I'm not familiar with where I might be able to ask on there either. Ornithoptera (talk) 02:19, 2 January 2025 (UTC)
- Hi Ornithoptera. I think you can take a picture of the where the restaurant once stood and upload it here, which should be fine under Freedom of panorama#Canada which also allows incidental inclusion of copyrighted, non-architectural work (also commons:COM:FOP Canada). You would still be able to release a photograph that you took of publicly viewable buildings under a free license.
- If you are asking about whether an inclusion of a non free file is allowed, I'm not sure but I think that it meets all the criteria seeing that the building was demolished and if a suitable free photo cannot be located. Justiyaya 05:48, 2 January 2025 (UTC)
- Hi Justiyaya, thank you so much for your time! Truthfully I'm uncertain about using a photograph of where the restaurant once stood because it would be irrelevant to the content of the article (or at least for the intended use within the infobox). I am more asking about if I am able to use a non free file, yes, is there a venue where I can have a conversation with individuals who are more familiar with the matter? I'm not entirely sure if there is a relevant noticeboard/talk page/etc where I can bring this matter up with, if you or one of the editors here know one do let me know! Ornithoptera (talk) 05:52, 2 January 2025 (UTC)
- @Ornithoptera I think it would be a good use of an non-free file. Wikipedia:Media copyright questions would be a relevant noticeboard for asking these types of questions. Commons won't be the place to go because they don't deal with non-free stuff. Hope this helps and happy editing! :D Justiyaya 06:06, 2 January 2025 (UTC)
- Hi Justiyaya, thank you so much for your time! Truthfully I'm uncertain about using a photograph of where the restaurant once stood because it would be irrelevant to the content of the article (or at least for the intended use within the infobox). I am more asking about if I am able to use a non free file, yes, is there a venue where I can have a conversation with individuals who are more familiar with the matter? I'm not entirely sure if there is a relevant noticeboard/talk page/etc where I can bring this matter up with, if you or one of the editors here know one do let me know! Ornithoptera (talk) 05:52, 2 January 2025 (UTC)
Why am I blocked from doing stuff on Russian Wikipedia?
According to my global account information I've been indefinitely blocked from doing anything other than viewing the Russian Wikipedia because I'm "using multiple accounts or having others promote me (so what I say is seen more correct)", but I'm not? I don't know anyone else with a Wikipedia account and as far as I know I only have one. It's just on the Russian Wikipedia, none of the others. I haven't really done anything on the Russian Wikipedia anyways. I've only edited my own user page. Why did this happen and how do I fix it?
More information: The exact stated reason was обход блокировки (bypass blocking) and it linked here. It also gave me a link to here. Красный Октябрь (talk) 04:25, 2 January 2025 (UTC)
- Hi @Красный Октябрь, welcome to the teahouse. English and Russian Wikipedia are moderated differently, so a block there should not affect editing here nor could the community here fix anything there. From what I've been able to see with google translate and all, it appears that you've been checkuser blocked by @Q-bit array without email or talk page access. The administrator appears to monitor cross wiki notifications so hopefully they would respond here. They also appear to monitor meta:User talk:Q-bit array. Usually appeals without talk page access go through WP:UTRS but I can't seem to find a similar system in ru-wiki. Justiyaya 05:34, 2 January 2025 (UTC)
Notice of geological updates
Hi, in the past few months, I was working on pages regarding the periods, ages and other units of the geological timescale, and noticed a recent update in the ICC Chronostratigraphic Chart (2024/12), compared to the previous version (2023/09), that affects the time boundaries of a lot of ages. I didn't know which talk page or portal I could specifically notify of this change, so I came here to speak about it.
Now, I don't want to linger on a change like this here, since that feels like advertising which I KNOW Wikipedia is not about (see WP:SOAPBOX). But I know a change like this is going to require a lot of edits to catch up with, and I for sure can't do it alone. So does anyone know a good project/portal page I can discuss this more on? — Alex26337 (talk) 05:05, 2 January 2025 (UTC)
- Hi Alex26337, thank you for your question. Wikipedia talk:WikiProject Geology would be the best venue to discuss this. I don't think this is soapbox like if you are advertising on-Wiki collaboration. The policy refers to more so advertising on Wiki things off-wiki. Discussions with other community members that share the same interest is what these Wikiprojects are for :D Justiyaya 05:58, 2 January 2025 (UTC)